KVS TGT Science Solved Papers and Practice Sets
 9415650134

Citation preview

KENDRIYA VIDYALAYA SANGATHAN

KVS TGT SCIENCE SOLVED PAPERS &

PRACTICE BOOK Chief Editor Anand Kumar Mahajan Computer Graphics by Balkrishna, Charan Singh Editorial Office Youth Competition Times 12, Church Lane Prayagraj-211002 Mob. : 9415650134 Email : [email protected] website : www.yctbooks.com Publisher Declaration Edited and Published by A.K. Mahajan printed by printing press Roop Printing Press, Prayagraj. Youth Competition Times 12, Church Lane, Prayagraj In order to Publish the book, full care has been taken by the Editor and the Publisher, yet it is necessary to present your cooperation and suggestions for any errors. Rs. : 295/In the event of any dispute, the Judicial area will be Prayagraj.

CONTENT SOLVED PAPER ■ KENDRIYA VIDYALAYA SANGATHAN, TGT 2013 SCIENCE Solved Paper with explanation (HELD ON 01.06.2014) ............. 5-13 ■ NAVODAYA VIDYALAYA SAMITI, TGT 2022 SCIENCE Solved Paper with explanation (HELD ON 29.11.2022) ........... 14-28

PRACTICE SET Practice Set-1 ............................................................................................... 29-32 Practice Set-1 Solution ................................................................................ 33-39 Practice Set-2 ............................................................................................... 40-43 Practice Set-2 Solution ................................................................................ 44-50 Practice Set-3 ............................................................................................... 51-54 Practice Set-3 Solution ................................................................................ 55-61 Practice Set-4 ............................................................................................... 62-66 Practice Set-4 Solution ................................................................................ 67-73 Practice Set-5 ............................................................................................... 74-78 Practice Set-5 Solution ................................................................................ 79-85 Practice Set-6 ............................................................................................... 86-89 Practice Set-6 Solution ................................................................................ 90-96 Practice Set-7 ............................................................................................. 97-101 Practice Set-7 Solution ............................................................................ 102-108 Practice Set-8 ........................................................................................... 109-113 Practice Set-8 Solution ............................................................................ 114-120 Practice Set-9 ........................................................................................... 121-125 Practice Set-9 Solution ............................................................................ 126-132 Practice Set-10 ......................................................................................... 133-137 Practice Set-10 Solution .......................................................................... 138-144 2

SYLLABUS Definition of matter; solid, liquid and gas; characteristics-shape, volume, density; change of state melting (absorption of heat), freezing, evaporation (cooling by evaporation), condensation, sublimation. Nature of matter: Elements, compounds and mixtures. Heterogeneous and homogenous mixtures, colloids and suspensions. Physical and chemical changes (excluding separating the components of a mixture). Particle nature and their basic units: Atoms and molecules, Law of Chemical Combination, Chemical formula of common compounds, Atomic and molecular masses.

Scheme of Exam for Direct Recruitment of Trained Graduate Teachers The written test is of 180 marks (180 objective type multiple choice questions) carrying 01 mark for each question. The duration of written test will be 180 minutes without any time limit for each part individually. Section Name- Nature of Questions Part I- Proficiency in Languages (20 marks): A. General English B. General Hindi

-

10 questions 10 questions

Part II- General awareness, Reasoning & Proficiency in Computers (20 marks)

Structure of atoms: Electrons, protons and neutrons, Valency, Atomic Number and Mass Number, Isotopes and Isobars.

4. General Awareness & Current Affairs (10 ques.) 5. Reasoning Ability (5 ques.) 6. Computer Literacy (5 ques.)

Chemical reactions:

Chemical equation, Balanced chemical equation, implications of a balanced chemical equation, types of (40 questions) chemical reactions: combination, decomposition, (f) Understanding the Learner(10 questions) displacement, double displacement, precipitation, endothermic exothermic reactions, oxidation and (g) Understanding Teaching Learning(15 questions) reduction. (h) Creating Conducive Learning Environment Acids, bases and salts: (i) School Organization and Leadership Their definition in terms of furnishing of H+ and (15 questions) OH– ions, General properties, examples and uses, (j) Perspectives in Education neutralization, concept of pH scale (Definition relating to Part IV- Subject-specific Syllabus (100 marks) logarithm not required), importance of pH in Part-III: Perspectives on Education and Leadership

everyday life; preparation and uses of Sodium Hydroxide, Bleaching powder, Baking soda, Washing soda and Plaster of Paris. Metals and nonmetals: Properties of metals and non-metals; Reactivity series; Formation and properties of ionic compounds; Basic metallurgical processes; Corrosion and its prevention. Carbon compounds: Covalent bonding in carbon compounds. Versatile nature of carbon. Homologous series. Nomenclature of carbon compounds containing functional groups (halogens, alcohol, ketones, aldehydes, alkanes and alkynes), difference between saturated hydro carbons and unsaturated hydrocarbons. Chemical properties of carbon compounds (combustion, oxidation, addition and substitution reaction). Ethanol and Ethanoic acid (only properties and uses), soaps and detergents.

Professional Proficiency Test: The Professional Competency Test is of 60 marks (including Demo Teaching-30 marks and Interview- 30 marks). Note : The weightage of Written Test & Professional Competency (Demo Teaching: 15 and Interview: 15 will be 70:30 Final merit list will be based on the performance of the candidate in Written Test & Professional Competency Test taken together.

Syllabus for the post of TGT - Science Subject specific syllabus includes the concepts of NCERT/CBSE syllabus and Text Books (Classes VI to X), however, the questions will be testing the depth of understanding and application of these concepts at the level of Graduation. Matter-Nature and Behaviour 3

Work, Energy and Power: Work done a force, Energy, power; Kinetic and Potential energy; Law of conservation of energy). Sound: Nature of sound and its propagation in various media, speed of sound, range of hearing in humans; ultra sound; reflection of sound; echo. Effects of Current Electric current, potential difference and electric current. Ohm's law; Resistance, Resistivity, Factors on which the resistance of a conductor depends. Series combination of resistors, parallel combination of resistors and its applications in daily life. Heating effect of electric current and its applications in daily life. Electric power, Interrelation between P, V, I and R. Magnetic effects of current Magnetic field, field lines, field due to a current carrying conductor, field due to current carrying coil or solenoid; Force on current carrying conductor, Fleming's Left Hand Rule, Electric Motor, Electromagnetic induction. Induced potential difference, Induced current. Fleming's Right Hand Rule, Electric Generator, Direct current. Alternating current: frequency of AC. Advantage of AC over DC. Domestic electric circuits. Food Production Plant and animal breeding and selection for quality improvement and management; Use of fertilizers and manures; Protection from pests and diseases; Organic farming. Natural Phenomena Reflection of light by curved surfaces; Images formed by spherical mirrors, centre of curvature, principal axis, principal focus, focal length, mirror formula (Derivation not required), magnification. Refraction; Laws of refraction, refractive index. Refraction of light by spherical lens; Image formed by spherical lenses; Lens formula (Derivation not required); Magnification. Power of a lens. Functioning of a lens in human eye, defects of vision and their corrections, applications of spherical mirrors and lenses. Refraction of light through a prism, dispersion of light, scattering of light, applications in daily life Our environment: Eco-system, Environmental problems, Ozone depletion, waste production and their solutions. Biodegradable and non-biodegradable substances.

Cell-Basic Unit of life : Cell as a basic unit of life; prokaryotic and eukaryotic cells, multi cellular organisms; cell membrane and cell wall, cell organelles and cell inclusions; chloroplast,

mitochondria,

reticulum, Golgi apparatus;

vacuoles, nucleus,

endoplasmic chromosomes-

basic structure, number. Tissues, Organs, Organ System, Organism: Structure and functions of animal and plant tissues (only four types of tissues in animals; Meristematic and Permanent tissues in plants). Life processes: 'Living

Being'.

Basic

concept

of

nutrition,

respiration, transport and excretion in plants and animals. Control and co-ordination in animals and plants: Tropic movements in plants; Introduction of plant hormones; Control and co-ordination in animals: Nervous

system; Voluntary, involuntary and

reflexaction; Chemical co-ordination: animal hormones. Reproduction: Reproduction in animals and plants (asexual and sexual) reproductive health- need and methods of family planning. Safe sex vs HIV/AIDS. Child bearing and women's health. Heredity and Evolution: Heredity; Mendel's contribution-Law for inheritance of traits: Sex determination: brief introduction evolution. Motion: Distance and displacement, velocity; uniform and non-uniform motion along a straight line; acceleration, distance-time and velocity-time graphs for uniform motion and uniformly accelerated motion, elementary idea of uniform circular motion. Force and Newton's laws: Force and motion, Newton's Laws of Motion, Action and Reaction forces, Inertia of a body, Inertia and mass, Momentum, Force and Acceleration. Gravitation: Gravitation; Universal Law of Gravitation, Force of gravitation of the earth (gravity), Acceleration due to Gravity; Mass and Weight; Freefall. Floatation: Thrust

and

Pressure.

Archimedes'

Principle;

Buoyancy. 4

KENDRIYA VIDYALAYA SANGATHAN TGT SCIENCE, 2013 [Exame Date: 01.06.2014] (Solved Paper with Explanation) 1.

Which of the following is not a chromosomal mechanism for sex determination? (a) Genic balance mechanism (b) Haploid diploid mechanism (c) Metabolic mechanism (d) Single gene mechanism Ans. (c) : Metabolic Mechanism is not a chromosomal mechanism for sex determination, metabolic regulation is the physiological mechanism by which the body takes in nutrients and delivers energy as required (Lipolysis, Gluconeogenesis, Ketogenesis) but chromosomal mechanism animal with two different sex chromosome are of heterogametic sex and produce two types of gamete, and there are 3 types of chromosomal sex determination. • Sex determination by allosomes • Diploid - haploid System • Genic balance System 2. Which assisted reproductive technology has been used for the longest time period? (a) Intracytoplasmic sperm injection (b) Artificial insemination (c) In vitro fertilization (d) Gamete intrafallopian transfer Ans. (b) : Reproductive Technology has been used for the longest time period are Artificial insemination. Long before the modern artificial assisted reproduction techniques were discovered people used Artificial insemination to breed horses and cattles. Artificial insemination is hence the longest used technique that is still around today. 3. Lysosomes containing inactive enzymes are called : (a) Autophagosomes (b) Residual bodies (c) Secondary lysosomes (d) Primary lysosomes Ans. (d) : Lysosomes containing inactive enzymes are called primary lysosomes. Lysosomes are an important cellular organelle that receive and degrade macromolecules from the secretory endocytic, autophagic and phagocytic membrane trafficking pathways, hence primary lysosomes contains inactive enzymes and secondary lysosomes is formed by fusion of primary lysosome with phagocytic vesicle. It contains active enzymes. KVS TGT Science 2013

4.

Transpiration pull will be maximum under which of the following conditions? (a) Open stomata, high humid atmosphere, and well irrigated soil (b) Open stomata, high humid atmosphere, and dry soil (c) Open stomata, dry atmosphere, and moist soil (d) Closed stomata, low light intensity and humid atmosphere Ans. (c) : Transpiration pull will be maximum under open stomata, high intensity of light and well irrigated soil it is essential that stomata is open for transpiration to occur. but for the stomata to remain in an open state, light is essential, high humidity means high water vapor content in the atmosphere and the source of water for transpiration out of the plant comes from soil. 5. Hepatic portal system starts from (a) Digestive system to liver (b) Kidney to liver (c) Liver to heart (d) Liver to kidney Ans. (a) : Hepatic portal system starts from digestive system to liver, hepatic portal system is the system of veins consisting of the hepatic portal vein and its capillaries. It is responsible for directing blood from different parts of the digestive tract to liver. The system begins with the capillaries and veins of digestive system. It collects venous blood from the lower esophagus, stomach, duodenum jejunum, ileum, colon, spleen and delivers it to the liver through portal vein. 6. Ivan Pavlov performed experiments on : (a) Simple reflexes (b) Conditioned reflexes (c) Cardiac reflexes (d) Origin of life Ans. (b) : Conditioned reflexes are acquired reflexes during the life time of an individual. They are absolutely an individual entity and Ivan Pavlov a Russian Physiologist discovered for the first time the existence of conditioned reflexes and therefore, he is called the father of conditioned reflexes. 7. Greenhouse effect is caused by : (a) Green plants (b) Infrared rays (c) UV rays (d) X - rays Ans. (b) : Greenhouse effect is caused by infraredrays the greenhouse gases present in the atmosphere absorb more of the infrared radiation as compared to UV, X, green plants. Which are unable to escape the atmosphere and cause the temperature of the earth to rise, risulting in greenhouse effect hence the correct option will be (b) Infrared rays. 5

YCT

8.

Free living nitrogen fixing bacteria are : (a) Nitrosomonas (b) Clostridium (c) Rhizopus (d) Anabaena Ans. (b) : Free living nitrogen fixing bacteria are Clostridium there are 2 types of nitrogen fixing bacteria, free living bacteria and non free living bacteria example of free living bacteria includes Cyanobacteria like Anabaena and Nostoc, Azotobacter, Clostridium out of these example Azotobacter is aerobic bacterium and Clostridium is anaerobic bacterium so, the correct answer is Clostridium. 9. Eutrophication is caused by excess of ____in water. (a) Nutrients (b) Heavy metals (c) Pesticides (d) Microbes Ans. (a) : Eutrophication is caused by excess of nutrients in water. Eutrophication occurs when the water body becomes overly enriched with nutrient various factors like excessive use of fertilizers untreated sewage, usage of detergents containing phosphorous and Industrial discharge of waste may contribute to eutrophication. 10. Photosynthetic pigment present in plants are : (a) Chlorophyll a and b (b) Chlorophyll a and c (c) Chlorophyll a (d) Chlorophyll b Ans. (a) : Photosynthetic pigment present in plant are Chlorophyll a and b, these 2 type of Chlorophyll are identical in composition apart from one side chain, composed of a - CH3 Chlorophyll a, while in chlorophyll b it is CHO. 11. In aerobic organisms, glucose is converted into _____as a final product : (a) Lactic acid (b) CO2 (c) O2 (d) Ethanol Ans. (b) : In aerobic organism, aerobic respiration is the biological process in which the glucose is converted in the presence of oxygen. Energy is releasing by splitting the glucose molecules with the help of oxygen. At the end, the energy, carbon dioxide and water get releases as the by products. Thus the correct option is 'B' Carbon dioxide. 12. The role of double fertilization in angiosperms is to produce : (a) Integuments (b) Endosperm (c) Endocarp (d) Cotyledons Ans. (b) : In flowering plant germinated pollen grain forms pollen tube. The end the pollen tube ruptures and two male gametes are released the embryosac out of two male gametes one gametes fuses with female gametes which is called fertilization another male fuses with the secondary nucleus and form endosperm. So flowering plants fertilization occurs twice hence it is called double fertilization in endosperm - cotyledons develops by utilizing endosperm. KVS TGT Science 2013

13.

Natural pacemaker in heart is : (a) SA node (b) Purkinje fibers (c) Chordae tendineae (d) Mitral valve Ans. (a) : Natural pacemaker in heart is SA node, the sinus node continuously generates electrical impulses thereby setting the normal rhythm and rate in a healthy heart. Hence the SA node is referred to as the natural pacemaker. 14. The excretory organs in arthropods are (a) Nephron (b) Flame cells (c) Tubefeet (d) Malpighian tubeles Ans. (d) : The excretory organ in arthropods are malpighian tubules, excretory system has two major role in an organism, the first one is the maintenance of homeostasis that is the balance between the internal and external surrounding and the second one is removal of the waste material produced by cells as metabolic activities arthropods also includes crustaceans and arachnid which have paired exretory organ are coxal gland but the major organ of excretion is malphighian tubules. 15. Which part of the brain in human body controls temperature? (a) Cerebellum (b) Medulla (c) Hypothalamus (d) Thalamus Ans. (c) : Hypothalamus contains a number of centres that control body temperature, urge for eating and drinking, sexual drive rage and anger etc. it keeps the body temperature at roughly 37ºC by means of complex thermostat system. So, the Hypothalamus has the centre for controlling the body temperature. 16. Addison's disease is caused by deficiency of a hormone which is secreted by : (a) Pitutiary gland (b) Adrenal cortex (c) Thyroid (d) Thymus Ans. (b) : Addison's disease is caused due to hyposecretion of adrenal hormones. It results from damage to the adrenal cortex. The damage causes the cortex to produce hormones level that are too low. Damage to adrenal cortex can be caused by Infections, such as tuberculosis, HIV, tumour, Hemorrhage into the adrenal glands. Hence, option adrenal cortex is correct. 17. Which of the following is a secondary pollutant? (a) Aerosol (b) CO (c) PAN (d) SO2 Ans. (c) : PAN (Peroxy Acetyl Nitrate) is a secondary pollutant. Pollutants formed by the chemical interaction of primary pollutants with atmospheric gas and moisture often catalysed by sunlight are called secondary pollutant. 6

YCT

(c) Sowing of fast growing crops with a slow growing crop (d) Second crop is sown during growth and often during fruiting of first crop Ans. (d) : Relay intercropping is a type of crop rotation in which one crop is planted while another is still growing. The first crop is usually a fast-maturing crop, such as radishes or lettuce, which is planted alongside a slower - maturing crop, such as tomatoes or corn. This method is used to increase crop yeilds by utilizing the same space for multiple crops, and can also help to control pests and diseases by disrupting their life cycle. 22. The shoots and roots of the plant show (a) Negative and positive geotropism respectively (b) Only negative geotropism (c) Only positive geotropism (d) Positive and negative geotropism respectively Ans. (a) : The movement of a plant part in response to gravity is called geotropism. The plant part moves in the direction of gravity, it is called positive geotropism. The plant part moves agains the direction of gravity, it is negative geotropism.

18.

The layer of earth's atmosphere in which clouds occur is : (a) Stratosphere (b) Troposphere (c) Thermosphere (d) Mesosphere Ans. (b) : The layer of Earth's atmosphere in which clouds occurs is the troposphere. This is the lowest part of the atmosphere the part we live in. It contains most of our wether - cloudes, rain, show. In the troposphere, temperature typically decreases at 5–70C per km. Stratosphere : This layer extends up 50 km an a thin layer of ozone is present at the height of 15 to 30 km. mesosphere : This layer start at 50 km above Earth's surface and goes up to 85 km. The top of the mesosphere is the coldest part of Earth's atmosphere. Thermosphere : It extends from about 90 km to between 500 and 1000 km above the Earth's surface. There is a relatively dense band of charged particles called the ionosphere. 19. Hybrid vigour is due to : (a) Mixing of the male and the female cytoplam (b) Homozygosity of pure characters (c) Superiority of all genes (d) Heterozygosity Ans. (d) : Hetrosis is also called as hybrid vigour. It is the increases in such characteristics as size, growth rate, fertility and yield of hybrid organism over those of its parents. However, if the hybrids are mated together. the increase in heterozygosity is the underlying cause of hybrid vigour. The increase in heterozygosity gives a higher mean value, which is proportional to the increase in the production trait So, the correct answer is heterozygosity. 20. Crosses between two plants by the same variety are called : (a) Intravarietal (b) Intervarietal (c) Interspecific (d) Intergenic Ans. (a) : In Intravarietal hybridization, the crosses are made between plants of the same variety. Thus option A is correct. Intra specific hybridization is a hybridization between two subspecies belonging to the same species. Example - Bengal Tiger with a siberian tiger offspring can be fertile, partially fertile. 21. Which of the following explain relay intercropping : (a) Component crops are totally mixed in the available space (b) Component crops arranged in alternate rows KVS TGT Science 2013

For Example, the shoot of the plant shows negative geotropism, whereas the roots of plant show positive geotropism. 23. Bacteria asexually reproduce by : (a) Amitosis (b) Meiosis (c) Conjugation (d) Transformation Ans. (a) : Bacteria reproduces asexually by binary fission in which cell division is of simple type called Amitosis in which spindle formation does not occur. Amitosis is a type of reproduction in which a single cell divides into two identical daughter cells without the formation of a mitotic spindle or the characteristic condensation of chromatin seen in mitosis. This process occurs in some types of bacteria, such as streptococcus pneumonia. It is also seen in some protozoan and algae. 24.

Retina is most sensitive at

:

(a) Optic disc (b) Periphery (c) Macula lutea (d) Fovea centralis Ans. (d) : The retina is most sensitive at Fovea Centralis. Fovea is the central pit of the yellowish pigmented spot called macula lutea present at the posterior pole of the eye lateral to blind spot. 7

YCT

25.

Astigmatism can be corrected by wearing : (a) Concave lens (b) Convex lens (c) Spherical cylindrical lens (d) Without any lens Ans. (c) : Astigmatism can be correctly by wearing spherical cylindrical lens, it lens will be result in a compromise in visual ocuity. The greater the astigmatism, the greater the compromise. Astigmatism is caused by an irregular curvature of the eye's cornea or lens. This can change the way to pass light and causes blurry or distorted vision. It is characterized by difficulty seeing at night. 26. When the egg does not get fertilized, which hormone is produced by ruptured follicle? (a) LH (b) FSH (c) Progesterone (d) Thyroxine Ans. (c) : Over the next four or five days the fertilized egg continues to divide and to travel toward the uterus. The hormone progesterone, which is secreted into your blood stream by the burst follicle called corpus luteum prepares the uterus for the egg to implant. 27. Pollination by insects is known as : (a) Entomophily (b) Anemophily (c) Chiropterophily (d) Ornithophily Ans. (a) : Pollinators range from physical agents, especially the wind or biotic agents such as insect birds bats and other animals, pollination by insects is called Entomophily, by birds Ornithophily. 28. Vitellogenesis is the process of formation of (a) Vitelline membrane around the developing egg cell (b) Yolk cell membrane around the yolk mass during development of reptiles and birds (c) Yolk and its deposition within the growing oocyte of amphibians (d) Archenteron around egg cell Ans. (c) : Vitellogenesis process is also known as yolk deposition Vitellogenesis is the protein from which the substance of yolk is derived. It is synthesized by the liver and induced by estrogen. The uptake of vitellogenin is stimulated by gonadotrophins and highly selective. Once in corporated into the Oocytes vitellogenin is converted within the yolk platelets into the yolk proteins. Hence the correct answer is option (c). 29. During which epoch of Cenozoic era, there was extinction of large mammals? (a) Oligocene (b) Pliocene (c) Holocene (d) Pleistocene Ans. (d) : The Pleistocene was the epoch of megafauna or very large mammals, which included mammoths giant ground sloths, giant beavers, and saber - toothed cats, like the ones picture, most of the megafauna were wiped out in a mass extinction that began towards the end of the pleistocene. KVS TGT Science 2013

30.

Fossils are found in : (a) Rocks (b) Amber (c) Asphalt (d) All of these media Ans. (d) : Fossils are found in sedimentary rocks and occasionally some fine grained, low - grade metamorphic rocks. Sometimes the fossils have been removed, leaving moulds in the surrounding rock or moulds may have filled by other materials forming costs of originals fossils 31. Vascular bundles in a dicot stem are : (a) Open, collateral and endarch (b) Closed, collateral and endarch (c) Closed, collateral and exarch (d) Open, collateral and exarch Ans. (a) : In the dicot stem, a strip of cambium is present between the xylem and phloem. so, the vascular bundle is of open type in the dicot stem. As the vascular bundle of the dicot stem is made up of xylem, phloem and cambium. That's why the stem of dicot plants is known as endarch. Hence, the vascular bundles in the dicot stem are open, collateral and endarch. 32. Hard, lignified, thick walled, long and pointed cells in a plant are (a) Parenchyma (b) Sclerenchyma (c) Collenchyma (d) Sclerids Ans. (b) : Cells of sclerenchyma are thick walled and are usually lignified. These cells are generally long and pointed in shape. The thickness is due to the formation of secondary wall. At maturity, the cells are devoid of protoplast. The cell wall encloses a cavity lumen. Parenchyma is thin walled, polyhedral, isodimetric, mostly non-lignified, soft living cells with sufficient cytoplasm and one or more nuclei. collenchyma is prismatic or elongated cells with thick non-lignified walls; the young cells are elastic while the mature ones are hard and brittle. * Sclereids are isodiametric, hard cells with a narrow lumen and are grouped according to their various shapes. 33. The layer of actively dividing cells of skin is termed as (a) Stratum compactum (b) Stratum corneum (c) Stratum lucidium (d) Stratum germinativum Ans. (d) : Stratum malpighii /stratum germinativum is the innermost layer, it actively divides to produce new cells by mitosis to replace the cells that are damaged or lost due to regular wear and tear. It lies on the basement layer. 34. Alternative forms of a gene are called (a) Loci (b) Multiples (c) Chromosomes (d) Alleles 8

YCT

Ans. (d) : The allele is the alternative forms of a gene. An allele is among two, or more, parts of the same gene on a chromosome at the same location. In their sequence, genes may develop mutations, resulting in numerous variations in the population, defined as alleles. These alleles form a protein with slightly different varients, creating different phenotypic characteristics. 35. What will be the genotypic ratio of the cross between Rr X rr? (a) 1 : 2 : 1 (b) 3 : 1 (c) 1 : 1 (d) 1 : 1 : 1 Ans. (c) : Genotype is the genetic constitution that helps in the expression of a particular character which is visible by the phenotype or physical appearance. Punnet square to study the genotype and phenotype after a cross. In this questions character 'r' and 'Rr' is a heterozygous allele with dominant character R, the cross will be.

38.

In this cross 2 individuals have similar genotypes so the genotypic ratio will be 1 : 1. 36. Which of the following does not occur within mitochondria? (a) Krebs cycle (b) Glycolysis (c) ETC (d) ATP Synthesis Ans. (b) : Glycolysis is the only metabolic process of the choices listed that does not occur in the mitochondria, it occurs in the cytoplasm. In eukaryotes, the krebs cycle reactions take place in the mitochondrial matrix, a dense solution that surrounds the mitochondria crests. Electron transport chain (ETC) occurs in mitochondria in both cellular respiration and photosynthesis. ATP is synthesized in the mitochondria by the process of oxidation phosphorylation. 37. Cell wall of the plant cell contain pit fields known as : (a) Mitochondria (b) Golgi bodies (c) Plasma membrane (d) Plasmodesmata Ans. (d) : Plasmodesmata are connections that are abundantly seen in the pits the structure of pit includespit membrane, pit cavity and pit aperture. Plasmodesmata are small channels that connect the cytoplasm of adjacent plant cells and allow for communication and transport of molecules between the cells. They play an important role in plant growth and development, including transport of nutrients, and defense against pathogens.

40.

KVS TGT Science 2013

Which phylum does not have a true coelom? (a) Arthropoda (b) Echinodermata (c) Porifera (d) Annelida Ans. (c) : Porifera phylum without a fluid-filled cavity or coelom in their bodies. Example- invertebrates like planarians, tapeworms and flukes, porifera and cnidaria are acoelomates.

39.

Which one of the following kind of animals is triploblastic? (a) Corals (b) Flatworms (c) Sponges (d) Ctenophores Ans. (b) : Triploblastic is a condition which describes an animal having a body composed of three embryonic layer : the ectoderm, mesoderm and endoderm. Most multicellular animals belonging to phylum platyhelmithes to phylum chordata are triploblastic. ctenophores, sponges and corals are diploblastic.

Phylogenetic system classification is based on (a) Phylogenetic system of classification is based on (b) Floral characters (c) Evolutionary relationship (d) Morphological features Ans. (c) : The phylogenetic system of classification exhibits evolutionary relationship of entities. Relationships are exhibited via a cladogram and a phylogram in this system. * It came into being after the acceptance of natural selection and the doctrine of evolution by Charles Darwin. * He proposed that the present - day plants/animals have origins from a few ancestral ones after experiencing a few periodical alternations. * Classification based on morphological features is called the Artificial system. * Classification based on chemical constituents is called chemotaxonomy.

41.

9

Mycorrhiza is correctly described as : (a) Parasitic association between roots and some fungi (b) Relation of ants with the stem of some trees (c) Symbiosis of algae and fungi (d) Symbiotic relationship between fungi and roots of some higher plants YCT

Ans. (d) : Mycorrhizae are a symbiotic association between plants roots and fungi their major role is to enhance nutrients and water uptake by the host plant by exploiting a larger volume of soil than roots alone can do. Mycorrhiza associated with roots of higher plants p e.gpine, birch etc. In this fungal hyphae take nutrition from the plant and in return increase surface area for absorption of water and minerals. 42. Common cold is not cured by antibiotics because it is : (a) Caused by a Gram negative bacterium (b) Not an infectious disease (c) Caused by a virus (d) Caused by a Gram positive bacterium Ans. (c) : The common cold can be caused by a number of different viruses, including rhinoviruses, parainfluenza and seasonal corona viruses. Antibiotics are anti-bacteria, hence viruses do not respond to antibiotics. 43. Prions affect which tissue in the body? (a) Endocrine (b) Cardiac (c) Connective (d) Nervous Ans. (d) : Most Prion disease affect the nervous system predominatly, the most common change caused by prions is the formation of tiny bubbles in brain 44. Gas leaked in Bhopal tragedy was : (a) Methyl isocyanate (b) Potassium isothiocyanate (c) Sodium isothiocyanate (d) Ethyl isothiocyanate Ans. (a) : The Bhopal disaster or Bhopal gas tragedy was a disaster that resulted from an accident that took place at a union carbide subsidiary pesticide plant on 3rd December, 1984 the plant released 42 tones of toxic methyl isocyanate (MIC) gas exposing more than 500,00 people to toxic gases. 45. Tuberculosis is caused by : (a) Mycobacterium (b) Vibrio (c) Clostridium (d) Nitrobacter Ans. (a) : Tuberculosis (TB) is known to be an infectious and fatal condition that is caused by bacteria called Mycobactria tuberculosis which affect lungs. 46. HIV mainly affects : (a) Cell mediated T lymphocytes (b) Helper T lymphocytes (c) Natural Killer cells (d) Cytotoxic T lymphocytes Ans. (b) : HIV mainly affects helper T-lymphocytes, HIV destroys CD4 T lymphocytes (helper T cells), Because of these, healthcare professionals measures CD4 levels to monitor HIV progression and AIDS helper T cells are crucial for immune system function and activate after encountering antigens from disease causing microorganisms. KVS TGT Science 2013

47.

Release of milk and contraction of uterus are caused by : (a) Prolactin (b) Oxytocin (c) Progesterone (d) Estrogen Ans. (b) : 'Oxytocin' is a natural hormones that stimulates Uterine contraction in childbirth and lactation after childbirth. It also affects aspects of human behaviour and the male and female reproductive system. 48. Which of the following are analogous organs? (a) Eye of cephalopods and vertebrates (b) Flippers of seal and wings of pigeon (c) Tendril of Passiflora and thorn of Bougainivillea (d) Legs of grasshopper and honey bee Ans. (a) : Analogous organ:- which have differ origin but there functions are same. As per given options eye of cephalopods and Vertebrates have different origin but they serve the same function. i.e. sight. Rest three options are example of homologous organs i.e. they have Same origin but are used for different functions. 49. Which of the following is an apocrine gland? (a) Mammary gland (b) Thyroid (c) Thymus (d) Hypothalamus Ans. (a) : Apocrine gland is the mammary glands, responsible for secreting breast milk. Apocrine secretion is less damaging to the gland than holocrine secretion but more damaging than mirocrine secretion. 50. The development of fruit without fertilization is : (a) Parthenogenesis (b) Parthenocarpy (c) Appomixes (d) Apogamy Ans. (b) : The fruit is formed without fertilization of the ovary, it is called a parthenocarpy. Parthenocarpic fruit is seedless fruit. Example of parthenocarpic fruit are bananas, grapes, etc. 51. Ranikhet attacks the (a) Pigs and poultry (b) Poultry and honey bee (c) Silkworm and camels (d) Poultry only Ans. (d) : Ranikhet is a highly infectious and fatal viral disease, it attacks poultry of all ages. Also known as New castle disease. Ranikhet disease is a respiratory disease caused in poultry by an Avian paramyxovirus type 1 (APMV-1) virus. 52. Knee or elbow joint is (a) Pivot (b) Condyloid (c) Gliding (d) Hinge Ans. (d) : Hinge joint is a type of synovial joint that includes ankle, knee and Elbow joints. Thus are formed between two or more bones where the bones are allowed to move along only one axis to flex. so, the correct answer is option (d)

10

YCT

53.

The product of conjugation in Spirogyra is (a) Akinete (b) Zygospore (c) Chlamydospore (d) Zoospore Ans. (b) : The correct option is Zygospore, when conjugation occurs in Spirogyra. Zygospores are formed, The product of conjugation in Spirogyra or fertilization of Chalamydomonas is zygospore both are the members of green algae where gametes are fused to form zygote which develops into thick walled zygospore. 54. Thylakoids occur inside (a) Endoplasmic Reticulum (b) Golgi Complex (c) Chloroplast (d) Mitochondria and chloroplast Ans. (c) : Thylakoids are the internal membranes of chloroplasts and Cyanobaclcria and provide the platform for the light reaction of photosynthesis. 55. Which state consists of super energetic and super excited particles? (a) Liquid (b) Gas (c) Plasma (d) Bose-Einstein Condensate Ans. (c) : Plasma stage is consists of super excited and super energetic particles that are present in form of ionized gases. Plasma is made by heating a gas to a very high temperature so that gas molecules lose all their electrons. It does not have a definite shape and volume. 56. According to Henry's law (a) doubling the partial pressure doubles the solubility (b) doubling the partial pressure triples the solubility (c) doubling the partial pressure decreases the solubility (d) doubling the partial pressure does not affect the solubility Ans. (a) : According to Henry's law "doubling the partial pressure doubles the solubility. 'Henry proposed a quantitative relation between pressure and solubility of a gas in a solvent which is known as Henry's law. • The solubility of a gas in a liquid at constant temperature is directly proportional to the pressure of the gas present above the surface of liquid or solution. • The partial pressure of the gas in vapour phase (P) is proportional to the mole fraction of the gas (x) in the solution. P ∝ x or p = kHx Here- P = Partial pressure of the gas in vapour phase X = Mole fraction of the gas in the solution kH = Henry's law constant. KVS TGT Science 2013

57.

The protecting power of lyophilic colloidal sol is expressed in terms of (a) Critical miscelle concentration (b) Oxidation number (c) Coagulation value (d) Gold number Ans. (d) : The protecting power of lyophilic colloidal sol is expressed in term of "Gold number". or The minimum mass of the protective colloid in milligrams that must be added to 10mL of a standard red gold sol. So that no. coagulation, occurs when 1mL of 10% NaCl solution is rapidly added to it is called the gold number of the lyophilic colloid (protective colloid). 58. Atoms having same number of neutrons but different mass number are called (a) isotopes (b) isobars (c) isotones (d) isotherms Ans. (c) : Atoms having same number of neutrons but different mass number are called 'isotones".

59.

Which of the following has largest negative electron affinity? (a) F (b) Cl (c) Br (d) I Ans. (b) : The amount of energy released when are electron is added to the valence shell of are isolated gaseous atoms is known as electron affinity.

X ( g ) + e−  → X −( g ) Going from the top to the bottom of a group of periodic table, the combined effect of the increase in the atomic size and the shielding effect decreases the effective nuclear charge. As a result, electron affinity decreases. However, the order of electron addition of halogen elements – Cl > F > Br > I. The electron addiction of fluorine is unexpectedly low compared to chlorine. In the following table, you will find the electron affinity of halogen. Element name Electron affinity Cl – 348.8 kJ mol–1 F –328.8 kJ mol–1 Br –324 kJ mol–1 I –295 kJ mol–1 So, chlorine has the largest electron affinity. 60. The family of elements with the highest ionisation enthalpy : (a) Alkali metals (b) Alkaline earth metals (c) Halogens (d) Noble gases

11

YCT

Ans. (d) : The family of element with the highest ionisation enthalpy is "Noble gases". Ionization energy is the minimum amount of energy required to remove an electron from the valence shell of a neutral gaseous atom. M(g) + IE → M(g) + e– If gaseous atom is to lose more than one electron they can be removed only one after another i.e. in succession and not simultaneously. This is known as successive ionisation energy or potential. 61. Which of the following has coordination bond? (a) N2 (b) CaCl2 (c) O3 (d) H2O Ans. (c) : Co-ordinate bond is a special type of covalent bond which is formed by the sharing of electron between two atoms. Where both the electron of the shared pair are contributed by one atom. since this type of sharing of electron exists in "O3, SO3, H2SO4. Therefore these contain co-ordinate bonds. 62. What will be the pH at 25ºC containing 0.10 M CH3COONa and 0.03 M CH3COOH ? pKa for CH3COOH = 4.57 (a) 4.87 (b) 3.33 (c) 5.09 (d) 4.05 Ans. (c) : According to Henderson equation pH of acidic buffer is Salt pH = Pka + log Acid

0.10 0.03 = 4.57 + [log 10 – log 3] = 4.57 + log 3.33 = 4.57 + 0.52 = 5.09 63. The function of alum used for the purification of water is to (a) coagulate the sol particles (b) disperse the sol partiles (c) emulsify the sol particles (d) absorb the sol particles Ans. (a) : The function of alum used for the purification of water is to "Coagulate the solution particles". One coagulated they sediment down. 64. Which one of the following is metal? (a) C (b) N (c) Na (d) O Ans. (c) : Sodium (Na) belong to group I which are metallic element and while C, N and O all are non metals. 65. Which non-metal is found in liquid state are room temperature? (a) Mercury (b) Zinc (c) Iodine (d) Bromine

Ans. (d) : At room temperature, "bromine" is the only non-metal that exists as a liquid. • Bromine belongs to the 17th group of the periodic table, a member of the halogen family. • Other than mercury which is a metal, bromine is one of the two elements on the periodic table that are liquid at room temperature. • It exists as a diatomic molecule at room temperature. 66. Oxides of non-metals are ____in nature. (a) Basic (b) Acidic (c) Neutral (d) Amphoteric Ans. (b) : Oxides of non - metals are acidic in nature. When a non-metal react with oxygen, it gives oxides which on dissolution with water gives on acid. Non metal oxides are usually gaseous at room temperature. 67. Which gas is produced when metal react with acids? (a) Oxygen (b) Nitrogen (c) Hydrogen (d) Carbon dioxide Ans. (c) : "Hydrogen" gas is produced when metal react with acid. An acid reacts with a metal and gives a salt with evolution of hydrogen gas. Acid + metal → salt + H2 Many reactive metals like sodium potassium etc. displace hydrogen from acid. 2HCl + 2Na → 2NaCl + H2 68. Which of the following can be shown to be both a Bronsted acid and a Bronsted base?

= 4.57 + log

KVS TGT Science 2013

(a) O-2

(b) HSO −4 1

(c) PO −4 3

(d) CN −1

Ans. (b) : A Bronsted - Lowry acid is defined as substance that releases H+ ions. A Bronsted - Lowry base is defined as substance that accepts H+ ion.

HSO −4 can accept H+ thus is a from H2SO4 and can release one H+ to form SO 24 − . Therefore it can act as both Bronsted acid and Bronsted base. HSO 4− → SO 42− + H +

HSO −4 + H + → H 2SO 4 69.

An example of an emulsifying agent would be ____ (a) Oil (b) Soap (c) Water (d) Salt Ans. (b) : "Soap" an example of an emulsifying agent. To prevent breaking of emulsion, a small amount of third substance called an emulsifiying agent or emulsifier is added to the emulsion during its preparation, usually, soaps, detergents, long chain. 70. Which is not an organic compound? (a) Methanol (b) acetonitrile (c) sodium cyanide (d) phenol

12

YCT

Ans. (c) : The few carbon - containing compound not 75. The compound formed as a result of oxidation of ethyl benzene by KMnO4 is classified as organic include. (a) Benzoic acid (b) Acetophenone e.g. :- sodium cyanides, carbides, carbonates, and (c) Benzophenone (d) Benzyl alcohol cyanides. Ans. (a) : The compound formed as a result of 71. In benzene, each carbon atoms undergoes oxidation of ethyl benzene by alkaline KMnO4 is yesv]peerve ceW, ØelÙeskeâ keâeye&ve hejceeCeg menve keâjlee nw: "benzoic acid". (a) sp Other reagent that can be used are acidified potassium (b) sp2 dichromate or dilute nitric acid. (c) sp3 OH (d) sp2 and sp3 hybridisation Alkaline Ans. (b) : In benzene each carbon atoms undergoes "sp2" hybridization. Sp2

KMnO4

Ethyl benzene

H

2

Sp

Sp

76.

C

H

C

H

C

C

C C

72.

H

H Sp2

Sp2 Sp2

H

[π and 3σ → sp2] The shortest C - C bond distance is found in : (a) ethane (b) ethene (c) ethyne (d) diamond

Ans. (c) : C2H2 (ethyne or acetylene) due to presence of carbon - carbon triple bond has shortest carbon - carbon bond length.

H

C

C

H

73.

Propene reacts with HBr in presence of organic peroxide to form : (a) 1-Bromopropane (b) 2-Bromopropane (c) 3-Bromopropane (d) 2-Bromopropene Ans. (a) :

CH3CH

CH2 + HBr

Peroxides

CH3CH2CH2HBr Hydrogen Halides like HBr usually react with alkene using an electrophilic addition mechanism. In the presence of organic peroxides HBr adds to carbon across number of double bond which carries a lesser number of hydrogen atoms. This is called Anti Markownikoff's rule. 74. The strongest acid among the following is : (a) C2H6 (b) C3H8 (c) C2H4 (d) C2H2 Ans. (d) : Acetylene (C2H2) is the strongest acid among the given compound whereas the acetylene is the strongest acid among the given compound. KVS TGT Science 2013

Benzoic acid

2

Which acid is used in the manufacture of synthetic rubber? (a) Acetic acid (b) Formic acid (c) Carbonic acid (d) Benzoic acid Ans. (a) : "Acetic acid" is used in the manufacture of synthetic rubber. 77. Not a characteristic property of ceramic material (a) high temperature stability (b) high mechanical strenght (c) low elongation (d) low hardness Ans. (d) : The hardness is the resistance to penetration. "Low hardness" is not a characteristic property of ceramic material. 78. Increasing concentration of CO2 in atmosphere is responsible for. (a) acid rain (b) greenhouse effect (c) lack of photosynthesis (d) death of aquatic life Ans. (b) : Global warming is caused by increased concentration of greenhouse gas. Carbon dioxide (CO2) is one of the greenhouse gas in the atmosphere. 79. The coating on modern non-stick cookware and electric iron is of (a) Terrycot (b) Rayon (c) Polyester (d) Teflon Ans. (d) : "Teflon" is a plastic on which oil and water do not stick. It is used for non-stick coating on cook wares and electric iron. 80. For corrosion of iron to take place (a) Presence of moisture is sufficient (b) Presence of moisture and oxygen is essential (c) Hydrogen is required (d) A strong acid is necessary Ans. (b) : Rusting of iron take place in the "presence of moisture and essential air, so the presence of air and water vapour in air are two necessary condition for rusting of iron.

13

YCT

NAVODAYA VIDYALAYA SAMITI TGT SCIENCE, 2022 [Exame Date: 29.11.2022] (Solved Paper with Explanation) which one of the four containers would have Which of the following is a heterogeneous maximum number of molecules of the gas? mixture? (a) An aqueous solution of ferrous sulphate (a) A (b) B (b) An alloy of iron, nickel and carbon (c) C (d) D (c) A mixture of sulphur and sodium chloride Ans. (c) : You have four closed container powder 10 A = CH4 CH4 = = 0.625 (d) A mixture of hydrogen, oxygen and nitrogen 16 gases B = N2 Ans. (c) : A heterogeneous mixture rotors to the non10 C = H2 N2 = = 0.357 uniform mixture of various substance is called 28 heterogeneous mixture. 10 Example- A mixture of sulphur and sodium chloride D = CO2 H 2 = = 5.0 2 powder. 10 • A mixture of sand and water. CO 2 = = 0.227 44 • A mixture of iron filings and sand. H2 is a container have maximum number of molecules 2. Which of the following is a set of isoelectronic of the gas. species? 5. Which of the following has the highest mass? (a) Li, Na, K (b) S-2, Cl–1, Ar + + + (a) 10 moles of H2O (c) Ar, K , Ca (d) Ne, Na , F (b) 0.5 moles of C12H22O11 Ans. (b) : Isoelectronic species- Isoelectronic species (c) 2.0 moles of CaCO3 are atom and ions that have the same number of (d) 4.0 moles of CO2 electrons. –2 Ans. (c) : 10 moles of H2O Example: S : In this give species, the number of 1 mole of water weight 18g. electron would be 18 that is (16+2) = 18. –1 10 moles of water = 10×18 = 180g Cl : In this given species, the number of (2) 0.5 moles of C12H22O11 electron would be 18. Calculating molar mass of sugar- C12H22O11 (17+1) = 18 Atomic mass of C → 12 gm mol–1 Thus, the correct option is “S–2, Cl–1, Ar” Atomic mass of O → 16 g mol–1 3. Select the correct statement from the following : So, molecular mass of C12H22O11→ 12(12) + (a) In an ion the number of electrons and protons 1(22)+16(11) = 342 g mol–1 is same. Let the given mass of sugar be x. (b) In an ion the number of electrons and Moles of sugar = 0.5 moles (given) neutrons is same. So, using formula (1) (c) In an ion the number of electrons and protons x can never be equal. 0.5 = 342 (d) In an ion the number of protons and neutrons is always same. x = 171g Ans. (a) : When an atom has an equal number of The mass of 0.5 moles of sugar = 171g. electron and protons, It has an equal number of negative (3) 2.0 moles of CaCO3 electric charges (the electron) and positive electric 1 mole of CaCO3 = 2×100 = 200g. charges (the protons). In an ion the number of electron (4) 4.0 moles of CO2 and proton is same. 1 mole of CO2 = 44gm 4. You have four closed containers A, B, C and D 4 moles of CO2 = 44×4 = 176g. containing CH4, N2, H2 and CO2 respectively. If the mass of gas in each container is 10 g, Thus, the highest mass of 2.0 moles CaCO3 is 200g. 1.

NVS TGT Science 2022

14

YCT

Consider the following statements about Rutherford's Nuclear Model of an atom : (A) The mass and positive charge of the atom is concentrated in the nucleus (B) . Electrons have negative charge (C). Neutrons exist in the nucleus (D). Most of the space in the atom is empty The correct statements are : (a) A, B and D (b) B and C (c) C and D (d) A and D Ans. (d) : Rutherford drew the following conclusions regarding the structure of atom : (i) Most of the space in the atom is empty as most of the α-particles passed through the foil undeflected. (ii) A few positively charged α-particles were deflected. The deflection must be due to enormous repulsive force showing that the positive charge of the atom is not spread throughout the atom as Thomson had presumed. The positive charge has to be concentrated in a very small volume that repelled and deflected the positively charged α-particles. (iii) Calculations by Rutherford showed that the volume occupied by the nucleus is negligibly small as compared to the total volume of the atom. According to Rutherford's atomic model (i) The positive charge and most of the mass of the atom was densely concentrated in extremely small region. This very small portion of the atom was called nucleus by Rutherford. (ii) The nucleus is surrounded by electrons that move around the nucleus with a very high speed in circular paths called orbits. Thus, Rutherford's model of atom resembles the solar system in which the nucleus plays the role of sun and the electrons that of revolving planets. (iii) Electrons and the nucleus are held together by electrostatic forces of attraction. 7. Which of these statements is/are true about Solder? (A) It is an alloy of Pb and Sn (B) It is used in making utensils (C) It is used in making electrical wires (D) Small amount of Cu is added to make it hard (a) A and B (b) Only A (c) A and C (d) C and D Ans. (b) : Solder is an alloy of lead (Pb) and tin (Sn). Its composition is 50 percent lead (Pb) and 50 percent tin (Sn). The melting point of solder is very low and is even lower than its constituting metals (Pb and Sn). Thus, the low melting point of solder makes it suitable for welding electrical wires. 8. Consider the following four elements where atomic numbers are given in parenthesis : A (9), B (11), C(12), D (13) The elements having same valency are

(a) A and B (b) B and C (c) C and D (d) A and D Ans. (a) : Element in same group show same valency because they have same number of electron in the outermost shell i.e. valence shall depend upon the valence electron they have same valency. Element Atomic No Valency

6.

NVS TGT Science 2022

A

9

1

B

11

1

C

12

2

D

13

3

The element have same valency of “A and B”. 9. The number of electrons in an element X is 18 and the number of neutrons present in its nucleus is 20. The correct representation 49 of the element X is 38

38

(a) 20

(b) 18

36

36

(c) 20

(d) 18

Ans. (b) : The ‘X’ is electron = 18 = no. of proton. Atomic number = number of electron = No. of proton. Atomic mass number = No. of neutron + No. of proton = 20+18 = 38 38 “ The correct representation = 18 X" 10.

15

Consider the following statements : (A) Cytoplasm is called protoplasm (B) Golgi apparatus is involved with the formation of Lysosomes (C) Mitochondria, nucleus and plastids have DNA, hence they are able to make their own structural proteins. (D) Mitochondria are said to be the powerhouse of the cell as ATP is generated in them The correct statements are : (a) A and B only (b) C and D only (c) C, D and A only (d) B and D only Ans. (*) : The golgi apparatus is usually present along with the endoplasmic reticulum (ER) of the cell. It packages, stores and modifies the products of ER. It is also involved in the formation of the Lysosome. • Cell organelles like mitochondria, plastids , and nucleus contain DNA. Mitochondria and plastids have circular DNA in them, so they are able to produce their own structural proteins. • The energy production inside the cell happens by a process called cellular respiration. Energy is produced in the form of ATP inside mitochondria. So, mitochondria are said to be the powerhouse of the cell. • Protoplasm is considered as the physical basis of life. The protoplasm of a cell. YCT

11.

Cell organelles that have own DNA and ribosomes are (a) Golgi apparatus and lysosomes (b) Mitochondria and plastids (c) Plastids and endoplasmic reticulum (d) Mitochondria and vacuoles Ans. (b) : Cell organelles that have their own DNA and ribosomes are mitochondria and plastids. • Mitochondria is an organelle present only in the eukaryotic cells and contains independent genetic material called the extrachromosal DNA. The ribosomes in the mitochondria are 70S type and similar to that of prokaryotic ribosomes. • Plastids are found in the cells of plants, algae, and some other eukaryoic organisms. Plastids like chloroplast have their own. DNA and ribosomes because of which they are able to synthesize some of their own proteins and replicate independent of the nucleus. 12. The proteins and liqids which are essential for building the cell membrane are manufactured by (a) Mitochondria (b) Plasma membrane (c) Golgi apparatus (d) Rough endoplasmicreticulum Ans. (d) : Endoplasmic reticulum (ER) is a complex network of tubular membranes exclusively present in the cytoplasm of the eukaryotic cell.There are two types of endoplasmic reticulum. Rough endoplasmic reticulum (RER) and smooth endoplasmic reticulum (SER). • RER main function is to produce proteins in the cells and ribosomes are attached to their surface. • SER main function is to produce lipids and ribosomes are not attached to their surface. 13. Consider the following statements about tissues (A) Apical and intercalary meristems are permanent tissues (B) Collenchymatous tissues are irregularly thickened at the corners (C) Meristematic tissue in its early stage lacks vacoules (D) Parenchymatous tissues have intercellular spaces The correct statements are (a) A and C only (b) B and D only (c) B and C only (d) B, C and D only Ans. (d) : Apical and intercalary meristems are meristematic tissues and not permanent tissues. Meristematic tissues are made up of continuously dividing cells and permanent tissues are made of nondividing cells. The apical meristem is located a the tips of roots and shoots and the intercalary meristem is present at internodes. NVS TGT Science 2022

• Cocllenchymatous tissues are supporting tissues composed of more or less elongated living cells with unevenly and there is no requirement of storing food. Also, they do not produce large amount of waste. • Parenchymatous tissue is a type of permanent tissue that makes a major part of ground tissues, where other tissues like vascular tissues are embedded. It consist of relatively large, thin-walled cells, and the cells are arranged loosely with intercellular spaces among them. 14. In an experiment a student removed the tip of a sugarcane plant. It was observed that the plant is still growing in length. It may be due to the presence of (a) apical meristem (b) cambium (c) intercalary meristem (d) lateral meristem Ans. (c) : The intercalary meristems are located at the internodes or the base of the leaves. The intercalary meristems help in increasing the length of the internode and are capable of cell division that allows for raped growth and regrowth of many monocots. Sugarcane is a monocot plant with nodes that have intercalary meristerm. So, even if the sugarcane’s tip is cut or removed it can still grow because of the intercalary meristem found at nodes. 15. While cycling, a student met with an accident in which two long bones of his right hand were dislocated. The possible reason for this dislocation is (a) tendon break (b) ligament break (c) skelatal muscle break (d) areolar tissue break Ans. (b) : The radius and ulna are long bones that make up the forearm, extending from the elbow to the wrist. The annular ligament or orbicular ligament is a strong band of fibers present on the head of the radius that retains it in contact with the radial notch of the ulna. Dislocation of bones occurs when joints held by ligaments get separated. So, the possible reason for this dislocation is ligament break. 16. Consider the following class of plants : (A) Bryophyta (B) Gymnosperms (C) Pteridophnyta (D) Thallophyta Classes of plants having specialized tissues for the conduction of water are (a) A and D only (b) B and C only (c) A, B and C only (d) B, C and D only Ans. (b) : Pteridophytes are the first terrestrial plants to have the specialized conducting tissue. Xylem and phloem tissues are important for conduction of water and mineral in plants. • Gymnosperms are plants which bear naked seeds and also contains the xylem and phloem for water and food conduction. • In Thallophytes and Bryophytes, the plant body is not differentiated into roots, stem, and leaves. They lack specialized tissues for the conduction of water.

16

YCT

17.

Consider the following statements : (A) Aves have feather covered body, forelimbs are modified as wings and breathe through lungs. (B) Aves are warm-blooded, egg-laying andhave four chambered heart. (C) Amphibians, fishes and reptiles are oviparous (D) Most of the mammals are viviparous The correct statements are : (a) A and B only (b) B and C only (c) A, B and D (d) B, C and D Ans. (c) : Aves are warm-blooded and oviparous (egglaying) in nature. They have feathers that provide them covering and the forelimbs are modified as wings to assist in their flight. The heart is four-chambered (two atria and two ventricles) and they breathe through lungs. • Most of the mammals are viviparous, which means they give birth to young ones. The duck-billed platypus and the spiny anteater are examples of oviparous mammals. • Several fishes amphibians, and reptiles are viviparous. Examples of viviparous fishes are sharks, stingrays, and giant rays. Several species of frogs and salamanders are viviparous. 18. Among the following, the true fish is (a) Dogfish (b) Jellyfish (c) Silverfish (d) Starfish Ans. (a) : Dogfish is a species of shark (scoliodon), and belongs to the class of cartilaginous fishes or chondrichthyes and hence is a true fish. It is referred to as dogfish because it has a strongly evolved sense of dog-like smell. • Jelly fish are invertebrates that belong to the phylum coelenterate. • Silverfish areinsects that belong to the phylum arthropods. • Starfish are invertebrates that belong to the phylum echinodermata. 19. In the context of four common gases, namely Oxygen (O2), Chlorine (Cl2), Carbon dioxide (CO2) and carbon monoxide (CO), select the correct option from the following table : Option Product of Product of Used in Acidic water respiration incomplete oxide combustion purification system A CO2 CO O2 CO B CO2 CO2 Cl2 CO2 C. O2 CO Cl2 CO2 D CO2 CO Cl2 CO2 (a) A (b) B (c) C (d) D NVS TGT Science 2022

Ans. (d) : Respiration is an oxidative process where the respiratory substrates like glucose are broken down into simpler substances. The inhaled O2 converts the glucose with the help of respiratory enzymes into CO2 and H2O with release of energy in the form of ATP. • Incomplete combustion is a type of combustion in which the combustible substances react partially with O2. It results in the formation of carbon monoxide (CO). • Chlorination is a method in which chlorine gas (Cl2) is passed through the water to kill germs and used in water purification system. • CO2 is an acidic oxide as it dissolves in water to form carbonic acid (H2CO3). 20. Four objects A, B, C and D are running on a levelled horizontal road with different speeds. The speed of every object remains uniform. If the distance (d)-time (t) graph of the four objects are as per the diagram, the correct statement about the motions of the objects is :

(a) B is faster than C (b) A is the slowest (c) C is faster than D (d) D is the slowest Ans. (b) : Let's see give figure, object A take lot time to travel small distance. That's why it is slowest. 21.

An object is covering distance in direct proportion to the square of time elapsed. The acceleration of the object is (a) constant (b) increasing (c) decreasing (d) zero Ans. (a) : Given, distance proportional to square of time

⇒ x ∝ t 2 ⇒ x = kt 2 So, velocity v =

dx = 2kt dt

d2x = 2k dt 2 Thus, The acceleration of object is independent to time. So, it is constant. and, acceleration a =

22.

17

A block of mass 2 kg is sliding on a smooth horizontal surface with a constant speed of 0.5 m/s. The force required to keep the block sliding with the same speed is (a) 400 N (b) 200N (c) 100 N (d) Zero YCT

Ans. (d) : Since, block is sliding with constant speed. So, There is no friction. Therefore, There is no force required to keep the block sliding with the same speed. 23.

In the game of cricket a fielder after catching the fast moving ball pulls his hands backwards with the moving ball. This enables the fielder to (a) Decrease the rate of change the momentum of the ball (b) Increase the rate of change of momentum of the ball (a) 0.001 N (b) 0.001 N (c) Increase the acceleration of the ball by (c) 0.01 N (d) 0.1 N increasing the time duration in which the high Ans. (b) : Given, m = 50g speed of the ball reduces to zero According to Newton's force law, (d) exert larger force on the ball to stop it. dp dv 50  0 − 20  1 F= m = × × Ans. (c) : After catching the fast moving ball, a fielder dt dt 1000  0 − 10  100 pulls his hands backwards because by doing this, he 1000 1 F= × = 10−3 N = 0.001N increases the time in which the velocity of ball will 1000 1000 become zero. Hence, he will not get injured by the ball. 26. An engine of mass 10000 kg pulls a train of 5 wagons each of 2000 kg, along a horizontal 24. A vehicle travels half the distance I. with speed trak. If the engine exerts of force of 45000 N V1, and the other half with speed V2, then the and the track offers a friction force of 5000 N, average speed of the vehicle is the acceleration of the train is v1 + v 2 v1 v 2 (a) 2.0 m/s2 (b) 2.25 m/s2 (a) (b) 2 (c) 4.0 m/s (d) 4.5 m/s2 v1 + v 2 2 Ans. (c) : Given, Mass of engine M = 10000 kg 2v1 v 2 v 2v 2 Number of wagons N = 5 (c) (d) 1 2 v1 + v 2 v1 + v 2 Total mass of wagons, m = s × 20000 = 10,0000 kg ∴M=m Ans. (c) : Given, Total distance = l and Friction force fs = 5000 N Net force Fa = F–fs = (45000 – 5000)N = 40000 N The acceleration of train Fa 4000 l/2 a= = = 4m / s 2 The time taken to cover half distance t1 = Total Mass 10000 v 1

Similarly, t 2 =

27.

l/2 v2

SI units of Universal Gravitational Constant (G) and acceleration due to gravity (g) are respectively (a) N kg2/m2 ; m/s2 (b) N m2/kg2 ; m/s2 2 2 (c) N m /kg ; m/s (d) N m2/s2 ; m/s2 Ans. (b) : As Newton's gravitational force GMm F= r2 Fr 2 G= Mm The unit of G (Universal Gravitational Constant) Newton − m 2 = kg − kg

So, Total time t = t1 + t2 Therefore, Average speed =

25.

Total distance Total Time

=

l l =  l l  l  v 2 + v1  +      2v1 2v 2  2  v1v 2 

=

2v1v 2 ( v1 + v 2 )

The velocity (v)-time (t) graph of a ball of mass [ G ] = N − m 2 / kg 2 50 g along a straight line on a horizontal surface is as shown. The magnitude of force and F = mg ⇒ g = F = ma = a m m exerted by the surface on the ball to bring it to 2 [g] = m / s rest is

NVS TGT Science 2022

18

YCT

28.

An object is thrown vertically upwards with a Ans. (b) : Given, Kinetic Energy of object = 72 Joule speed of 20 m/s. If the air resistance is mass of object m = 1 kg negligible, the greatest height to which this 1 So, mv 2 = 72 object would rise is (g=10 m/s2) 2 (a) 10 m (b) 20 m ⇒ v2 = 12 × 12 ⇒ v = 12 m/s (c) 40 m (d) 200 m The momentum of object = mv = 1 × 12 = 12 kg-m/s Ans. (b) : Given, initial velocity u = 20 m/s 32. The given displacement-time (d-t) graph At highest point, final velocity v = 0 represents a disturbance travelling with a So, speed of 1200 m/s. The wavelength of the v2 = u2 – 2gh (in case of vertical thrown) disturbance is 2 O = (2∆) – 2gh 20 × 20 ⇒h= = 20m 2 × 10 29. An object weights Wa in air. When immersed fully in water it weighs Ww. If on immersing in (a) 1.2 × 10-4 m (b) 2.4 × 10-4 m a liquid, the object weight Wl, the relative -3 (c) 1.2 × 10 m (d) 2.4 × 10-3 m density oft he liquid is Ans. (d) : Given, speed v = 1200 m/s Wa Wa (a) (b) Let's see figure, The time period for complete one cycle Wa − Ww Wa − Wl T = 2µs = 2 × 10–6 s Wl Wa − Wl (c) (d) λ Since, v = = νλ Ww Wa − Ww T Ans. (d) : Given, ⇒ λ = ν × T = 1200 × 2 ×10 −6 Weight of object in air = Wa λ = 24 × 10−4 m = 2.4 × 10 −3 m Weight in water when fully immersed = ww 33. People living in an overcrowded and poorly According to Archmedes Principle, ventilated house are likely to suffer from weight of liquid displaced = weight of body in air – (a) Airborne diseases (b) AIDS weight of body in liquid (c) Cancer (d) Cholera ρwVg = Wa – Ww ....... (1) Ans. (a) : The diseases which are transmitted through Similarly for liquid contaminated air are known as airborne. People living ρ1Vg = Wa − W1........(2) in an overcrowded and poorly ventilated house are On diving (2) and (1), we get likely to suffer from airborne diseases, because the infected person easily speeds the droplets through ρ1 W − W1 = a breathing and sneezing. Due to poor ventilation, the ρ w Wa − Ww infectious droplets keep on circulating in the closed 30. Consider the following units : areas and hence more and more people can become (A) kilowatt hour infected with the disease. Examples of airborne diseases are common cold, influenza, chickenpox, mumps, (B) joule measles, whooping cough (pertussis), TB, diphtheria, (C) newton metre SARS-CoV-2, etc. (D) pascal metre 34. Producing antiviral drugs (for example The units of energy are : producing vaccine for Covid-19) was a much (a) A and B only (b) C and D only harder task than making anti-bacterial (c) A, B and C (d) B, C and D drugs/medicines because the Ans. (c) : S.I. Unit of Energy = Joule (a) viruses have a protein coat Kilowatt hour (KWh), Newton-meter, Ergs, calori etc. (b) viruses are on the borderline of living and Also, Pascal - m3 is the unit of energy. non-living 31. The kinetic energy of an object of mass 1 kg is (c) viruses make use of the host machinery for 72 J. The momentum of the object is their life processes (a) 6 kg m/s (b) 12 kg m/s (d) viruses have very few biochemical (c) 9 kg m/s (d) 24 kg m/s mechanisms of their own NVS TGT Science 2022

19

YCT

Ans. (d) : Virus is a non-living substance when present in the external environment. Virus becomes active when it enters the host cell. Viruses have very few biochemical mechanisms of their own and depends on host cell machinery for their life-processes. So, the production of antiviral durgs is a much harder task than the production of anti-bacterial drugs. Antiviral drugs can damage host cells where the viruses reside. • Antibacterial drugs stop certain biochemical reaction in the bacteria that kills the bacteria. 35. The meaning of 'Ozone hole' is : (a) Creation of a large sized hole in the ozone layer (b) Small holes scattered in the ozone layer (c) Thinning of ozone layer in the upper reaches of the atmosphere (d) Hinderance in the production of ozone from molecular oxygen in the higher level of the atmosphere Ans. (c) : Ozone holes can be termed as the thinning or depletion of the ozone layer which is present in the upper atmosphere, i.e., stratosphere at the polar region of earth. The ozone layer is very important for life on earth because it has the property of absorbing the most damaging form of UV radiation, UV-B radiation. The ozone hole has developed because of the pollution of atmosphere with chlorofluorocarbons (CFCs), halons, and carbon tetrachloride. 36. Select from the following a species of honey bee which has high honey collection capacity and is commonly used for commercial honey production : (a) Apis carana indica (b) Apis dorsata (c) Apis florae (d) Apis mellifera Ans. (d) : Apis mellifera is an Italian honey bee which has high honey collection capacity and is commonly used for commercial honey production. They are capable of producing 150-250kg of honey per colony per year and sting significantly less than other types of bees. They stay in a given beehive for long periods and breed very well. They are also more capable of defending itself against enemies. • Apis cerana indica is commonly known as the Indian bee. • Apis dorsata is the gaint wild honey be commonly known as the rock honey bee or forest honey bee. • Apis florae is commonly known as the red dwarf honey bee and it is among the smallest honey bees. 37. A student burns a magnesium ribbon on a bunsen flame and collects the ash formed on a watch-glass. Which of the following statements depicts the correct observation made by the student? NVS TGT Science 2022

(a) The colour of the ash formed is brown (b) The ribbon burns with a brilliant white light (c) Reddish-brown gas with a pungent smell evolves during burning (d) The gas evolved burns with a pop sound Ans. (b) : Magnesium is a silver - white metal. When Magnesium (Mg) burns in the air emitting a bright white flash and then combines with oxygen to form basic magnesium oxide 2Mg + O2 → 2MgO (MgO-basic) The experiment shows that burning magnesium ribbon in air is a direct combination reaction. 38. What is observed when you place a shining copper plate in a beaker containing aqueous ferrous sulphate solution for about 30 minutes? (a) The solution turns blue and a grey layer is formed on the copper plate. (b) The solution becomes colourless and a grey layer is formed on the plate. (c) The solution remains green and no change takes place in the copper plate (d) The solution turns blue and the copper plate becomes thinner. Ans. (c) : When a shining copper plate is added to ferrous sulphate solution (green colour) there will be no change and no reaction will be observed. This is because copper is less reactive than iron, therefore, copper will be unable to displace iron from its solution. Cu(s) + FeSO4(aq) → No reaction 39. Consider the following chemical reaction : MaO2 + 4 HCI → MaCl2 + H2O + Cl2 In this reaciton (a) HCI is oxidised to Cl2 and MaO2 is reduced to MnCl2 (b) HCI is oxidised to H2O and MnO2 is reduced to MnCl2 (c) HCI is reduced to Cl2 and MnO2 is oxidised to MnCl2 (d) HCI is reduced to H2O and MnO2 is oxidised to MnCl2 Ans. (a) : MnO2 + 4HCl → MnCl2+2H2O+Cl2 Substance oxidized = HCl Substance reduced: MnO2 Oxidising agent : MnO2 Reducing agent : HCl Here, Mn in +4 oxidation state in MnO2 while it is in +2 oxidation state in MnCl2.Thus a decrease in oxidation no. shows that MnO2 undergo reduction and so it will work as an oxidizing agent and it will work as an oxidizing agent. Similarly in HCl chlorine is in –1 oxidation state while it become 0 in Cl2, the increase in oxidation and so it will act as a reducing agent.

20

YCT

40.

Two gases which can be used for the storage of fresh sample of an oil for a long time are (a) Nitrogen and Carbon dioxide (b) Nitrogen and Helium (c) Helium and Oxygen (d) Nitrogen and Oxygen Ans. (b) : Helium as well as nitrogen can be used for storage of a fresh sample of an oil for a long time because they abstain the oil from rancidity. The contact of air and oil is prevented by using nitrogen or helium as blanketing gas. 41. Two miscible liquids having boiling point 1320 C and 1500C can be separated by (a) Centrifugation (b) Distillation (c) Separating funnel (d) Fractional distillation Ans. (d) : A mixture of two miscible liquids can be separated by the process of fractional distillation. The separtation of two liquid by fraction distillation depends on the difference in their boiling points. 42. Aqueous solution of which one of the following carbon compounds would have the maximum concentration of hydrogen ions? (a) C3H7OH (b) C2H5CHO (c) CH3COCH3 (d) C2H5COOH Ans. (d) : C2H5COOH aqueous solution of which one of the following carbon compound would have the maximum concentrate of hydrogen ions. − + C 2 H5COOH C 2 H5COO + H 43.

Ans. (b) : The element is magnesium (mg) metal the balance equation when this oxide is dissolve in water given by

MgO (s) + H 2 O ( l )  → Mg(OH) 2( l ) The modern periodic table is belong to third period and second group.

45.

Which of the following statements are correct? (A) All metal oxides react with water to give salt and acid. (B) Some non-metal oxides react with water to form acids (C) Some of the metals react with acids to give salt and hydrogen gas (D) All metal carbonates react with acids to give salt, carbon dioxide and water (a) B and C (b) C and D (c) A, B and C (d) B, C and D Ans. (d) : All metal oxides react with water to give salt and acid is not correct. • Some non-metal oxides react with water to form acid. • Some of the metals react with acid to give salt and hydrogen gas. • All metal carbonates react with acid to give salt carbon dioxide and water.

46.

Which of the following is a thermite reaction (a) Fe(s) + CaSO4(aq) → FeSO4(aq) + Cu(s) (b) 3MaO2(s) + 4 Al(s) → 3 Mn(I)+2Al2O3(s) + Heat (c) Fe2O3(s) + 2Al(s) → 2 Fe(l) + Al2O3(s) + Heat

Which of the following sets of compounds depict same molecular formula but different structural formula?

(A)

CH3 | (B) CH 3CH 2CH 2CH 3 and CH 3 − C − H | CH3

Heat (d) Cu 2 O + Cu 2S  → 6Cu (s ) + SO 2( g )

Ans. (c) : The process of igniting aluminium and ferric oxide is called thermite reaction. This is an exothermic reaction. Iron obtained in this process is in molten state.

H H | | H − C − C − H

Fe 2 O3(s) + Al(s)  → Al 2 O3 2Fe( l ) + Heat Thermite reaction used for the welding of rails (Joining metals) Called as thermite welding. 44. A metal reacts with water to form an alkaline solution. If the aqueous solution of the chloride of this metal is known as brine, the period and group to which this metal belongs to in the Modern Periodic Table is (a) Second period, Third group (b) Third period, Second group (c) Second period, First group (d) Third period, First group

NVS TGT Science 2022

21

|

(C)

|

CH 3CH 2 CH 2CH 3 and H − C |

− C

H

H

− H

|

CH3 | (D) CH 3CH 2CH 2CH 3 and CH 3 − CH − CH 2 − CH 3 (a) B and D (c) B, C and D

(b) A, B and D (d) A and C

YCT

Ans. (c) : The sets of compound depict same molecular 49. formula but different structure formula, B, C and ‘D’ CH3

(a) 13 single and 1 double bond (b) 12 single and 1 double bond (c) 15 single bonds only (d) 10 single and 2 double bonds Ans. (c) : The molecular formula of cyclopentane is C5H10. In it structure-there are- ‘15’ single bond only. H H H

|

(B) CH 3CH 2CH 2CH 3 and CH 3 − C − H | CH3 (C) CH 3 CH2 CH2 CH3 and H

H

H

C

C

H

C

C

H

H

H

0

H

The molecular formula of cyclopentane is C5H10. In its structure there are

CH3 | (D) CH 3CH 2CH 2CH 3 and CH 3 − CH − CH 2 − CH 3 47.

H

A group of three metals which can be obtained 50. by the electrolysis of their molten chloride is (a) Aluminium, Zinc, Sodium (b) Calcium, Magnesium, Sodium (c) Sodium, Manganese, Magnesium (d) Potassium, Sodium, Nickel

Ans. (b) : Electrolysis is a process that leads to chemical change when an electric current is passed through an electrolyte. During the electrolysis process the negative charge electrodes loses the electron and undergoes oxidation while the positively charged electron gain the reduction process. Sodium and calcium are obtained by electrolysis of their chlorides in a molten state both reactive metal. Electrolysis of molten sodium chloride• During electrolysis of molten sodium chloride. • The chemical reaction at anode and cathode can be depicted At anode 2Cl–(aq)

C

C C

H H H

H H Cyclopentane C5H10. Consider the four carbon compounds A, B, C and D given below : (a)

(c)

(d)

Select the correct statement about their compounds from the following : (a) Addition of KMnO4 changes D to C (b) B cannot be obtained from C (c) A and C are saturated hydrocarbons (d) Addition to hydrogen in the presence of catalysts such as palladium converts B and A Ans. (d) : Addition to hydrogen in the presence of catalysts such as palladium converts B to A.

→Cl2(g) +2e–

The overall reaction can be depicted as. 2NaCl(l) → 2Na(S) +Cl2(g)

Select a covalent compound from the following : (a) NaCl (b) KNO3 (c) CCl4 (d) CaSO4

Ans. (c) : Carbon tetrachloride (Chemical formula) “CCl4” is known as covalent compound since it features four non polar covalent bond between carbon and 51. chlorine.

→ Na+ +Cl– (ionic compound) KNO3 → K+ + NO3– (ionic compound)

NaCl

CaSO4 → Ca 2+ + SO −4 (ionic compound)

NVS TGT Science 2022

C

(b)

At cathode 2Na + (aq) + 2e − → 2Na

48.

C H

22

The formula of a carbon compound X is C2H6O which is used as an antifreeze in car fuels. This compound on oxidation produces compound Y which on being treated with baking soda solution gives effervescence. X and Y respectively are YCT

(a) methanol and methanoic acid (b) ethanol and ethanal (c) ethanol and ethanoic acid (d) ethanone and ethanoic acid Ans. (c) : Carboxylic acid reacts with sodium hydrogen carbonate (Baking soda) to form effervescence. It is given that compound Y gives effervescence with baking soda solution. So, Y must be a carboxylic acid and ‘X’ must be an alcohol. As the compound C2H6O has two carbon atoms. So, ‘X’ is ethanol (C2H5OH). It is also used on antifreeze. The oxidation of ethanol in the presence of alkaline KMnO4 gives ethanoic acid. AlkMnO4 → CH 3COOH ( l ) + H 2 O (l ) C 2 H 5OH ( l ) + 2 [ O ] 

ethanol

ethanoic acid.

The reaction of ethanoic acid with baking soda produce sodium ethanoate water and carbon dioxide gas. CH 3 COOH(l) + NaNCO3 (aq) → CH 3 COONa(aq) + H 2 O(l) + CO 2 .

(D) Organisms repair and maintain their body structure The correct statements are : (a) A and B only (b) C and D only (c) A, B and D (d) A, B and C Ans. (c) : Energy is essential for life process inside every cell. Energy is required to break down and build up molecules, and to transport many molecules across plasma membranes all of life’s work needs energy. • Organisms grow with time because through tissues, cells grow and divide allowing organisms to grow. • Organisms repair and maintain their body structure because if they do not do this, then they will no longer be able to survice. • There is movement of molecules among cells by active and passive transport. Osmosis and diffusion are the most important transport phenomenon. 55. Consider the following statements about the events that occur during photosynthesis : (A) Reduction of CO2 to carbohydrates (B) Oxidation of carbon to CO2 (C) Absorption of photons (light energy) b chlorophyll (D) Conversion of light energy to chemical energy The correct statements are : (a) A and B only (b) C and D only (c) A, C and D (d) B, C and D Ans. (c) : Photosynthesis is the conversion of light energy into chemical energy by photosynthetic organisms, which are used to power cell activity. The correct order of steps during photosynthesis are. • Absorption of photons (light energy) by chlorophyll. • Conversion of light energy into chemical energy. • Splitting of water molecules into hydrogen and oxygen. • Reduction of CO2 to form carbohydrates. 56. In human beings the posture and balance of the body is controlled by (a) cerebellum (b) cerebrum (c) pons (d) medulla Ans. (a) : The cerebellum (“little brain”) is a structure that is located at the back of the brain, underlying the occipital and temporal lobes of the cerebral cortex. The cerebellum is important for making postural adjustments in order to maintain balance. It is a part of the hindbrain. Through its input from vestibular receptors and proprioceptors, it modulates commands to motor neurons to compensate for shifts in body position or changes in load upon muscles. Any irregularity or damage in the cerebellum can lead to problems with posture, balance, and uncoordinated voluntary motor muscles.

Therefore, X is ethanol (C2H5OH) and “Y” is ethanoic acid (CH3 COOH) X is ethanol (C2H5OH). 52. Which of these statements is not true for Mendellev's Periodic Table? (a) Elements were arranged on the basis of increasing atomic mass. (b) Anomalies regarding atomic mass were removed. (c) No place for elements yet to be discovered. (d) No proper place for hydrogen Ans. (b) : The various anomalies of Mendeleev's periodic table were removed by the Modern periodic table. Modern periodic table was based on the atomic number rather than atomic weights of elements. In Mendeleev's periodic table the wrong order of atomic masses of some element like cobalt with atomic mass 58.9 and nickel with atomic mass 58.7 could not be explained. 53. The part of alimentary canal which receives bile from the liver is (a) Oesophagus (b) Stomach (c) Large intestine (d) Small intestine Ans. (d) : Bile juice is produced in the liver and stored in the gall bladder. The small intestine is part of alimentary canal that receives the bile juice from the liver. The bile juice helps in the breakdown of fats. When the acidic food (Chyme) enters the small intestine, the bile juice pancreatic juice and intestinal juice are the secretions released into the small intestine.Pancreatic juice and bile are released into the small intestine through the hepoto-pancreatic duct. 54. Consider the following statements : (A) Energy is essential for life processes (B) Organism grow with time (C) There is no movement of molecules among cells NVS TGT Science 2022 23

YCT

57.

The growth of pollen tubes towards ovules within stylar tissue is due to (a) chemotropism (b) geotropism (c) hydrotropism (d) phototropism Ans. (a) : The growth of pollen tubes towards ovules within stylar tissue is due to chemotropism. Chemotropism is defined as the movement or the growth of the organism in response to a chemical stimulus. When a pollen grain comes in contact with the stigma, the ovary produces a positive chemical response for the pollen grain. This helps the pollen tube to grow towards the ovary for the process of fertilization. • Geotropism is the growth of the parts of plants in response to the force of gravity. • Phototropism is the ability of a plant to grow either by responding to the source of light or growing away from the source of light. • Hydrotropism is the growth or movements of plants towards water or environmental moisture.

Ans. (b) : Parents: Round green (RRyy) X Wrinkled yellow (rrYY) Gametes : RY rY F1 : RrYy (Round yellow) Selfing of F1 : RrYy X RrYy Gametes : Ry, Ry, rY X RY, Ry, rY, ry F2 : RRyy, RRYy, RrYY, RrYy (Round yellow) RRYy, RRyy RrYy, Rryy (Round yellow) RrYY, RrYy, RrYy, Rryy (Round yellow) rrYY, rrYy, rrYy (Wrinkled yellow) rryy (Wrinkled green) • So, the two new combination of characters obtained in the plants of F2 generation are round yellow and wrinkled green. 60.

58.

Consider the following statements : (A) Thyroxin regulates carbohydrates, proteins and fat metabolism of the body. (B) Iron is essential for the synthesis of thyroxin (C) Thyroxin is also called thyroid hormone (D) Thyroid glands require iodine for the synthesis of thyroxin The correct statements are : (a) A and B only (b) C and D only (c) A, C and D (d) B, C and D Ans. (c) : Thyroxine (T4) and Triiodothyronine (T3) are the two main thyroid hormones produced by the thyroid gland. Iodine is required for T3 and T4 producion in the hyroid gland. These hormones are responsible for carbohydrate, protein, and fat embolic regulation of the body. Thyroxine hormone maintains and regulates basal metabolic rate (BMR). • Iron is not essential for the synthesis of thyroxine.

59.

Ans. (d) : Variation refers to an individual with characteristics different from the others of the same kind. It results from change in genetic composition or genetic recombination. Genetic variations that alter gene activity or protein function can introduce different traits in an organism. • The selection of variants by environmental factors forms the basis of evolutionary processes. Advantageous variations are selected by environmental factors that leads to evolution and speciation. • Variation is minimum in asexual reproduction as changes in the DNA is the only contributing factor. • All variations in a species do not have equal chance of survival. Some of the variations may be so drastic that the new DNA copy cannot work with the cellular apparatus it inherits. Such a newborn cell dies soon.

Two pea plants, one with round green seeds (RRyy) and another with wrinkled yellow (rrYY) seeds produce plants of F1 generation that have round yellow (RrYy) seeds. When the plants of F1 generation were self-pollinated, the two new combinations of characters obtained in the plants of F2 generation were 61. (a) (Round, yellow) and (Round, green) (b) (Round, yellow) and (Wrinkled, green) (c) (Wrinkled, yellow) and (Round, green) (d) (Wrinkled, yellow) and (Wrinkled, green)

NVS TGT Science 2022

24

Consider the following statements. (A) Variation is minimum is sexual reproduction (B) Change in genetic composition results in variation (C) All variations in a species have equal chance of survival (D) Selection of variants by environmental factors forms the basis of evolutionary processes The correct statements are (a) A and C only (b) B and C only (c) A and D only (d) B and D only

Select from the following a medium having mass density and optical density, both lesser than those of water. (a) Alcohal (b) Benzene (c) Kerosene (d) Ice YCT

64. A student focused the image of a candle flame Ans. (d) : on a screen using a convex lens. He noted down The mass density of water = 997 (~1000) kg/m3 the positions of the candle, screen 104 and the Since optical density of water is directly proportional to lens on the scale as under : refraction index. Alcohol, benzene and kerosene have higher mass Position of candle = 10.0 cm density and optical density as compare to of water. Position of lens = 50.0 cm Ice is less denser than water. Also, the refractive index Position of screen = 90.0 cm of ice is 1.31, which is low compare to water (1.33). So, The power of the lens is: optical density of ice is also lesser than of water. (a) 2.5 D (b) 5.0 D 62. A ray of light propagating through a medium A (c) 10.0 D (d) 20.0 D enters another medium B such that the angle of incidence and angle of refraction are α and β. Ans. (d) : Using the formula 1 1 1 1 1 = − Given that sin α =   and sin β =   , the f v u x y where, refractive index of the medium A with respect to medium B is u = 50 – 10 = 40 u = – 40 cm x x (a) (b) v = 90 – 50 = 40 cm y y 1 1 1 1 = − (c) xy (d) f 40 − 40 xy 1 2 = Ans. (b) : Given, f 40 1 = 20.0cm f so, the power of the lens is 20.0 D. 65.

1 x 1 sin β = y

A person is using lenses of power +0.2 D in his spectacles for both his eyes. It means the person is suffering from (a) Myopia and the focal length of the lenses used is 50 cm (b) Myopia and the focal length of the lenses used is 5.0 m. (c) Hypermetropia and the focal length of the lenses used is 5.0m. (d) Hypermeropia and the focal lenses used is 50cm Ans. (c) : Given, Power of lenses P = +0.2 D 1 Since, P = D f (m)

sin α =

{nA = Refractive index of medium A

nB = refractive index of medium B Shell's Law, n A sin α = n B sin β



n A sin β = n B sin α



n A 1/ y x = = n B 1/ x y

63.

In case of human eye, when light rays falls on the eye, most of the refraction for the light rays occurs at the (a) crystalline less (b) iris (c) pupil (d) outer surface of the cornea Ans. (d) : In case of human eye, when light rays falls on the eye, most of the refraction for the light rays occurs at the outer surface of the cornea. The cornea is the transparent portion of eye that allows the light to enter the eye and is made up of transparent tissue. The cornea’s refractive power bends the light rays in such a way that they pass freely through the pupil. About 80% of the refraction occurs in the cornea and about 20% in the inner crystalline lens.

NVS TGT Science 2022

1 1 = = +5m = 500cm P +0.2 Since focal length of lenses is positive, i.e., it is convex lens. Convex lenses are used to solve the Hypermetropia. ⇒f =

66.

25

In the filament of an electric bulb a current of 1 ampere flows for 16 seconds. The number of electrons the pass through a cross-section of the filament during this period is close to (charge on an electron is 1.6 ×10-19 colomb) (a) 1016 (b) 1018 (c) 1020 (d) 1022 YCT

Ans. (c) : Given, current I = 1A Time t = 16 second. suppose the number of electrons passing through a cross-section of filament = n So, Total charge Q = ne = It It 1× 16 ⇒n= = e 1.6 ×10−19 ⇒ n = 1 × 1020 electrons 67. The potential difference between two points A and B is 6V. The amount of work done in moving a charge of 3.0 coulombs from A to B is (a) 0.5 joules (b) 2.0 joules (c) 18.0 joules (d) 54.0 joules Ans. (c) : Given, Potential VAB = 6V Charge Q = 3 coulomb Work done W = Q VAB = 3 × 6 = 18 Joule 68. Select the correct statement about the circuit diagram given below:

(a) This circuit can not be used to study the relation between the potential difference and current (b) The resistor, voltmeter and key are connected in series (c) No deflection will be observed in voltmeter and ammeter (d) The voltmeter is connected in parallel to the resistor whereas battery and ammeter are connected in series Ans. (d) : Since, Ammeter measures the current passing through it Ammeter is connected in series with resistance and battery. Voltmeter measures the potential difference across its end. That's why it is needed to connected in parallel with resistance. Here, voltmeter is connected in paralel with resistance. Thus, given circuit will be work properly and deflection will be observed in voltmeter and ammeter. 69. Assuming that the change in temperature of a conductor is negligible during a change in current flowing through it, if the current flowing through the conductor is increased accidentally by 100%, the increase in the power dissipated in the conductor would be (a) 100% (b) 300% (c) 400% (d) 800%

NVS TGT Science 2022

Ans. (c) : Let current flowing in conductor = I Now, 100% increase the current, then I' = I+I×100% I' = 2I Power in conductor P = I2R (Q R is Resistor) After increase in current, P' = I'2 R = (2I)2R P' = 4I2R = 4P increase in power dissipated = P' –P = 4P – P = 3P P '− P 3P Therefore, × 100% = × 100% = 300% P P

70.

From the given V-I graph for three resistors R1, R2 and R3 it may be concluded that

(a) R1>R2>R3 (c) R1=R2=R3

(b) R1R3

Ans. (b) : Let's see the given graph, The slope of V–I graph gives resistance (R). V As ohm's law, v = IR ⇒ =R I The order of slope, Slope of R3 > Slope of R2 > Slope of R1 ⇒ R3 > R2 > R1 71.

26

In an electric circuit two resistors of 4Ω and 8Ω are connected in series to a 12V battery. The heat dissipated by 8Ω resistor is 10s is (a) 160 J (b) 120 J (c) 80d J (d) 40 J

Ans. (c) : Given, Resistors 4Ω and 8Ω are connected in series. So, Req = 4 + 8 = 12Ω Voltage V = 12v V 12 So, current I = = = 1A R 12 Power dissipated by 8Ω resistor in 10s is : P = I2Rt = 1 × 1 × 8 × 10 P = 80 Joule YCT

72.

In the circuit shown, the ratio of current I1,  I1   l  is  

(c)

(d)

1 2 (a) (b) 5 5 3 4 (c) (d) 5 5 Ans. (a) : From given figure, 10Ω, 20Ω and 60Ω are connected in parallel. So, 1 1 1 1 6 + 3 +1 = + + = R 1 10 20 60 60

Ans. (b) :

74.

⇒ R1 = 6 Ω Also, R1 and 24Ω are connected in series. Then equivalent resistance of circuit R = 6 + 24 = 30Ω v v Total current i = i1 + i 2 = + R1 R 2

A constant current of 2A is flowing through a long thick copper wire held horizontally in east-west direction. If the direction of current in the wire is from west to east, the direction of the magnetic field produced would be from north to south directly

1 1  i = 12  +  {Q R 2 = 2uΩ}  6 24  5 5 i = 12 × = = 2.5A 24 2 I = i = 2.5A (a) Above only at the mid-point of the wire current flowing across 24Ω resistor is (b) Below only at the mid-point of the wire 12 I1 = = 0.5A (c) Above at all points of the wire 24 (d) below at all points of the sire I1 0.5 1 = = Ans. (c) : Right hand thumb rule states that if the right I 2.5 5 73. The circuit diagram in which the ammeter and thumb points in the direction of a current, the magnetic field at a point will be given by the direction of curl of voltmeter are connected property is fingers at that point. When thumb points in the west to east direction, then (a) the direction of magnetic field will be above at all points of the wise. 75. The magnetic field inside a long currentcarrying solenoid. (a) Decreases as we move towards the ends of the solenoid. (b) (b) Increases as we move towards the ends of the solenoid. (c) Is zero along the axis of the solenoid. (d) Is the same at all points inside the solenoid. NVS TGT Science 2022

27

YCT

Ans. (d) : The magnetic field inside a long current carrying solenoid is the same at all points inside the solenoid because the magnetic field lines are parallel to each other inside the solenoid. solenoid behaves as bar magnet. 76.

An electron enters a uniform magnetic field at right angles to it as shown. The direction of force acting on the electron is (consider the plane of the page to be horizontal)

(a) Vertically upward (b) Vertically downward (c) To the right (d) To the left Ans. (a) : According to Fleming's left hand rule, The direction of current, magnetic and force are perpendicular to each other. Also, the direction of current is opposite to the direction of flowing electrons. Here, magnetic field is in vertically upward direction

Ans. (d) : DC transmission lines are free from the skin effect. Hence, the effect resistance of line in small. There is no capacitance in the DC transmission. Thus, there is no power loss. In India, the standard frequency of household current is 50Hz. Therefore, The direction of alternative current changes after every (1/100) second. 78. In our country the major problem in harnessing nuclear energy is of (a) Converting nuclear energy into electrical energy (b) Establishing nuclear reactors (c) Splitting of nuclei (d) Safe disposal of the spent ‘fuel’ of the reactor Ans. (d) : Nuclear power plants produce radioactive wastes, such as Uranium mill tailings, spent (used) reactor fuel etc. These waste materials can remain radioactive and dangerous to human and animals. 79. In the following food chain if the energy available to the organisms of the fourth trophic level is 15J, the energy made available by the producer for the organisms of the immediate next trophic level is Grass →Grasshoper→Frog →Snake → Hawk (a) 150000J (b) 15000J (c) 1500J (d) 150J Ans. (b) : According to the 10 percent law, only 10 percent of the energy available in atrophic level is passed on to the next level. Hence, if the amount of energy available at the fourth trophic level is 15J, the energy available at the producer level will be 15000J. Grass → Grasshopper→Frog→Snake→Hawk (15000J) (1500J) (150J) (15J) (1.5J) 80.

77.

Consider the following statements: (a) Fleming’s left hand rule is used to determine the direction of force on a current-carrying conductor placed in a magnetic field (b) The right-hand thumb rule determines the direction of magnetic field due to a straight current carrying conductor (c) In our country the direction of alternative current changes after every 0.02 second (d) Advantage of DC over AC is that the electric power can be transmitted to distant places without appreciable loss of energy The correct statement are (1) A and B only (2) A and C only (3) B and D only (4) A, B and D

NVS TGT Science 2022

28

The pH of water sample collected from a river was found to be acidic in the range of 3.4 to 4.6. On the bank of the river wire several factories that wire discharging effects into the river. The effluents polluting the river and disturbing the pH of water are most likely to be of (a) Soap and detergent manufacturing factor (b) Lead battery manufacturing factory (c) Alcohal distillery (d) Plastic toys manufacturing factory Ans. (b) : Effluent is the waste water produced by various industries either during production process or industrial washings. The standard pH to be maintained before releasing effluents to the river is 5 to 8.5. If the effluents from the lead industry are released to the river, water turns more acidic and pH of river water reach value 3.5-4.5. Sulphuric acid is one of the major electrolyte used in lead industries that is highly acidic. The manufacturing factory of lead batteries used acids in lead batteries. YCT

PRACTICE SET - 1 1.

2.

3.

4.

5.

What will be the degree of freedom for a rigid body having N particles? (a) N (b) 3N (c) 3 (d) Infinite Which one of the following is a permanent effect? (a) Inductive effect (b) Electrometric effect (c) Resonance effect (d) Baker Nathan effect In a safety fuse, the temperature to which the gets heated is directly proportional to the (a) current (b) (current)2 3 (c) (current) (d) (current)4 Which material is used in transformer core? (a) Nickel (b) Soft iron (c) Steel (d) Copper r For the conservative electric field E − r r (a) div E = 0 (b) curl E = 0 b r uur r E.dl = 0 (c) (d) grad div E = 0



13.

14.

15.

16.

17.

a

6.

Which of the following particles will have the maximum frequency of revolution when projected with the same velocity v perpendicular to magnetic field ? (a) Electron (b) Proton (c) He+ (d) Li ++ 7. Which of the following quantities increases in a step-down transformer? (b) Voltage (a) Current (c) Power (d) Frequency 8. A D.C. equivalent circuit is derived from the original circuit by opening all (a) Resistors (b) Capacitors (c) Inductors (d) Transistors 9. The maximum scattering angle for equal masses in laboratory system is (a) Zero (b) 90º (c) 180º (d) 360º 10. Which of the following statement is correct about optically active substances? (a) Optically active substances produce polarized light. (b) Optically active substances rotate the plane of polarization of polarized light. (c) Optically active substances produce double refraction. (d) None of the above 11. Spherical aberration in a lens is due to (a) larger focal length for marginal rays as compared to paraxial rays (b) larger focal length to paraxial rays as compared to marginal rays (c) variation of refractive index with colour (d) irregularities on lens surface 12. Newton’s formula for the focal length of a thin lens is (a) xx' = –ff ' (b) xx' = ff ' (c) xx' = –f2f ' (d) xx' = f/f ' Practice Set 1

18.

19.

20.

The numerical aperture of a microscope is given by (a) µ/sin i (b) µsin i (c) sin i/µ (d) sin–1 i/µ If the refractive index of glass is 1.5, the speed of light in glass is (a) 2.3 ×108 m/s (b) 3.0 ×108 m/s (c) 1.5 ×108 m/s (d) 2.0 ×108 m/s Which mirror is best suited to focus parallel rays at a point? (a) Spherical convex (b) Spherical concave (c) Hyperbolic (d) Parabolic Select correct statement in which Doppler effect is not applicable(a) Discovery of twin stars (b) To determine the velocity of milky way (c) To determine the radius of earth (d) To determine the velocity of submarine Sound waves in hydrogen gas are of which type? (a) Transverse (b) Longitudinal (c) Stationary (d) Electromagnetic An external force F0 sinpt is applied on a particle, executing simple harmonic motion with natural frequency ω0 . In steady state, the frequency of oscillator will be (a) p (b) ω0 (c) ω 0 − p (d) p − ω0 Which of the following cannot represent an appropriate wave function, A & α are constants? (a) A sin αx (b) A cos αx − iαx (c) ae (d) Aeαx The expression for the velocity of longitudinal waves in gases is expressed as : (a) ν = Eρ

21.

22.

23.

29

(b) ν = E / ρ

(d) ν = Eρ (c) ν = E / ρ A driven (forced) harmonic oscillator has settled down to a steady state of oscillation. In steady state, the ratio of average power absorbed and average power dissipated will be 1 1 (a) (b) 2 3 (c) 1 (d) 2 The correct expression for the critical velocity vc, is : Kη Kρη (a) vc = (b) vc = ρr r Kρr Kρ (c) vc = (d) vc = η ηr The escape velocity of a body is proportional to nth power of its mass. Here n is. (a) Zero (b) 1 (c) 1/2 (d) 2

YCT

24.

25.

26.

27.

28.

29.

30.

31.

32.

33.

34.

The angular momentum of a planet of mass m about the centre of its circular orbit around the sun is : 1 (a) Am2 (b) Am 2 1 (c) 2 Am (d) Am2 4 When a explosive shell travelling in a parabolic path under the gravity explodes, the centre of mass of the fragments will continue to move : (a) along a hyperbolic path (b) vertically upwards and then vertically downwards (c) along the original parabolic path (d) horizontally and then follow the parabolic path Which of the following is conservative force? (a) Frictional force (b) Viscous force (c) Gravitational Force (d) None of the above The angular momentum of a moving body remains constant if – (a) Net external force is applied (b) Net pressure is applied (c) Net external torque (d) Net external torque is not applied Two masses of 2 kg and 4 kg are joined by a spring of force constant 1.33 N/m and are placed on a frictionless horizontal table. The masses are slightly drawn apart and released. The ratio of their kinetic energies shall be : (b) 1/2 (a) 2/1 (c) 3/4 (d) 4/3 A women is in a car on a level road sees an airplane travelling in the same direction that is climbing at an angle of 300 above the horizontal. By driving at 110km/h she is able to stay directly below the airplane. The airplane's velocity is : (a) 127km/h (b) 110 km/h (c) 220km/h (d) 144 km/h How much force is required to change the velocity of a body of mass 1 kg from 20 m/s to 30 m/s in 2 seconds? (a) 1 N (b) 5 N (c) 10 N (d) 25 N On the basis of ionic sizes, which of the salts given below will have maximum solubility in water? (a) Mg (OH)2 (b) Ba (OH)2 (c) MgCO3 (d) BaCO3 Given –ionic size (pm) Mg2+(65), Ba2+(135), OH–(140), CO32–(185) Which of the following acts as both a Bronsted acid and a Bronsted base? (a) HNO3 (b) HS– (c) H2SO4 (d) NH3 The surface tension of a liquid vanishes roughly X0C above the critical temperature. The value of X is (a) 40 (b) 20 0 (c) 6 (d) 30 Which of the following order of reaction is rare? (a) Zero order (b) First order (c) Second order (d) Fourth order

Practice Set 1

35. 36.

37. 38.

39.

40.

41. 42.

43. 44. 45.

46.

47.

30

Tyndall effect is shown by: (a) Zsigmondy (b) Langmuir (c) William and Chang (d) Ostwald A salt of Aluminium chloride is placed in water. After some time the pH of resulting solution is determined which of the following statements about pH of this solution is correct? (a) pH will be more than 7 (b) pH will be less than 7 (c) pH will be equal to 7 (d) pH will be zero The term gold number was coined by (a) Zsigmondy (b) Langmuir (c) William and Chang (d) Ostwald The precipitation of a salt takes place if its ionic product is: (a) greater than its solubility product (b) less than its solubility product (c) equal to its solubility product (d) None of these The effect of 2-fold pressure increase on the equilibrium composition of the reaction : N 2(g) + 3H 2(g)  2NH 3(g) is (a) 4-fold increase in kx (b) 2-fold increase in kx (c) 3-fold increase in kx (d) No effect is kx In kinetic studies of the chemical reaction the correct statement is : (a) Rate of a reaction may be independent of the concentrations of the reactants (b) The half life period of a second order reaction is independent of initial concentrations of the reactants (c) Molecularity of a reaction can be zero (d) The faster step is the rate determining step, if a reaction occurs in several steps Select the crystal system which has no symmetry (a) Tetragonal (b) Monoclinic (c) Triclinic (d) Hexagonal The collision theory or reaction rates is only valid for (a) Complex reactions (b) Parallel reactions (c) Simple bimolecular reactions (d) All the reactions The most acidic chloride is : (a) VCl2 (b) VCl3 (c) VCl4 (d) VOCl3 A process used for refining of metals is (a) Roasting (b) Smelting (c) Cupellation (d) Froth floatation process In which of the following pairs, compounds of the ion with lower oxidation state are more common and stable? (a) Cuperous, Cupeeric (b) Ferrous, Ferric (c) Thallous, Thallic (d) Mercurous, Mercuric Out of the following, which is most covalent halide? (a) AlI3 (b) AlCl3 (c) AlBr3 (d) AlF3 When pH reduces then oxygen binding power in Haemoglobin– (a) decreases (b) increases (c) remains unchanged (d) None of the above YCT

48.

In Schrodinger's wave equation Hψ = Eψ the symbol ψ represents (a) Energy of the spherical wave (b) Frequency of spherical wave (c) Amplitude of spherical wave (d) Laplacian operator 49. Which has maximum spin multiplicity? (a) p1 (b) p2 3 (c) p (d) p4 50. What will be the mass of 3.2 gram atoms of oxygen? (a) 10.24 gram (b) 102.4 gram (c) 1.024 gram (d) 1024 gram 51. Among the following the one that gives positive iodoform test upon reaction with I2 and NaOH is (a) CH3CH2CH(OH)CH2CH3 (b) C6H5CH2CH2OH

59.

60.

61.

62.

(c) (d) Ph CH OH CH3 63. 52. Which one of the following compounds does not give Cannizzaro's reaction? (a) Acetaldehyde (b) Benzaldehyde 64. (c) Formaldehyde (d) Trimethyl acetaldehyde 53. The product of the condensation : 65. p - CH 3. C6 H 4 C H O + (CH 3 CH 2 CO)2 O

54.

55.

(a) p-CH3. C6 H4 CH2 COOH (b) p-CH3. C6 H4 CH=C(CH3) COOH (c) p-CH3. C6 H4 CH=CH COOH (d) p-CH3. C6 H4 (CH3) C=CH COOH Which one of the following undergoes the 66. Diels-Alder reaction? (a) Thiophene (b) Pyridine (c) Pyrrole (d) Furan On treatment with bromine water, salicylic acid gives – 67. (a)

(b) 68.

(c) 56.

57.

58.

(d)

Electrolysis of concentrated solution of sodium propanoate produces the hydrocarbon 69. (a) methane (b) ethane (c) propane (d) butane Allene on reaction with dilute sulphuric acid gives (a) CH3CH2CHO (b) CH3COCH3 (c) CH3COCOOH (d) CH3CH2CH3 Which of the following CH3 – X bond has the 70. highest polarity? (a) C – Br (b) C – F (c) C – Cl (d) C – l

Practice Set 1

31

Which of the following alkyl halides will react according to SN1 mechanism : (a) 1-chloropentane (b) 2-methyl-2-chloropentane (c) Neopentyl chloride (d) 2-chloropentane Reaction intermediate of E 1cB reaction is (a) Carbo cation (b) Six-membered cyclic transition state (c) Carbanion (d) Carbene Both Prokaryotic and Eukaryotic cells have the common (a) Mitotic apparatus (b) Histone (c) Genetic code (d) Mitochondria Which among the following cell organelles is not surrounded by any membrane? (a) Lysosome (b) Ribosome (c) Plastid (d) Endoplasmic reticulum Chloride secreting cells of fishes are found in their : (a) Gills (b) Kidneys (c) Liver (d) Posterior pituitary In prophase of meiosis chiasmata are due to : (a) Mutation (b) Crossing over (c) Translocation (d) inversion Centriole takes part in : (a) initiation of plate formation (b) initiation of spindle formation (c) initiation of nucleus formation (d) initiation of cell division The process of determination of the sequence of genes on a chromosome is termed: (a) Gene localization (b) Gene sequencing (c) Chromosomal mapping (d) DNA sequencing Homologous structures found in some species have no clear function. These are called : (a) Underdeveloped organ (b) Dormant organ (c) Dwarf organ (d) Vestigial organ 'Ontogeny recapitulates phylogeny-this phenomenon is explained by : (a) Mutation theory (b) Inheritance theory (c) Recapitulation theory (d) Natural-selection theory Lamarck theory of organic evolution is usually known as : (a) Natural selection (b) Inheritance of acquired character (c) Descent with change (d) Continuity of germplasm The latest theory which explains evolution is : (a) Lamarckism (b) Darwinism (c) Mutation theory (d) Synthetic theory YCT

71.

72.

73.

74.

75.

76.

77.

78.

79.

80.

81. 82. 83.

84. 85.

The digestive tract of human adult lacks_____enzyme. (a) Pepsin (b) Lipase (c) Rennin (d) Trypsin Volume of air inspired or expired during a normal respiration is termed as (a) Tidal volume (b) Residual volume (c) Vital capacity (d) Inspiratory reserve volume Hamburger's phenomenon is also called (a) Bicarbonate Shift (b) Chloride Shift (c) Hydrogen shift (d) Sodium shift Urinary bladder in birds is(a) One (b) Two (c) Well developed (d) Absent The nervous system develops from which germinal layer ? (a) Ectoderm (b) Mesoderm (c) Endoderm (d) None of these Which of the following biogeochemical cycles is a sedimentary cycle? (a) Carbon cycle (b) Sulphur cycle (c) Nitrogen cycle (d) Hydrogen cycle Stratification can be seen best in : (a) Tropical rain forest (b) Grassland (c) Stream (d) Pond In the Wild Life (Protection) Act, 1972, important wild animals have been placed in (a) Two Schedules (b) Three Schedules (c) Five Schedules (d) Six Schedules Which of the following is a primary air pollutant? (a) Ozone in troposphere (b) Dioxides of carbon, sulphur and nitrogen (c) Ozone in stratosphere (d) SO3 Which of the following represents the probability, P of an event which is certain? (a) P = 1 (b) P = 10 (c) P = 50 (d) P = 100 Chloroplasts contain maximum quantity of : (a) RUBP carboxylase (b) Hexokinase (c) Pyruvate carboxylase (d) None of these The metal involved in stomatal regulation is : (a) Iron (b) Magnesium (c) Zinc (d) Potassium Nilgiri Biosphere Reserve do NOT stretch over which of the following States in India ? (a) Tamil Nadu (b) Andhra Pradesh (c) Karnataka (d) Kerala One fibril of cell wall is made-up of : (a) 20 microfibrils (b) 100 microfibrils (c) 250 microfibrils (d) 50 microfibrils CO2 acceptor in sugarcane is : (a) Phosphoglycolic acid (b) Oxaloacetate (c) RuBp (d) Phosphoenolpyruvic acid

Practice Set 1

86.

Researcher himself is a source of : (a) Primary (b) Secondary (c) Tertiary (d) None of these 87. In eukaryotic cell cycle, nuclear DNA synthesis occurs during : (a) M-phase (b) S-phase (c) G1-phase (d) G2-phase 88. In photosynthesis, which of the following absorb light energy of sun and changes it to chemical energy ? (a) Chlorophyll b (b) Chlorophyll c (c) Chlorophyll a (d) Xanthophyll 89. Who coined the term ecotype? (a) Odum (b) Turesson (c) Tansley (d) Warming 90. Synthesis of amino acid by a mixture of gases under artificial conditions was done by : (a) Oparin (b) Fox (c) Woese (d) Miller 91. Upward translocation of water in a plant is called correctly : (a) Transpiration (b) Absorption of water (c) Ascent of water (d) Ascent of sap 92. Competition between the individuals of the same species is called : (a) Specific struggle (b) Interspecific struggle (c) Intraspecific struggle (d) Extra-specific struggle 93. Division of cytoplasm after completion of nuclear division is called : (a) Cytomixis (b) Cytokinesis (c) karyokinesis (d) Apomixis 94. The site of dark reaction of photosynthesis is: (a) Grana (b) Stroma (c) Mitochondria (d) F1 particle 95. Which of the following is not a component of nitrogenase complex? (a) MO (b) Fe (c) Zn (d) S 96. Cell organelles can be physically separated of each other by : (a) Centrifugation (b) Microscopy (c) Chromatography (d) Radioactive tracers 97. In which of the following family C3, C4 and CAM plant species are found? (a) Crassulaceae (b) Cactaceae (c) Poaceae (d) Euphorbiaceae 98. Karyotype refers to : (a) Chemical composition of chromosomes (b) Phenotypic appearance of chromosome set (c) Phenotypic appearance of one chromosome (d) genetic map of chromosome 99. The transitional zone between two distinct communities is known as : (a) Ecosphere (b) Ecotone (c) Ecocline (d) Rhizosphere 100. Enzyme concerned with ammonia assimilation is : (a) Nitrogenase (b) Arginase (c) Urease (d) Glutamine synthetase

32

YCT

SOLUTION : PRACTICE SET- 1 ANSWER KEY 1. (b) 11. (b) 21. (c) 31. (b) 41. (c) 51. (d) 61. (c) 71. (c) 81. (a) 91. (d)

2. (a) 12. (b) 22. (a) 32. (b) 42. (c) 52. (c) 62. (b) 72. (a) 82. (d) 92. (c)

3. (b) 13. (b) 23. (a) 33. (c) 43. (d) 53. (c) 63. (a) 73. (b) 83. (b) 93. (b)

4. (b) 14. (d) 24. (c) 34. (d) 44. (c) 54. (d) 64. (b) 74. (d) 84. (c) 94. (b)

5. (b) 15. (d) 25. (c) 35. (a) 45. (c) 55. (c) 65. (b) 75. (a) 85. (d) 95. (c)

6. (d) 16. (c) 26. (c) 36. (b) 46. (a) 56. (d) 66. (c) 76. (b) 86. (a) 96. (a)

7. (a) 17. (b) 27. (d) 37. (a) 47. (a) 57. (b) 67. (d) 77. (a) 87. (b) 97. (d)

8. (b) 18. (a) 28. (a) 38. (a) 48. (a) 58. (b) 68. (c) 78. (c) 88. (c) 98. (b)

9. (b) 19. (d) 29. (a) 39. (a) 49. (c) 59. (b) 69. (b) 79. (b) 89. (b) 99. (b)

10. (b) 20. (c) 30. (b) 40. (a) 50. (b) 60. (c) 70. (d) 80. (a) 90. (d) 100. (d)

SOLUTION 1. (b) Rigid body is a system of mass points subjected to the holonomic constraints that the distance between all pairs of points remain constant throughout the motion. The degree of freedom for a rigid body having N particles is given by 3N. It is the maximum possible degree of freedom for N particle of rigid body with any constraints. 2. (a) Inductive effect is a phenomenon in which there is a permanent displacement of shared electron pair in a carbon chain towards more electronegative atom. It is transmitted through sigma bond and produces a permanent state of polarization. The effect involving the complete transfer of e– s to one of a atoms joined by a multiple bond (= or ≡) at the requirement of attacking reagent is called Electrometric effect. It is a temporary effect. Resonance effect is a electron redistribution that can take place in unsaturated and especially in conjugated system via their π-bonds. The delocalization of σ-electrons or lone pair of electrons into adjacent π-orbital or p-orbital is called Hyper conjugation or Baker Nathan effect. Note:- Question deleted by Commission 3. (b) From Joule's heat effect, H = i2Rt ⇒ H∝i2 That is heat ∝ (current)2 4. (b) The transformer core is made of soft iron because it has high permeability so it provides complete linkage of magnetic flux of the primary coil to the secondary coil. Therefore it has high coercivity and low retentivity. The hysteresis loss of soft iron is very less. 5. (b) For the conservative electric field, ur b E.dl = 0 ⇒ ∇ × E = 0

∫ a

6. (d)

Since E = hv 1 ⇒ hv = mv2 2 ⇒v∝m

Practice Set 1

Since Li has highest mass hence its frequency of revolution is maximum. 7. (a) When a high potential is passed in the primary coil of a step-down transformer, a low potential is obtained in its secondary coil. Therefore the current in the step-up transformer increases. 8. (b) A DC equivalent circuit can be derived from the original circuit by following ways (i) Replacing all capacitances with open circuits. (ii) Inductances with short circuit. (iii) Reducing AC sources to zero (i.e. replacing AC voltage sources by short circuits and AC current sources by open circuits). 9. (b) Scattering angle in laboratory system is,

m  ϕmax = sin −1  2  (where m1 & m2 are masses)  m1  Given m2 = m1 So ϕmax = sin −1 (1) ϕmax = 90º 10. (b) Optically active substance rotate the plane of polarization of polarized light by the interaction of the electromagnetic radiation of polarized light with an unsymmetrical electric field generated by the electrons in a chiral molecule (molecules having nonsuperposable mirror image). e.g. : Optically active crystals - Quartz and cinnabar optically active solutions - sugar, tartric acid. 11. (b) In spherical aberration rays of light from a point on the optical axis of a spherical lens don’t meet at the same image point. Spherical aberration occurs when paraxial rays have a larger focal length than the marginal rays, both types of rays can't meet at principal focus due to which the image will be blurred. 12. (b) Newton's formula for the focal length of a thin lens isx f' = f x' 33 YCT

Where , x = distance of an object from focus f y = A cos αx x ' = distance of an object from focus f ' y = Ae − iKx Therefore, xx ' = ff ' [k = α] = Ae − iαx ≅ Note - f = –f ' 2 20. (c) xx' = – f Longitudinal wave : 13. (b) The waves in which displacement of particle is The numerical aperture of a microscope is in parallel to the direction of wave propagation. N.A. = µ sin i Speed of longitudinal waves in gas given by. Where µ → Refractive index E Vv = i → Half of the maximum cone of light that can enter ρ the lens. For the different medium, E represent the different 14. (d) quantity Given For solid medium E represent young modulus. Refractive index of glass (n) = 1.5 For liquid medium E represent shear modulus 8 Velocity of light = 3 × 10 m/s For gas medium E represent Bulk modulus. Velocity of light 21. (c) The speed of light in glass = Before achieving steady state, oscillator consumed n some energy to increase its temperature but after 8 3 × 10 achieving steady state it does not consumed energy and V= 1.5 dissipate energy equal to amount of energy absorbed before achieving steady state. So, V = 2 × 108 m / s Average power absorbed =1 15. (d) Average power dissipated Parabolic mirror is best suited to focus parallel rays at a 22. (a) point. v ρr from the formula of Reynold's number K = c η Kη Hence, critical velocity = vc = ρr where, vc = Critical velocity, ρ = Density of the liquid, r = radius of the tube, η = coefficient of viscosity Note:- The spherical concave mirror can also focus parallel rays at a single point but the accuracy of 23. (a) parabolic mirror is more because spherical concave Escape velocity is the minimum velocity required to mirror concentrate incoming parallel rays into a larger escape the gravitational field of the earth. volume than a diffraction-limited spot. 2GM The escape velocity Ve = 16. (c) R Doppler effect is a phenomenon which is observed Where G is universal gravitational constant when there is a change in frequency of sound or light when a source is moving with respect to an observer. M is mass of earth Increase in frequency represents moving of source R is radius of earth towards observer and decrease in frequency represents So escape velocity is independent of the mass of body. moving away of source from observer. It is used in 24. (c) discovery of twin stars, to determine the velocity of By kepler's law, Milky Way, to determine velocity of submarines etc. dA ' L = (L = Angular momentum, m = mass of the 17. (b) dt 2m Sound waves in hydrogen gas are of longitudinal type. In this individual Particles of the medium execute planet dA ' = A ) simple harmonic motion about their mean position dt along the same direction in which the wave is L propagated. A= 2m 18. (a) An external force F0 sin pt is applied on a particle, ⇒ L = 2Am executing simple harmonic motion with natural 25. (c) frequency ω0. In steady state the frequency of oscillator Moving on a parabolic path under the influence of gravity, the center of mass of the pieces of an explosive will be p. sphere will continue to move along the same parabolic 19. (d) path. The reason for this would be the velocity and Only possible wave function aredirection of explosives and their components would be y = A sin αx where α & A is constant the same. Practice Set 1 34 YCT

26. (c) Conservative force : A force said to be conservative if the work done by or against force is dependent only on the initial and final position of the body and not on the path followed by body. Example - Gravitational force, electrostatic force etc. Non conservative force : A force is said to be nonconservative if the work done by or against the force is dependent on the path, followed by body. Example-Friction, Viscous force, induction, force in cyclotron etc. So gravitational force is conservative force. 27. (d) If the total external torque exerted an a system of particles is zero, then the total angular momentum of that system is conserved i.e. remains constant. τext = 0 dL =0 dt ∴ L = Constant 28. (a) p2 K= 2m 1 K∝ (P is constant) m K1 m 2 2 = = K 2 m1 1 29. (a) Given, θ = 30º, velocity of the plane · v velocity of the car = 110 km/h According to question, v cos 300 = 110 110 110 v= = cos 30 3

The bronsted base is any species that converts a proton to an aceptor in. HS– + H +  → H 2S Bronsted base Conjugate acid HS– – H +  S2– → Bronsted base Conjugate base

33. (c) from the Ramsay shield equation 2/3

M λ   = k ( tc − t − 6 ) P The surface tension of a liquid becomes zero at about 6°C above the critical temperature. 34. (d) Fourth order reaction is rare because of the low probability of the reacting species colliding together. 35. (a) If a strong converging beam of light is passed through a colloidal solution placed in dark room, the path of beam gets iluminated with a bluish light when viewed at right angles to the direction of the passage of light, the path of the light becomes visible due to scattering of light by the colloidal partides the phenomenon was observed by Tyndall in 1869 and is called tyndall effect. Tyndall effect may be defined as the scattering of light by the collodial particles present in a colloidal solution. There is a large difference in the dispersed particles is not much smaller as compared to the wavelength of the light used, there is large difference in the rafractive indices of the dispersed phase and the dispersion medium. Important tyndall effect based on tyndall effect. Zsigmondy in 1903 discovered in instrument called ultramicroscope. 36. (b) AlCl3+3H2O→Al (OH)3 + 3HCl Here AlCl3 salt react with water and produce strong acid of HCl in solution. So value of pH is less than 7. 37. (a) Protective properties of a lyophilic colloid, to express the protective properties zsigmondy determined the 2 principle of gold number, the gold number is defined as ⇒ v = 127km / h follows. the amount in a milliliter of a protective calloid 30. (b) that prevants possible coagulation by 10 me of gold sol Force is given by, (anti- liquid colloid) in a 10% ml solution of NaCl, then F = m×a the gold number the lower the gold number of a protective colloid, the greater is its protective power. Where m = mass of body 38. (a) a = acceleration of body The precipitation of a salt takes place if ionic product is greater than its solubility product.  V2 − V1  F = m×  Ionic product > solubility product  t  or IP > KSP  30 − 20  39. (a) F = 1×   2  As the pressure is increased, the volume decreases concentration by decreasing the volume concentration 10 F = 1× increases. the concentration doubles in the forward 2 direction due to decrease in volume, this is a four - fold F = 5N increase in 31. (b) N2(g) + 3H2(g)  2NH3(g) According to Fajans' rule, The greaer the size of the 40. (a) cation in a salt and the in water. Thus Ba(OH)2 will The rate of a reaction can be independent of the have the highest solubility in water. concentration of the reaction, for example- in a zero 32. (b) order reaction, the rate of reaction is independent of the concentration of the reactants. Bronsted acid is any species, who is able to donate H+. Practice Set 1

35

YCT

41. (c) Any symmetry in a triclinic (symmetry) is not found. Its axial angle (α ≠ β ≠ γ ≠ 90°) is not equal and at the same time the unit length (a ≠ b ≠ c) is also not equal. Example: turquoise, rnodonite, serenite, aximite microcline, wollastonite etc. 42. (c) Collision theory is applicable to bimolecular reaction and reaction with molecularity greater than two. It is not applicable to unimolecular reaction. The basic requirement of collision theory is that the reacting species (atoms, ions or molecule) must come to gether and collide in order for the reaction to occur collision are possible in bi-molecular reaction. Collision theory of reaction rate depending on the reaction simple bimolecular does is. 43. (d) VOCl3 is the most acidic chloride because its oxidation state is +5. V2O5 + HCl → VOCl3 + H2O 44. (c) Cupellation is a process where ore are treated under high temperature and controlled operation to separate moble metal like gold and silver, from base metal like led, copper, zinc and other present in the ore or alloyed metal. 45. (c) Among following pairs compound of the ion with lower oxidation state are more common and stable is thallous thalic. In the 13th group, thallium can exibit 1 + 1 and +3 oxidation state but it is stable in + 1 oxidation state only due to inert pair effect. In the 14th group lead shows both +2 and +4 oxidation state but it is stable in +2 oxidation state due to inert pair effect. 46. (a) The most covalent halide is AlI3 Since lesser the electronegativity difference, more covalent is the aluminium halide. Hence the most covalent halide among the given option is aluminium iodide (AlI3). 47. (a) Red blood cells contain a protein pigment called haemoglobin due to which the colour of these particles is red. Haemoglobin is composed of a protein globin (96%) and a pigment heme (4%) at the center of the heme molecule is iron which has the ability to bind and release oxygen. Haemoglobin transport oxygen in blood cells in the addition to convention by controling the acidity alkalinity in the body's internal environment Haemoglobin performs the important function of maintaining it pH in suitable state. Haemoglobin to O2 when pH decreases due to Bohr effect the attraction power of decreases on lowering the pH value, the oxygen binding capacity of Haemoglobin decreases. 48. (a) In Schrodinger's wave equation Hψ = Eψ the symbol ψ represents the energy of the spherical wave. 49. (c) Hunds rule of maximum multiplicity rule state that for a given electron configuration the term with maximum multiplicity falls lowest in energy according to this rule electron pairing in p, d and f orbital can not occur unil each orbital of a given subshell contains one electron each is singly occupied. Practice Set 1

Hence, the correct option (c) p3 is a half filled orbital show. So it is stable.

50. (b) The mass of 3.2 gram molecule of oxygen 102.4g. 1 gram molecular mass oxygen = 32 g given 3.2 gram molecule of oxygen, therefore 3.2 gram molecule of oxygen = 3.2 × 32 = 102. 4g 51. (d) Alcohol react with (I2) and NaOH and gives the iodoform test. Methyl alcohol does not give iodoform reaction, ethyl alcohols, acetaldehyde, acetone and those alcohols containing CH2–CH (OH) – group give iodoform test. 52. (c) Acetaldehyde does not undergo cannizzaro reaction because it retains hydrogen is cannizzaro contain αhydrogen cannizzaro reaction benzaldehyde, formaldehyde and trimethyl are found in such acetaldehyde becauseit does not contain α-hydrogen. O

O

1. CH3 – C – H + CH3 – C – H NaOH O 2.

O

O

H–C–H +H–C–H

No reaction

NaOH

CH3–OH + Na + O – C – H

53. (c) A condensation reaction is a reaction in which two molecule combine to form a single molecule a small molecule, often water, is usually removed during a condensation reaction,

54. (d) Furan undergoes Diels-Alder reaction. Because the electronegativity of the oxygen atom is high. Furan has the least aromatic property compared to pyrrole and thiophene, which behaves as a diene in the Diels-Alder reaction.

55. (c) Salicyclic acid reacts with bromine water to form 2, 4, 6-tribromophenol.

36

YCT

is composed of one or more r RNA molecules and a 56. (d) Electrolysis of concentrated solution of sodium variety of proteins. It is devoid of any membrane. propanoate produces the butane hydrocarbon. This is 63. (a) the Kolbe electrolytic method alkenes are formed when Chloride secreting cells of fishes are found in their gills. saturated monocarboxylic acid are electrolysed in These cells in the gills of marine bony fish remove concentrated aqueous solution of potassium or sodium chloride from the blood and secrete it into the sea, there salts. by maintaining the osmotic composition of the body. O These cells are also said to make their appearance in || electrolysis fresh-water fishes that have been experimentally 2R − C − O Na + + 2H 2 O  →R −R + subjected to a saline medium. These are mitochondrion 2CO2 + 2 NaOH rich cell involved in osmoregulation process is fishes. electrolysis 2C2 H5COONa + 2H2O  → C2H5 − C2 H5 + 2CO2 + 2NaOH 64. (b) Butane In prophase of meiosis, chiasmata is formed in the 57. (b) diplotene phase of prophase-I. It is the point of react attachment between non-sister chromatids of two Allene with dilute sulfuric acid (dilute H2SO4) homologous chromosomes. Crossing over is the exchange of genetic material between non-sister with 2-propanone chromatids of homologous chromosomes, which results in new allelic combinations in the daughter cells. 65. (b) The centrosome is a cellular organelle which is the main microtubule organizing centre is an animal cell which 58. (b) also regulates the process of cell-cycle. They are made The decresing order of electro negativity F>Cl>Br>I up of 2 centrioles which are right angled to each other CH3 – X bond where x is halogen. The difference between carbon and halogen bond and is composed of a protein known as tubulin. Its main electronegativity gives act polarity. The electro function is to organize microtubules (spindle fibres) and pull chromatids apart during cell divison.) negativity of halogens are: F (4.0), Cl (3.0), Br (2.8) and I (2.5). 66. (c) Hence, 'C-F' bond has the highest polarity because The process of determination of the sequence of genes on a fluorine is most electronegative. chromosome is termed as Chromosomal mapping. In order (C-F)>(C-Cl)>(C-Br)>(C-I) to map DNA segments on specific chromosomes it is 59. (b) necessary to test a number of close family relatives. Substitution reaction among tertiary alkyl halides 1 mainly SN type reaction take place. The order of this 67. (d) reaction is one. Only one molecule participates in their Homologous structures found in some species have no slow step. If the carbon atom is asymmetric in then as a clear function. These are called vestigial organ. The key difference between homologous structures and vertigial result a racemic mixture is obtained. structures is that homologous structures are the 60. (c) Although E1 reaction typically involve a carbocation anatomically similar structures found in different intermediate the E1 CB reaction reaction utilizes a organisms that share a common ancestor, while vestigial carbonion intermediate. A proton adjacent to a carbonyl structures are the anatomical structures which have lost group is removed strong base this proton is acidic their usefulness to an organism. Examples of vestigial because the resulting conjugate base anion is stabilized structure include the human appendix, the pelvic bone of a snake, and the wings of flightless birds. by delocalization on to the carbonyl group. 68. (c) 61. (c) Ontogeny recapitulates phylogeny, this phenomenon Both prokaryotic and eukaryotic cells have the common is explained by recapitulation theory. genetic code. It is the sequence of nucleotides in DNA and RNA that determines the amino acid sequence of Recapitulation theory was given by Ernst Haeckel in proteins. It is a set of rules defining how the four letter 1866. It states that 'ontogeny repeats phylogeny'. code of DNA is translated into the 20-letter code of Ontogeny is the life history of an organism while amino acids. The genetic code is universal. All known phylogeny is the evolutionary history of the race of living organisms use the same genetic code. that organism. This means that an organism repeats its 62. (b) ancestral history during its development. Eukaryotic nucleus in enclosed by a double membrane 69. (b) called as the nuclear envelope. Chloroplasts have outer Lamarck's theory of organic evolution is usually known and inner membrane separated by a intra membranous as inheritance of acquired character. This theory states space of 10 to 20 nm. Mitochondria too has inner and that modifications which the organism acquires in outer membrane. The inner membrane folds to from adaptation to the environments which it meets during its cristae. lifetime are automatically handed down to its Ribosomes are the translational apparatus of the cell descendants and so become part of heredity. with two major components the small ribosomal subunit Lamarckism is the first theory of evolution, which was which reads the RNA, and the large subunit which joins proposed by Jean Baptiste de Lamarck, a French amino acids to form a polypeptide chain. Each subunit biologist.

Practice Set 1

37

YCT

70. (d) The latest theory which explains evolution is synthetic theory. The modern synthetic theory of evolution describes the evolution in terms of genetic variations in a population that leads to the formation of a new species. This theory is also referred to as the Neo-Darwinian theory and its merges the concept of Darwinian evolution with Mendelian genetics. It was introduced by a number of evolutionary biologist such as T. Dobzhansky, J.B.S. Haldane, R.A. Fisher, Sewall Wright, G.L. Stebbins and Ernst Mayr. 71. (c) Rennin is the enzyme present in the gastric juice of only infants and not of adult human. In adults, it is replaced by prorennin that is activated by H+ ions to form rennin. Its function is to coagulate milk. The optimal pH for the enzyme is 1.8 72. (a) Tidal volume is the lung volume that measures the amount of air that is inspired and expired during a normal breath, extra effort is not applied. The average human has a tidal volume (TV) of approximately 500 ml. 73. (b) The Humburger process is called chloride shift. The exchange of Chloride and bicarbonate between plasma and erythrocytes is called chloride shift. This process takes place when oxygen is released from the blood and carbon dioxide is received. This method help in establishing an equilibrium between the cell and the fluid found around it. 74. (d) Urinary bladder in birds is absent. Birds have kidneys that are involved in the removal of nitrogen, but do not have urinary bladder. Birds release waste through an organ, called cloaca. Urine and feces both drain into the cloaca, where they mix together before being expelled. Liquid waste is sent straight from the kidneys to the cloaca. Birds like ostrich and American rhea are exceptions that secretes urine separated from feces. 75. (a) The nervous system develops from ectoderm. Following fertilisation the nervous system begins to form in the 3rd week of development. Gastrulation leads to the formation of three germ layers, ectoderm (outer), mesoderm (middle) and endoderm (inner). The ectoderm develops into the surface ectoderm, neural crest and the neural tube. The neural tube of ectoderm develops into brain and spinal cord. After gastrulation, the embryo goes through a process called neurulation which starts the development of nervous system. 76. (b) Sulphur cycle is a sedimentary cycle. ⇒ A biogeochemical cycle is the pathway by which a chemical substance cycles is turned over or moves through the biotic and the abiotic components of earth. Biogeochemical cycle important to living organisms include the water, carbon, nitrogen, phosphorus and sulphur cycles. ⇒The sulphur cycle is a biogeochemical cycle in which the sulphur moves between rocks, water ways and Practice Set 1

living systems. Sedimentary cycle includes phosphorus, sulphur, calcium and magnesium cycle. 77. (a) Stratification can be seen best in tropical rain forest. It is the vertical distribution of various species in different layers structurally. The tropical rain forest is considered to have five strata ; three tree layers, a layer of shrubs and a ground layer of herbaceous plants. The tropical rainforest is earth's most complex biome in terms of both structure and species diversity. Presence of these different layers and vegetation helps in growing a balanced ecosystem. 78. (c) In the wildlife (Protection) Act, 1972, important wild animals have been placed in schedule five. The Act was enacted for the protection of plants and animal species. It has divided the protection status of various plants and animals under six schedules. Schedule five contains small wild animals that carry disease and destroy plants and food. These animals can be hunted. It includes four species of wild animals; common crows, fruit bats, rats and mice. 79. (b) A primary pollutant is an air pollutant emitted directly from a source e.g. carbon monoxide (CO), oxides of nitrogen (NOx, NO) sulfur oxides (SOx), SO2, volatile organic compounds (VOCs) and Particulate matter (dust, ash, salt Particles). They further react in the Presence of sunlight to form secondary pollutants such as ozone. 80. (a) In P = 1, P represents the probability. Probability means possibility. It is a branch of Mathematics that deals with the occurence of a random event. The probability of an event is a number between 0 and 1 where 0 indicates impossibility of the event and 1 indicates certainty. 81. (a) Pyruvate carboxylase is a mitochondrial enzyme that catalyzes carboxylation of pyruvate into oxaloacetate. Hexokinase is an enzyme of glycolysis and is present in the cytoplasm. RUBP carboxylase catalyzes carboxylation of RUBP during Photosynthetic carbon fixation. Chloroplasts is the site of photosynthesis and contain maximum quantity of RUBP carboxylase. 82. (d) The metal involved in stomatal regulation is Potassium. Regulation of transpiration is achieved primarily through the opening and closing of stomata on the leaf surface. Stomata are surrounded by two specialized cells celled guard cells. Increase in K+ ion concentration in the guard cells makes them hypertonic, so more water is drawn inside the cells. When plants are K+ ion deficient, these guard cells do not function properly and moisture can escape. 83. (b) Nilgiri Biosphere Reserve do not stretch over Andhra Pradesh. The biosphere shares its boundaries with Tamil Nadu, Karnataka and Kerala. It is located in the Western Ghats and Nilgiri Hills and covers an area of 5520 km2. The Nilgiri Biosphere reserve was declared as World Heritage site by UNESCO in 2012.

38

YCT

84. (c) One fibril of cell wall is made up of 250 microfibrils. Microfibril is a very thin, filament-like structure made up of glycoproteins and cellulose. On absorption of water, the volume of cell increase and the existing microfibrils separate and new ones are formed to help increase cell strength. 85. (d) PEP is 3C compound which serves as primary CO2 acceptor in the mesophyll cell cytoplasm of C4 plants like maize sugarcane, sorghum etc. Sugarcane is a C4 plant. In C4 plants, the primary acceptor of carbon dioxide is PEP (Phosphoenolpy-ruvate). PEP combines with CO2 and forms a 4- carbon compound. 86. (a) The researcher himself is a primary source. Primary consumers are those and nutrients. In ecology, organisms that feed on autotrophs are classified as consumers. 87. (b) Nuclear DNA synthesis occurs during S-phase of mitotic interphase. During the S-phase or synthetic phase of interphase, replication of DNA (DNA doubling) and synthesis of histone proteins occur and centriole duplicates in the cytoplasm. 88. (c) In photosynthesis, chlorophyll a absorb light energy of sun and changes it to chemical energy. Chlorophyll a is the primary pigment of photosynthesis, that absorbs light from the orange-red and violet-blue areas of the electromagnetic spectrum, and transfers energy to the reaction center and donates two excited electrons to the electron transport chain. From there on, the energy from the sun will ultimately become chemical energy that can be used by the plants. 89. (b) The term ecotype was proposed by Turesson in 1922. The subpopulation of genetically distinct species confined to a specific habitat area is called an ecotype. 90. (d) Synthesis of amino acid by a mixture of gases under artificial conditions was done by Stanley Miller. Miller and Urey provided first proof for chemical synthesis of proteins by amino acids. They simulated the primitive earth conditions as proposed by Oparin and synthesized simple organic compounds (amino acids, fatty acids, sugars etc) from a mixture of methane, hydrogen, water vapour and ammonia. As in primitive atmosphere, hydrogen served as reducing agent while methane provided hydrogen skeleton for synthesis of organic compounds. 91. (d) Upward translocation of water in a plant is called Ascent of sap. In plants, minerals and water are transported through the xylem tissue from the soil to the leaves. The xylem cells of the stem, roots, and leaves are interconnected forming a conducting channel reaching all plant parts. The cohension-tension mechanism, maintains the flow of sap in the xylem tissues. 92. (c) Competition between the individuals of the same species is called Intraspecific struggle. Intraspecific struggle takes place due to the requirement of same Practice Set 1

species like food, shelter, breeding spaces etc., are similar. While interspecific struggle takes place between individuals of different species. 93. (b) Division of cytoplasm after completion of nuclear division is called cytokinesis. Cytokinesis is the physical process of cell division that divides the cytoplasm of a parent cell into two daughter cell produced either via mitosis or meiosis. This is also often known as cytoplasmic division or cell cleavage. Cytokinesis occurs in the late telophase of mitosis in an animal cell. 94. (b) The dark reaction of photosynthesis takes place in the stroma of the chloroplasts as the enzymes of stroma do not require light but require ATP and NADPH2 produced in the light reaction. It is also known as light independent reaction. 95. (c) Zn is not a component of nitrogenase complex. This enzyme is two component system comprising an iron protein and molybdnum- iron (MoFe) Protein, each containing FeS cluster which are responsible for the electron flow. 96. (a) Cell organelles can be physically separated of each other by centrifugation. This technique involves the application of centrifugal force to separate particles from a solution. In a centrifuge, the particles sediment at different rates when an accelerating force is subjected. The rate of sedimentation depends upon the size of the particles, its shape and density. 97. (d) C3, C4 and CAM plants are found in Euphorbiaceae family. Euphorbiaceae is spurge family of flowering plants, containing some 6745 species in 218 genera. The key difference between C3, C4 and CAM plants is the synthesis of different products during the grasping of CO2 for photosynthesis from the sunlight and then conversion of it to glucose. 98. (b) Karyotype refers to phenotypic appearance of chromosome set. ⇒ A karyotype is an individual's collection of chromosomes. The term also refers to a laboratory technique that produces an image of an individual's chromosomes. The karyotype is used to look for abnormal numbers or structures of chromosomes. 99. (b) The Transitional zone between two distinct communities is known as Ecotone. 100. (d) Glutamine synthetase plays an important role in ammonia assimilation. The reaction facilitated by following :

Glutamine Glutamate + NH3  Glutamine + H 2 O  Synthetase  39

YCT

PRACTICE SET - 2 (c) f1 and f2 have opposite sign and the lenses are Two vectors A and B lie in a plane, another made up of same glass vector C lies outside this plane, then the resultant of these three vectors i.e., A+B+C : (d) f1 and f2 have opposite sign and the lenses are made up of different glasses (a) can be zero 11. Two lenses of powers +4D and –2D are kept in (b) cannot be zero contact. The combinations focal length will be (c) lies in the plane containing (A+B) (a) 100cm (b) 75cm (c) 50cm (d) 25cm (d) lies in the plane containing (A–B) 2. The peak value of the a.c. voltage across the 12. The radius of curvature of either surface of an secondary of the transformer in a half wave equiconvex lens is 'R'. If the refractive index of its material is 1.5 with respect to medium in rectifier with no filter circuit is 9 2 V. The which it is placed, its focal length is; maximum d.c. voltage across the load is: (b) R/4 (c) R (d) 2R (a) R/2 (a) 4 V (b) 8 V 13. In any two media the reversibility of light rays (c) 1.4 V (d) 9 V obeys : 3. In given circuit, if source deliver maximum (a) 1 n 2 ×2 n1 = 1 (b) 1 n 2 = 2 n1 power to load, then load impedance must be (c) (1 n 2 ) = (2 n1 )2 (d) None of these 14. A person of height 2 m stands in front of a plane mirror. What must be the minimum vertical height of the mirror so he can see his full image? (a) 2 m (b) 1 m (c) 0.5 m (d) 0.25 m (a) 120 + 40j (b) 120 – 40j 15. The shape of the Lissajous Figures depends on (c) 40 + 120j (d) 110Ω (a) The amplitude of the superposing waves 4. Ampere's Circuital law is− (b) Frequencies of superposing waves (c) The phase difference between the superposing (a) div B = 0 (b) curl B = 0 waves (c) curl B = µ 0 I (d) φBd l = µ0 I (d) All of the above An electron is injected into a region of uniform 16. For diffraction at a single slit, the width of the 5. magnetic flux density with components of central maximum is____ the wavelength of light velocity parallel to and normal to the flux. The (a) Proportional to path of the electron shall be (b) Proportional to the square of (a) a helix (b) a parabola (c) Inversely proportional (c) a straight line (d) a circle (d) Independent of 6. If a negligible small current is passed through a 17. A tuning fork of frequency 500 Hz is vibrated wire of length 15 m and of resistance 5Ω having with a sonometer wire and 8 beats per second uniform cross-section of 6 ×10–7 m2. Then are heard. The beat frequency reduces if coefficient of resistivity of the material is tension in the wire is slightly increased. The original frequency of the wire is (a) 1 × 10–7Ω –m (b) 2 × 10–7Ω –m –7 –7 (a) 516 Hz (b) 508 Hz (c) 3 × 10 Ω –m (d) 4 × 10 Ω –m (c) 500 Hz (d) 492 Hz 7. In Millikan's oil drop experiment : (A) Observations are taken on drops moving with 18. In a waveguide which condition will always hold good? uniform velocity (a) Phase velocity = c (b) Phase velocity > c (B) Velocity of drops becomes uniform due to (c) Phase velocity < c (d) Group velocity = c equality of gravitational force and force due to 19. For formation of standing waves, the medium electric field should (C) Possible values for charges on drops are (a) have infinite extension 6.4 × 10–19C, 7.2 × 10–19C, 12.8 × 10–19 C. (b) be bounded Out of the above, true statements are only : (c) be a gas only (a) A (b) B and C (c) C (d) A and C (d) be having zero inertia and zero elasticity 8. The method of partial wave is suitable for 20. The coordinates of the displacement of a (a) low energy scattering particle are x = a sin ωt and y = bcosωt .Path (b) medium-high energy scattering of the particle is (c) high energy scattering (a) Linear (b) Circular (d) for all energy scattering (c) Elliptical (d) None of the above 9. When a thin transparent plate is introduced in 21. Critical velocity of a liquid may not possible be the path of interfering beams, the fringe width will related to: (a) increase (b) decrease (a) radius of tube (b) density of liquid (c) become zero (d) remains the same (c) coefficient of viscosity (d) mass of liquid 10. A useful achromatic combination of two lenses of 22. There is no atmosphere on the moon because : focal lengths f1 and f2 can be made if: (a) it is closer to the earth (a) f1 and f2 have same sign and the lenses are made up of same glass (b) it revolves round the earth (b) f1 and f2 have same sign and the lenses are (c) it gets light from the sun made up of different glasses (d) none of the above 1.

Practice Set 2

40

YCT

23.

24.

25.

26.

27.

28.

29.

30.

31.

32.

The distance of two planets of the sun are 1013 and 1012 m respectively. Ratio of time periods of these planets is : (a) 1/ 10 (b) 100 (c) 10 10 (d) 10 The weight of a certain body on the surface of earth is 90 kg-wt. If the mass of the mars is 1/9 times of the earth and its radius be half that of earth what will be the weight of the body on the mars : (a) 10 kg-wt (b) 40 kg-wt (c) 50 kg-wt (d) 90 kg-wt Which of the following can NOT be obtained from rotational levels? (a) Moment of inertia (b) Force constant (c) Internuclear distance (d) Rotational frequencies The correct relation is(a) L = r × p (b) L = r .p (c) L = r × F (d) L = r . F If a light and a heavy body have equal kinetic energy, which one has a greater momentum? (a) The light body (b) The heavy body (c) Both have equal momentum (d) Nothing can be said The lift is moving down with an acceleration a. A man in the lift drops a ball inside the lift. The acceleration of the ball as observed by the man in the lift and a man standing stationary on the ground are, respectively. (a) g, g (b) g – a, g – a (c) g – a, g (d) a, g A man is running on a 100 m straight track. The number of degrees of freedom associated with the motion of the man is (a) 1 (b) 2 (c) 3 (d) 0 Material having low retentivity is suitable for making (a) Weak magnets (b) Temporary magnets (c) Permanent magnets (d) Electro magnets Which of the following is the conjugate acid of NH3? (a) NH −2 (b) NH 2 OH (c) NH +4 (d) N 2 H 4 Which one of the following relations is correct for degree of hydrolysis of CH3COONH4? (a) h =

Kh Ka Kb

(b) h =

35.

36.

37.

38.

39.

40. 41. 42.

43.

44.

Ka Kb Kh

Kh Ka (d) h = c Kb Which of the following satisfy the Gay-Lussac's law of constant volume ? 45. (a) S2 + O 2(g ) → SO 2(g ) (b) CH 3 COOH ( l ) + C 2 H 5 OH ( 1) → CH 3COOC 2 H 5(1) + H 2 O ( 1) 46. (c) 2NH 3(s) → N 2(g ) + 3H 2(g ) (d) CaCO 3(s ) → CaO (s) + CO 2(g ) The values of Vander waals constant 'a' for gases N2, O2, NH3 and CH4 are 1.39, 1.36, 4.00 and 2.253 dm6 atm mol–2 respectively. The gas which can be most easily liquified is (a) N2 (b) O2 (c) NH3 (d) CH4 (c) h =

33.

34.

Practice Set 2

41

Which of the following pairs of Solutions is isotonic pair at the same temperature? (a) 0.1 M glucose and 0.1 M KCl (b) 0.1 M glucose and 0.1 M MgCl2 (c) 0.1 M KCl and 0.1 M K2SO4 (d) 0.1 M K2SO4 and 0.1 M Cd (NO3)2 On adding few drops of dilute HCl to freshly Precipitated ferric hydroxide a red coloured colloidal solution is obtained due to : (a) Dissolution (b) Dialysiss (c) Peptization (d) Congulation Which one of the following is responsible for lesser pressure of a real gas in comparison to an ideal gas? (a) Finite size of molecules (b) Enhancement of kinetic energy of molecules (c) Viscosity (d) Intermolecular forces For a given reaction of first order, it takes 20 minutes for the concentration to drop from 1.0 M to 0.6 M. The time required for concentration to drop from 0.6 M to 0.36 M will be: (a) More than 20 minutes (b) Less than 20 minutes (c) Equal to 20 minutes (d) Infinity Which one of the following is correctly matched : (a) Phase rule - Irving Langmuir (b) Chemical equilibrium - Le Chatelier (c) Entropy - J.J. Thomson (d) Photoelectric effect - Ostwald In the co-ordination compound K4[Ni(CN)4], what is the oxidation state of Nickel? (a) –1 (b) 0 (c) +1 (d) +2 Argentite s an ore of which of the following? (a) Au (b) Ag (c) Pt (d) Cu The rate of a reaction was found to be equal to the rate constant at any concentration of the reactant. The order of the reaction is: (a) 1 (b) 2 (c) 1.5 (d) zero Why usually covalent compouds have low melting and boiling points? (a) Strong Vander Waal's force (b) Weak Vander Waal's force (c) No Vander Waal's force (d) Strong magnetic force Consider the following statements and select your answer from the codes given below: Answer (A) : Lyophilic colloids are more stable than lyophobic colloids. Reasons (R) : Lyophobic Colloids extensively solvated. Codes: (a) Both A and R are correct and R is the correct explanation of A. (b) Both A and R are correct but R is not correct explanation of A. (c) A is correct but R is wrong. (d) A is wrong but R is correct. The maximum number of electrons, that can be accommodated in an orbital is : (a) 2 (b) 6 (c) 10 (d) 14 Some postulates of atomic theory of matter are quoted below. One of the above postulates was not part of Dalton's atomic theory. Identify it. (a) The ultimate particles of a given pure substance, called atoms, are chemically alike. (b) Atoms can neither be created nor destroyed. (c) Atoms though chemically identical, differ slightly in their weights. (d) Chemical reactions do not change nature of amtoms, but cause their rearramgement. YCT

47.

48.

The primary amines can be distinguished from 56. secondary and tertiary amines by– (a) Liebermann's nitroso reaction (b) Carbylamine reaction (c) Reaction with Grignard reagent (d) Action of sodium on them Which one of the following will NOT give Diels- 57. Alder reaction? (a)

(b)

58. (c)

(d)

59. 49.

50.

51.

Which one of the following compounds will not give yellow precipitate with I2 and NaOH ? 60. (a) Ethanol (b) Acetone (c) Diethylketone (d) Acetophenone Which of the following will not react with hydroxyl ions by nucleophilic attack ? 61. (a) Diethyl ether (b) Ethyl acetate (c) Acetamide (d) Acetonitrile Which one of the following is the major product of the bromination of m-nitrobenzene sulphonic acid? 62.

63.

64.

65.

52.

53.

54.

55.

The treatment of an ester with LiAlH4 by acid hydrolysis produces (a) two acids (b) two alcohols (c) two aldehydes (d) one molecule of alcohol and another of carboxylic acid Using urotropine, which of the following compounds gives benzaldehyde? (a) Toluene (b) Benzyl chloride (c) Benzal chloride (d) Chlorobenzene C6H5CONH2 + P2O5  ∆→ Product. The product of the above reaction is (a) C6H5NH2 (b) C6H5CN (c) C6H5CH2NH2 (d) C6H5NC Which one of the following will undergo only E1 reaction with KOH ? (a) CH3-CH2-CH2- CH2-Br (b) (CH3)2-CH2-CH2-Br (c) (CH3)3-C-CH2-Br (d) CH3-CH2-CHBr-CH3

Practice Set 2

66.

67.

68.

69.

42

The process of converting alkyl halides to alcohols involve(a) Addition reaction (b) Rearrangement reaction (c) Substitution reaction (d) Dehydrohalogenation reaction Which of the following electronic configuration obeys Hund's rule? (a) 1s2 2s2 2px2 2py1 2pz1 (b) 1s22s22px22py22pz0 (c) 1s22s22px22py12pz0 (d) 1s22s22px22py32pz1 H2O2 can act in different ways, out of the following select which is not true for H2O2? (a) Reducing agent (b) Oxidising agent (c) Dehydrating agent (d) Bleaching agent An aqueous solution of copper sulphate is– (a) Acidic (b) Neutral (c) Basic (d) Amphoteric Metal hydroxides are insoluble and used as better Antacids. Because (a) pH7 (c) pH = 7 (d) pOH = 9 A prokaryotic cell lacks ? (a) only histones in its DNA (b) only cytoskeleton (c) only sub-cellular organelles (d) all of the above Golgi complex is found in : (a) All the cells (b) All the cells except RBC (c) All the except prokaryotic cells RBCs (d) All the animal cells and bacterial cells Nucleolus is the site for synthesis of : (a) Protein (b) Messenger RNA (c) Transfer RNA (d) Ribosomal RNA General life cycle has the sequence : (a) 1N → Meiosis → 2N → Fertilization → 1N (b) 2N → Meiosis → 1N → Fertilization → 2N (c) 1N → Mitosis → 2N → Fertilization → 1N (d) 2N → Mitosis → 1N → Fertilization → 2N Which one of the following is a free living nitrogen fixing bacteria? (a) Azotobactor (b) Rhizobium (c) Streptococcus (d) Clostridium An example of a living fossil is a species of : (a) Archaeopteryx (b) Uromastyx (c) Latimeria (d) Columba The phenomenon of Industrial melanism demonstrates – (a) Induced mutation (b) Reproductive isolation (c) Natural selection (d) Geographic isolation Which one of the following gases was not present in the free state when life orginated on earth : (a) H2 (b) NH3 (c) CH4 (d) O2 Theory of acquired characters in living being are given by : (a) Mendel (b) Darwin (c) Lamarck (d) Linnaeus YCT

70.

71.

72. 73.

74.

75.

76.

77.

78.

79.

80.

81.

82. 83.

84.

Natural selection means : (a) Better adaptability (b) Better survival (c) Elimination of less adapted (d) All of the above Rennin is found in the : (a) stomach of infants (b) stomach of adults (c) pancreatic juice (d) saliva For a healthy man, the tidal volume of lungs is: (a) 200 ml (b) 650 ml (c) 500 ml (d) 1100 ml If human blood mixed with an anticoagulant is centrifuged, its various components (starting from the bottom of the centrifuge tube) will separate in which of the following sequences? (a) Red cells → white cells → platelets → plasma. (b) Plasma → red cells → platelets → white cells. (c) Platelets → plasma → white cells → red cells (d) White cells → red cells → platelets → plasma. Glucose reabsorption occurs in the : (a) Proximal tubule (b) Loop of Henle (c) Distal tubule (d) Medullary collecting duct Aqueous humor is secreted by : (a) choroids (b) retina (c) ciliary body (d) lacrymal gland In a food web vultures act as : (a) primary consumer (b) primary producer (c) scavenger (d) decomposer The two gases making the highest relative contribution to the greenhouse gases, are(a) CO2 and CH4 (b) CH4 and N2O (c) CFC and N2O (d) CO2 and N2O 'Nokrek' Biosphere Reserve is located in(a) Arunachal Pradesh (b) Assam (c) Meghalaya (d) Tripura PAN is: (a) Peroxyamyl nitrate (b) Polyoxyamyl nitrate (c) Peroxyacetyl nitrate (d) Polyoxyacetyl nitrate A method of birth control is : (a) GIFT (b) HIF (c) IVF-ET (d) IUDS An example of enucleated cell in plants is : (a) Parenchyma cell (b) Meristematic cell (c) Tracheid (d) A mature sieve tube cell of phloem The main component of mangroves vegetation: (a) Ficus (b) Rhizophora (c) Mangifera (d) Prosopis Tropical plants like sugarcane show high efficiency of CO2 fixation because of : (a) Hatch and Slack cycle (b) CAM pathway (c) Calvin-Benson cycle (d) EMP pathway Lomasome represents : (a) Membrane structure (b) Mitochondrial equivalent (c) Plastid (d) lrregular structures

Practice Set 2

85.

Bio-fertilizers are (a) dead remains of plants and animals (b) decomposed animal excreta (c) living cells of nitrogen fixing organisms (d) organic chemical 86. Which one of the following is called the brain of the cell : (a) Ribosomes (b) Nucleus (c) Plasma membrane (d) Mitochondria 87. CAM plants at night produce : (a) PEP (b) Pyruvic acid (c) Malic acid (d) Citric acid 88. Soil water, absorbed by plants is called : (a) Chresard (b) Hollard (c) Echard (d) Fullard 89. Which of the following cell organelle possesses a single unit membrane? (a) Chloroplast (b) Lysosome (c) Mitochondria (d) Nucleus 90. The major greenhouse gas is : (a) Freon (b) CO2 (c) CF2 (d) CO 91. Embryologically, Which family has been separated from onagraceae : (a) Lecythidaceae (b) Myrtaceae (c) Trapaceae (d) Barringtoniaceae 92. During centrifugation which cell organelle will sediment at lower speed? (a) Mitochondria (b) Nuclei (c) Golgi complex (d) Ribosome 93. Ecosystem has two components which are : (a) Plants and animals (b) Weeds and trees (c) Biotic and abiotic (d) None of the above 94. Which of the following glycosidic bonds is found in sucrose? (a) α 1-4 (b) β 1-4 (c) α 1-6 (d) α-1, β -2 95. Absence of sexual union and failure of meiosis is called : (a) anisospory (b) Amphimixis (c) apomixis (d) anisogamety 96. Organisms which usually live as parasites but can grow saprophytically in the absence of their host plant are called : (a) Obligate parasites (b) Facultative Parasites (c) Obligate saprophytes (d) Facultative saprophytes 97. Fucoxanthin pigment is found in : (a) Blue-green algae (b) Green algae (c) Red algae (d) Brown algae 98. The size of ribosomes in Archaea is : (a) 80s (b) 70s (c) 70s and 80s both (d) none of the above 99. The common mode of action of herbicides is the blocking of : (a) Photosystem I (b) Photosystem II (c) Xylem (d) Pholem 100. An example of saprophytic orchid is (a) Vanda (b) Orchis (c) Neottia (d) Oberonia

43

YCT

SOLUTION : PRACTICE SET- 2 ANSWER KEY 1.(b) 11.(c) 21.(d) 31.(c) 41.(b) 51.(b) 61.(d) 71.(a) 81.(d) 91.(c)

2. (d) 12.(c) 22.(d) 32.(a) 42.(d) 52.(b) 62.(c) 72.(c) 82.(b) 92.(b)

3.(b) 13.(a) 23.(c) 33.(b) 43.(b) 53.(b) 63.(d) 73.(a) 83.(a) 93.(c)

4. (d) 14.(b) 24.(b) 34.(c) 44.(c) 54.(b) 64.(b) 74.(a) 84.(a) 94.(d)

5.(a) 15.(d) 25.(b) 35.(d) 45.(a) 55.(b) 65.(a) 75.(c) 85.(c) 95.(c)

6.(b) 16.(a) 26.(a) 36.(c) 46.(c) 56.(c) 66.(c) 76.(c) 86.(b) 96.(d)

7.(b) 17.(d) 27.(b) 37.(d) 47.(b) 57.(a) 67.(c) 77.(a) 87.(c) 97.(d)

8.(a) 18.(b) 28.(c) 38.(b) 48.(b) 58.(c) 68. (d) 78.(c) 88.(a) 98.(b)

9.(d) 19.(b) 29.(a) 39.(b) 49.(c) 59.(a) 69.(c) 79.(c) 89.(b) 99.(b)

10.(d) 20.(c) 30.(b) 40.(b) 50.(a) 60.(b) 70.(d) 80.(d) 90.(b) 100.(c)

SOLUTION 8.(a) 1. (b) Partial wave is suitable for low energy scattering. Two vectors A and B lie in a plane, another vector C ur ur ur At low energies (i.e. when is much larger than the range lies outside this plane therefore A + B+C ≠ 0 Since the third vector has a component which lies of potential) partial waves with, in general, make a outside the plane of the remaining two, hence this extra negligible contribution to the scattering cross section. It component cannot be cancelled by any other follows that at these energies, with a finite range potential only wave scattering is important component during addition, so sum can never be zero. r r 9.(d) Also, the sum lies outside r ther plane of A + B (which is The expression for the fringe width in young's double the same plane as A and B ) because of this extra slit experiment is component of C. λD 2. (d) β= …(i) d VO 9 2 Vac = = = 9 Volt Where, 2 2 β is the fringe width 3.(b) λ is the wavelength Maximum power to load that mean if the source d is the distance between the slits impedance is (R + jX)Ω to have maximum power D is the distance between the slit and screen. transfer, the load impedance must be (R – jX) Ω From equation (i) it can be concluded that the fringe width ZS = 120 + 40j depends on the wavelength (λ) of light, distance d between So, that maximum power transfer the slits and the distance between the slit and screen. The Load impedance must be = 120 – 40j. When a thin transparent sheet is placed in front of the 4. (d) double slit. The distance between the slits and screen Ampere's circuital law, remains same. Since sheet is transparent the light will pass through the sheet and hence wavelength of light Bdl = µ 0 I will also remain unchanged. Hence all the quantities in the given expression remains 5.(a) same. The fringe width will also remains the same. Since a charged particle moving parallel to magnetic 10. (d) field then path is straight line. A useful achromatic combination of two lenses of focal when a charged particle moves perpendicular to magnetic lengths f and f can be made if f and f have opposite 2 1 2 field then the path is circular and in case of having sign and 1the lenses are made up of different glasses components in both direction the path will not be parallel 11.(c) nor perpendicular. The path will become helix. The combinations focal length will be, 6.(b) P = P1 + P2 The Coefficient of resistivity of the material is

∫

P = 4 − 2 = 2D 100 100 f= = = 50 cm P 2 12.(c) 7.(b) from the formula, In Millikan's oil drop experiment, both the forces gravitational force and force due to electric field becomes 1 = (µ − 1)  1 − 1    zero. So the velocity of drops remains uniform. f  R1 R 2  RA 5 × 6 × 10−7 = l 15 ρ = 2 × 10 −7 Ω − m

ρ=

Practice Set 2

44

YCT

1 1 1 = (1.5 − 1)  +  f R R 1 2 = 0.5 × f R f =R 13.(a) Principle of reversibility : The principle of reversibility states that light follows the same path if the direction of light is reversed. The refractive index of light when it passes from the first medium to the second medium is same as the refractive index of light when it goes from second medium to the first medium. Principle of reversibility follows both reflection and refraction.

always greater than the speed of light. This implies the group velocity is small.

19. (b) For the formation of stationary waves, it is necessary that the medium should not be unlimited but it should have boundary. The wave propagating in such a medium will reflect at the boundary and produce a wave of the same kind travelling in the opposite direction. The position of two waves will give rise to a stationary wave. At free end, transverse wave is reflected without change of phase. Hence as essential requirement for the formation of stationary wave is that the medium must be bounded having definite boundaries. 20.(c) 1 2 Standard equation of ellipse is given by, So, that n 2 × n1 = 1 x 2 y2 14.(b) + = 1.......(i) In order to see full image of person, the minimum size a 2 b2 of the mirror should be one half the person's height. It Putting x = a sin ωt, y = b cos ωt in equation (i). does not matter how far you are from the mirror. So to 2 ( a sin ωt ) ( b cos ωt ) 2 see the image of a person of height 2m, a 1 m long + =1 mirror is required. a2 b2 15.(d) sin 2 ω t + cos 2 ωt = 1 The Lissajous figures is used to measure frequency and phase difference of a system. It is an elliptical figure So path of particle is elliptical. and follows following set of equation is given as 21.(d) (1) η = A sin (at + δ) kη from formula v = (2) y = B sin (bt) ρr From above two equations it can be concluded that the v = Critical velocity k = Reynold's number Lissajous figures depends on Amplitude, frequency, and phase difference of superposing waves. η = viscosity coefficient ρ=density of the liquid 16.(a) r = radius of the tube For diffraction at a single silt, Hence, the critical velocity of a liquid is not related to The angular distance between the two first order the mass of the liquid. minima is called the angular width of central maximum, 22.(d) given by– There is no atmosphere at moon because escape 2λ velocity is less than the root mean square velocity of 2θ = a the molecules at moon. Hence all molecules escape. The linear width is as follows 23.(c) By kepler's third law, 2Lλ w = L.2θ = a T 2 ∝ r 3 where, r = R e + h So the width of the central maximum in the diffractional 3 formula is directly proportional to the wavelength of light. T 2  R e + h1  1  ⇒ = 17.(d) T 2  R e + h2  v2 could be 508 Hz or 492 Hz for 8 beats/second. If the 2 tension of the sonometers wire is slightly increased, v2 3 increases, but the beat reduces. Therefore, the original T 2  6.4 × 106 + 1013  1 frequency of the string is 492 Hz  ⇒ = v2 T 2  6.4 × 106 + 1012  2 T1 ⇒ = 10 10 T v1 2 24.(b) Weight on the surface of earth W = mg = 90 kg-wt Acceleration due to gravity GM e g= 18.(b) R e2 In air medium the phase velocity is assumed to be the value of g on mars speed of light. For waveguides, the phase velocity is Practice Set 2

45

YCT

1 G × Me 4 GM e 4 9 = = = g 2 9 9 R e2  Re   2   therefore, weight on mars =

4 × 90 = 40kg.wt 9

to given conjugate base NH3 . Therefore NH +4 is the conjugate acid of ammonia. NH3 + H+  NH +4 Base Conjugate acid 32.(a) Degree of hydrolysis of CH3COONH4

25.(b) Kh The rotational energy levels for a nonlinear molecule h= are a function of moment of inertia, internuclear Ka Kb distance (rotating particle's range of attraction) and 33.(b) rotational frequencies. CH3COOH(l ) + C2H5OH(l ) → CH3COOC2H5 (l ) + H2O( l ) 26.(a) r r r Gay-Lussac's law of constant volume was proposed by L = r×P Gay-Lussac in the year (1808) . According to this law where L is angular momentum, P is linear momentum, gases react with each other chemically so their reactant r is the perpendicular distance. interact with each other in volume and form in the 27.(b) reaction the volume of the gaseous product have a P2 simple ratio provided that all, volume are measured at Relation KE = [P = MV] the same temperature and pressure. 2M 34.(c) ⇒ P = 2MKE The value of vander waals constant 'a' represents the P M force of attraction between the gaseous molecules thus ⇒ = ⇒ M > M ′ ⇒ P > P′ P′ M′ higher the value of 'a' greater will be the attraction Momentum is dependent upon mass. Hence the heavy between the molecules of the gas, therefore it the value body has a greater momentum. of 'a' is high it would be easily liquified. According to the question, the value of 'a' for N2, O2, 28.(c) NH3 and CH4 respectively 1.39, 1.36, 4.00 and 2.53 are Case I–The acceleration seen by person inside the lift given. Since, for NH3 the value of 'a' is highest (4.00) will be the difference between the acceleration of the hence, NH3 gas is easily liquefied. person and the ball. 35.(d) a lg = a b − a m1 (both are in motion) In the same concentration of solution of different compounds having the same number of moles, such a lg = g − a solution is called isotonic solution. alg = acceleration of lift with respect to ground Case II– When the person outside will see the K 2SO4  2k + + SO−4 − acceleration of the ball, he will see both the lift and the 3 mole ball. Cd(NO3 )2  Cd ++ + 2NO−2 Therefore a b = g 3 mole 29.(a) So, 0.1M K SO and 0.1M Cd(NO3)2 are isothermally 2 4 A man running on a 100 m straight track has a degree of freedom 1. His motion is restrained in other direction coupled at the same temperature. 36.(c) since he is running in forward direction only. Peptization is the process responsible for the formation 30.(b) Materials of low retentivity are used to make temporary of stable dispersion of colloidal particles in dispersion magnets. Iron and cobalt are the two material that you medium so that the reason of Peptization. can use for making the temporary magnet these The addition of dilute hydrochloric acid to freshly ferric materials will be behaving in the form of magnet only hydroxide to get red coloured colloidal slution is peptization. The precipitate is broken with the help of when they are held near the strong magnet. Retentivity of material is its capacity to remain electrolyte and is dispersed into colloidal medium to magnetized even after external magnetizing field has form stable colloids. removed. Therefore, material used in temporary 37.(d) magnets should have low retentivity, so that it doesn't Due to Intermolecular force the pressure of real gas get magnetized or gets demagnetized easily. becomes less than that of ideal gas. The force of 31.(c) attraction or repulsion between the particles of a In the Bronsted - Lowry system an acid react by substance is called intermolecular force. donating a proton to a base. In doing so the acid 38.(b) becomes its conjugate base, the formula of the Equal time in first order reaction the percentage will conjugate acid is the formula of the base plus one decay. hydrogen ion. Therefore, Ammonia or NH3 is a base. It accept a proton to give 0.6 in 20 minute × 100 = 60% its conugate acid NH +4 . Similarly, NH +4 loses a proton 1 Practice Set 2

46

YCT

having offensive smell. carbylamine reaction.

0.36 × 100 = 60% in 20 minute 0.6 because at the concentration depend on the first order of reaction. 39.(b) ∴

Phase rule Josiah Willard Gibbs Chemical equilibrium Le Chatelier Entropy Rudolf Clausius Photoelectric effect Albert Einstein * Here, chemical equilibrium defined by Le Chatelier is correctly mathched. 40.(b) K4[Ni(CN)4] Oxidation state of Ni = +1 (4) + x + 4 (–1) = 0 4+x–4=0 x=4–4=x=0 41.(b) Argentite is ore of (Ag), and the general formula is Ag2S. 42.(d) For the zero order reaction rate of reaction does not depend on concentration of reactant. So the rate constant for zero order reaction not affected at any concentration of the reactant. 43.(b) Covalent compouds have low melting and boiling points because they are composed up electrically neutral molecules and thus very weak vander Waal's forces between molecules. 44.(c) Lyophilic colloids are more stable than lyophobic colloids due to Lyophilic colloids are extremly solvated than lyophobic so statement (A) is Correct but reason (R) is wrong. 45.(a) According to the pauli exclusion principle, a single orbital can have a maximum of two electrons with opposite spins. That is, no two electrons in an atom can have the same value of all four, quantum number n, l, m, s. So it is clear that 'maximum' 2 electrons can be accommodated in an orbital. 46.(c) From the given postulates - Atoms being chemically similar, yet they may have slight difference in their weights. It is part of Dalton's atomic theory of matter. According to Dalton's atomic theory of matter. 1. Matter is made up of tiny indivisible particles or (atoms) which can not be created or destroyed. 2. All atoms of an element are identical mass and physical and chemical properties are the same. 3. Atoms of different elements have different physical and chemical properties and mass. 4. Atoms of different elements by combining simple whole numbers makes compounds. 5. Atoms of an elements cannot be changed. In atoms of other elements, reorganization of chemical reactions is involved. 47.(b) Carbylamine reaction -: When a primary amine (aliphatic or aromatic) is warmed with chloroform and alcoholic KOH. It forms an isocyanide or carbylamine

Practice Set 2

The reaction is called +

Θ

CHCl3 / alc KOH R − NH 2 → R − N ≡ C+ 3KCl + 3H 2 O Carbylamine reaction is not primary and secondary amine.

 → CH CH NC + 3KCl + 3H O CH3 − CH 2 − NH2 + CHCl3 + KOH  3 2 2

Ethyl amine Ethylisocynide. Since secondary and tertiary amines (aliphatic or aromatic) do not give this reaction. It used as a test for primary amines and also for the distinction of primary amine from secondary and tertiary amines. 48.(b) The [4+2]- cycloaddition of a conjugate alkyne), an elecrocyclic reaction that involves the 4π electron of the diene and 2π electron of the dienophile. The driving force of the formation of new σ-bonds, which are energetically more stable than the π bond. 4+2 cyclo addition is not possible in option 'B' so this compound do not undergo diels alder reaction. 49.(c) Carbonyl compound which are present in the carbonchain (CH3CO-) group is present they give iodoform test of compound I2 and NaOH. In which yellow of iodoform precipitation is obtained. So diethyl ketone. O || C2 H5 − C − C2 H5 no CH3CO- group is present it does not give iodoform test and will not give yellow precipitate with I2 and NaOH. 50.(a) Acetamide, acetonitrile, and methyl acetate contains an unsaturated carbon atom which can be easily attacked by nucleophile such as hydroxyl ion. In diethyl ether, all carbon atoms are saturated. Hence, diethylether it resists attack by hydroxyl ion. 51.(b)

As here both group (SO3H, NO2) are deactivating group so they direct electrophile attack to meta position. 52.(b) The treatment of an open chain ester with LiAlH4 followed by acid hydrolysis produces two alcohols. LiAlH

4 → CH CH OH + CH OH CH3 COOCH 3 + 4  H +   3 2 3

methyl acetate

methanol

ethanol

53.(b) Benzyl chloride using urotropene forms benzaldehyde. this reaction is called sommelet reaction. Formaldehyde and conc. ammonia react to form hexamethylene tetramine which is called hexamine or urotropine. Urotropine is a solid compound, it is used as medicine in rheumatism and urinary disease. 47

YCT

O  → ( CH 2 )6 N 4 +5H 2 O 6H–C–H+4NH 3  Formaldehyde

Hexamethylene tetramine

54.(b)

Product → C6H5–CN 55.(b) (CH3)2-CH2-CH2-Br having linear structure and has E1 reaction with Alcoholic KOH. 56.(c) The process of converting alkyl halides into alcohols involves in substitution reaction. R − X + KOH → R − OH + K − X

(Alkyl halide) (Aq.) (Alcohol) Since X– is replaced by OH– group, it is a nucleophilic substitution reaction. 57.(a) According to Hund's rule all orbitals will be singly occupied before any is doubly occupied and all unpaired electrons must have same spin. The electronic configuration of elementis– 1s2, 2s2, 2px2, 2py1, 2pz1 58.(c) H2O2 is a pale blue liquid and it can be oxidised by ozone. H2O2 acts as both oxidizing and reducing agents. the value of dipole moment of H2O2 is 2.1D which suggest it cannot be planar infact it has open book like structure. Dehydrating agent is not work of H2O2 . 59.(a) Aqueous solution of copper sulphate is acidic because it is soluble in water it dissolves copper hydroxide in water Cu(OH)2, hydroxides Cu(OH) 2 and sulphuric acid (H2SO4) form of acid and removes H+ ions from the soltion. CuSO 4 + 2H 2 O → Cu(OH) 2 + H 2SO 4 60.(b) Hydroxides of metals are strong bases. Its value is greater than 7 (pH > 7) , Mg(OH) 2 in water is insoluble it is also called milk of magnesia it's used as an antacid to relives acidity. 61.(d) A prokaryotic cell lacks sub-cellular organelles, cytoskeleton, and histones in its DNA. The absence of all these structures differentiates prokaryotes from eukaryotes. Most prokaryotes carry a small amount of genetic material in the form of a single molecule, or chromosome, of circular DNA. Prokaryotic cells are surrounded by a plasma membrane, but they have no internal membrane bound organelles. 62.(c) Golgi complex is found in all the cells except prokaryotic cells and RBCs. It is also known as golgi body or golgi apparatus. It is a membrane bound organelle, involved in the processing of proteins Practice Set 2

usually, each cell has one golgi body, but some may have more than one. It is situated near the nucleus and consist of 5-8 flattened membranous sac called the cisternae. 63.(d) Nucleolus was discovered by Fontana (1781) and given name by Bowman (1840). It does not have membrane and is attached to chromatin at nucleolar organiser region (NOR). Nucleolus is the site for elaboration of rRNA and synthesis of ribosomes, hence called ribosomal factory. 64.(b) In eukaryotic organisms, gamete, mother cells are diploid (2N) and have two complete sets of chromosomes. Meiosis in male and female gamete mother cells form haploid male and female gametes (N) respectively. This occurs since meiosis reduces the number of the chromosome to half in the daughter cells. The fusion of haploid male and female gametes during fertilization restores the diploid chromosome number of the species and forms diploid zygote (2N). Repeated mitotic divisions in the diploid zygote form the diploid organism. 65.(a) Azotobacter is free-living nitrogen fixing bacteria. It is a gram negative, aerobic soil dwelling oval or spherical bacteria that from thick walled cysts. Reduction of inert atmospheric nitrogen to ammonia by living organisms is called biological nitrogen fixation. There is unique system of three distinct nitrogenase enzymes makes theses bacteria of particular interest in understanding nitrogen fixation. Azotobacter have the highest metabolic rate of any organisms. 66.(c) Latimeria is an example of a living fossil species. Archaeopteryx is a connecting link between birds and reptiles. Uromastyx are commonly called spiny-tailed lizards. Columba comprises a group of medium to large pigeons. 67.(c) The phenomenon of Industrial melanism demonstrates natural selection, Industrial melanism is a term that describes how some animals change color in response to environmental charges caused by pollution. The term was coined just after the Industrial Revolution when coal was used to power factories in cities such as London and New York. This phenomenon was discovered in 1900 by geneticist William Bateson, The increase in dark-colored math is an example of industrial melanism. 68. (d) Early atmosphere of earth, when free atoms combined to form molecules and compounds of H2, N2, NH3, CH4 and H2O. But there was no free oxygen. According to Oparin, the primitive atmosphere of the earth consisted of numerous hydrogen atoms that combined with all oxygen atoms to form water and leaving no free oxygen.

48

YCT

69.(c) Theory of acquired characters in living being was given by Lamarck. Inheritance of acquired characteristics are those changes in the structure or function of an organism that are the result of use, disuse, disease, environmental influences. 70.(d) Natural selection means better adaptability, better survival and elimination of less adapted. It is a mechanism of evolution. Organisms that are more adapted to their environment are more likely to survive and pass on the genes that aided their success. This process causes species to change and diverge over time. 71.(a) Rennin is found in the stomach of children. It is important that rennin is not secreted in adult cow and human. Rennin is secreted by the gastric gland and convert the milk soluble protein casein into the insolubles protein calcium paracaseinate. 72.(c) Tidal volume is the lung volume representing the normal volume of air displaced between normal inhalation and exhalation when extra effort is not applied. In a healthy, young human, tidal volume is approximately 500 ml. 73.(a) Red cells → White cells → Platelets → Plasma, will be the sequence when human blood is mixed with an anticoagulant is centrifuged. 74.(a) Glucose reabsorption occurs in the proximal tubule. Tubular reabsorption is the process by which water, electrolytes and other substances are transported from renal tubules back to the blood. It is also known as selective reabsorption because the tubular cells reabsorbs only the essential substances such as glucose, amino acids and vitamins. Substances reabsorbed from proximal convoluted Tubule (PCT) are glucose, amino acids, Na, K, Ca, Bicarbonates, Chlorides, Phosphates, urea, uric acid and H2O. 75.(c) Aqueous humor is secreted through the ciliary body. The aqueous humor nourishes the cornea and lens. It is filled between the cornea and lens. Aqueous humor is the clear liquid inside the front part of the eye. It nourishes the eye and keeps it inflated. Imbalance of aqueous humor can lead to high intraocular pressure (IOP or eye pressure) and it is a major cause of glaucoma and can damage vision. 76.(c) In a food web vultures act as scavenger. A scavenger is an organism that mostly consumes decaying biomass, such as meat or rotting plant material. 77.(a) The two gases making the highest relative contribution to the greenhouse gases, are CO2 and CH4. ⇒ Greenhouse gases are those gases in the atmosphere that have an influence on the earth's energy balance. These gases absorbs and emits radiant energy within the thermal infrared range, causing the greenhouse effect. Practice Set 2

The main gases responsible for the greenhouse effect include carbon dioxide (CO2), methane (CH4), nitrous oxide (N2O), water vapour and ozone (O3). 78.(c) 'Nokrek' Biosphere Reserve is located in Meghalaya. It is located on the Tura range which forms part of the Meghalaya plateau. The biosphere reserve derives its name from the Nokrek Hill, the highest peak of the Garo Hill. It was added to the list of biosphere reserves by UNESCO in May 2009. It has a huge population of rare plants and animals including Citrus indica (Local Name-Memang Narang), Pangolin, hoolock gibbon, python, hornbill and red pandas. The total area is around 47.48 square kilometers looked after by the northern Nokrek and the southern Nokrek range of the Meghalaya state forest Department. 79.(c) PAN is peroxyacetyl nitrate. Peroxyacetyl nitrate is a peroxyacyl nitrate. It is a secondary pollutant. Present in photochemical smog. It is thermally unstable and decomposes into peroxyethanoyl radicals and nitrogen dioxide gas. It is a lachrymatory substance meaning that it irritates the lungs and eyes. 80.(d) A method of birth control is IUDS, (Intra Uterine device system). It is a small often T-shaped birth control device that is inserted into the uterus to prevent pregnancy. IUDs are one form of long acting reversible birth control. There are many different methods of birth control. These include condoms, IUDs, birth control pills, male sterilization and tubal ligation. 81.(d) A mature sieve tube cell of phloem is an example of enucleated cell in plants. The phloem is responsible for the conduction of food in the plants. The nuclei of the sieve tube elements are fragmented and eventually disappeared. Sieve tube cells are closely associated with the companion cells through the plasmodesmata. The nuclei of the companion cells control the activities of the sieve tube cells through the plasmodesmata. 82.(b) The main component of mangrove vegetation is rhizophora. It is a genus of tropical mangrove trees, sometimes collectively called true mangroves. It is also known as Red mangrove and is strong and tough Rhizophora grows in marshes or nitrogen and air deficient soil. These plants possess vivipary and pneumatophores or respiratory roots. 83.(a) Tropical plants like sugarcane show high efficiency of CO2 fixation because of Hatch and Slack cycle. This cycle was discovered by Marshall Davidson Hatch and Charles Roger Slack in 1960's and is also known as C4 carbon fixation, because the first formed stable compound is oxaloacetic acid (4 carbon compound). In this cycle, CO2 is added first to the phosphoenolpyruvate by the action of the enzyme PEP carboxylase. It is also referred to as the Carboxylation pathway and cooperative photosynthesis. 84.(a) Lomasome represents membrane structure. It is a complex invagination of the plasma membrane into the

49

YCT

cytoplasm. Lomasomes have been identified in some fungal hyphae and spore-producing structures, algal cells, and in some cells of higher plants. It facilitate vesicular transport and increase the surface area. Moore and McAlear coined the term lomasome. 85.(c) Bio-fertilizers are living cells of nitrogen fixing organisms. Bio-fertilizers promotes growth of plants by increasing the supply or availability of primary nutrients to the host plant. e.g. Azotobacter, Nostoc, Azolla, Anabaena, rhizobium etc. are act as bio-fertilizers. 86.(b) The nucleus is called the brain of the cell because all the biological activities that takes place in the cytoplasm are conducted by the nucleus. It is a special part of dense dark colored protoplasm, an almost spherical shape immersed in the cytoplasm. Nucleus acts as the cell's brain by telling it what to do, how to grow and when to reproduce. It is like the remote control centers of the cell. 87.(c) In cam plants, organic acid content increases during the night because the stomata remain open at the night and close during the daytime to conserve water. When stomata are open during the night, carbon dioxide is fixed by the action of Phosphoenolpyruvate carboxylose to malic acid, this is called as acidification. During daytime, when the stomata are closed, malic acid is converted to Pyruvic acid and carbon dioxide, this is called as deacidification. Hence acid concentration increases at night and decreases at daytime. 88.(a) Soil water, absorbed by Plants is called Chresard. Chresard means the total water available to the plant for absorption and not soil water available to plants. Hollard is the total water present in the field. Out of this water, the water available to the plants (usually the capillary water) is called chresard. Echard is the amount of water, which is usually not available to the plants. Whereas gravitational water is the water which has entered the water table under the influence of gravity and hygroscopic water is the water the presents in a thin film around soil particles and is not available to plants. 89.(b) Lysosomes were discovered by Christian de Duve in 1955. These are sacs surrounded by a single layer of membrane of about 0.25 to 0.50 µm diameter. It contains hydrolytic enzyme. The pH of its liquid is about 5. Lysosomes, are formed from the sacs of the Golgi or the endoplasmic reticulum. Lysosomes help in intra cellular digestion. It is also known as 'suicidal bag'. 90.(b) There are several green house gases such as CO2, CH4, N2O, water vapours that are responsible for global warming in which CO2 is the major green house gas. 91.(c) Embryologically, Trapaceae family has been separated from Onagraceae. Onagraceae family is characterized by the presence of peculiar monosporic four-nuclate Onagrad-type of embryo-sac in all its genera except trapa.

Practice Set 2

92.(b) In the process of centrifugation, first the nucleus settles at a low speed in the sediment, after which the mitochondria, chloroplasts, lysosomes, peroxisomes, microsomes and chromosomes settle down to the cytosol sediment respectively as the speed increases. 93.(c) An ecosystem is a community made up of living organisms and non-living components such as air water and mineral soil. Ecosystem had two component - biotic and abiotic component. Biotic components comprises living things like Plant, animals and decomposers and Abiotic component are temperature, light, soil, air, organic and inorganic matter etc. 94.(d) α-1, β-2 glycosidic bond is found in sucrose. In sucrose the component glucose and fructose are linked via an ether bond between C1 on the glucosyl subunit and C2 on the fructosyl unit. The bond is called a glycosidic linkage. 95.(c) Failure of meiosis and the absence of sexual union is called apomixis. Apomixis is a type of asexual reproduction. In this, there is an absence of fusion of the gametes but the fertilization of the egg takes place and embryos are formed without the need for fertilization and meiosis. 96.(d) There are some organisms which are parasitic in general. But in the absence of nutrients, the saprophytic living beings are called facultative Saprophytic. For example, Taphrina are a Facultative saprophytes. 97.(d) Fucoxanthin (C42H58O6) pigment is found in brown algae. Fucoxanthin is the main Xanthophyll pigment. It is found as an accessory pigment in the chloroplast of brown algae. Fucoxanthin makes green colour of chlorophyll pigments and that is why these algae are brown in colour. 98.(b) Ribosome's size in Archaea is 70s which consist of two sub-units 50s and 30s. Ribosomes are nucleo protein particle. They contain 3 rRNA molecules (16s, 23s and 5s) and up to 68 ribosomal proteins. The primary sequences of both, archaeal rRNA and rproteins are closer to those of eukaryotes than to those of bacteria. 99.(b) The common mode of action of herbicides is the blocking of Photosystem II. Herbicides mostly act between quinone and plastoquinone of Photosystem II (second) and blocks the flow of electron in the leaves. Herbicides binds to D1 proteins of the photosystem II complex in chloroplast thylakoid membranes. This blocks the electron transport and stops CO2 fixation and production of energy needed for plant growth. 100.(c) Neottia is an example of Saprophytic Orchid. Neottia is a genus of Orchid Neottia (Bird's Eye nest) which lack chlorophyll. 50

YCT

PRACTICE SET - 3 Which of the following physical quantity is a 11. For a thin convex lens, if object is at distance of x1 from its first focus and image at a distance of scalar? x2 from the second focus, if f1 and f2 are (a) Current (b) Area respectively the first and second focal lengths (c) Momentum (d) Angular velocity of the lens, the condition which is satisfied is The power in an alternating current is 2. (a) x1 f1 = x2 f2 (b) x1 f2 = x2 f1 measured by (c) x2 - x1 = f2 - f1 (d) x1 x2 = f1 f2 12. The Rayleigh scattering is proportional to: (a) I02 / R (b) I 2rms / R 1 1 (c) Vrms I rms cos φ (d) Vrms Irms (a) (b) 2 λ 3. Two coils are wound on a cylindrical piece of λ wood one another. If the self-inductance of each 1 1 (c) 3 (d) 4 coil is 0.1 H, the mutual inductance will be (in λ λ henry) 13. The magnifying power of an astronomical (a) 0.1 (b) 0.2 telescope is 10 and the focal length of its eyepiece (c) 0.05 (d) 0.01 is 20 cm. The focal length of its objective will be : 4. Radio waves are readily diffracted around (a) 200cm (b) 2 cm building where as light waves are negligible (c) 0 . 5 cm (d) 0.5×10–2 cm diffracted around buildings. This is because 14. When a monochrome light passes from vacuum radio waves to a material and vice-versa, which of the (a) are plane polarized following characteristics of the light beam does (b) are nearly monochromatic not change (c) have much longer wavelengths than light waves (a) velocity (b) intensity (d) have much shorter wavelengths than light waves (c) wavelength (d) frequency An electron is injected into a region of 15. Quartz is a 5. magnetic flux density with components of (a) Negative bi-axial crystal velocity parallel to and normal to the flux. The (b) Positive bi-axial crystal path of the electron is (c) Negative uni-axial crystal (a) Helix (b) Parabola (d) Positive uni-axial crystal (c) Straight line (d) Circle 16. A sound source of frequency 440 Hz is tied to the end of a 1.5 m long string and rotated in a 6. The direction of the induced current in a condhorizontal plane at an angular velocity of 20 uctor which moves in a magnetic field is given by radians/sec. What is the frequency range of the (a) Maxwell's cork-screw rule sound heard by a listener standing for away? (b) Ampere's swimming rule (Velocity of sound in air = 330 m/s) (c) Fleming's right hand rule (a) 440 – 480 Hz (b) 403 – 484 Hz (d) Fleming's left hand rule (c) 400 – 500 Hz (d) 408 – 490 Hz 7. A steady electric current flows in a metallic 17. If two tuning forks A and B are sounded conductor of non-uniform cross-section. The together, they produce 4 beats per second. A is quantity/quantities constant along the length of then slightly loaded with wax, they produce two the conductor is/are beats when sounded again. The frequency of A is (a) current, electric field and drift speed/ 256. The frequency of B will be (a) 250 (b) 252 (b) drift speed only (c) 260 (d) 262 (c) current and drift speed 18. The time in which the amplitude of a damped (d) current only system falls to 1/e of its initial value is called: 8. The angle between the equipotential surface (a) Damping time (b) Reverberation time and the electric lines of force is (c) Relaxation time (d) None of these (a) 90° (b) 180° 19. The internal radius of a 1 m long resonance (c) 0° (d) 45° tube is 3 cm. With a tuning fork of frequency 9. Polaroid A and Polaroid B are in crossed 2000 Hz, the first resonating position is 4.6 cm position. On Polaroid A unpolarized light of and the second resonating position is 14.0 cm. 2 intensity 100 W/m is incident. By rotating Speed of sound at the room temperature is Polaroid B by 60° the intensity of light coming (a) 336 m/s (b) 376 m/s out of B will be ; (c) 332 m/s (d) 340 m/s (a) 75 W/m2 (b) 37.5 W/m2 20. For the wave 50 x   t (c) 1000 W/m2 (d) W / m2 y = 10sin2π  −  3  0.03 200  the ratio of amplitude to wavelength is : 10. Chromatic aberration is due to (a) 1 : 20 (b) 1000 : 3 (a) Reflection (b) Refraction (c) 20000 : 2 (d) 2000 π : 3 (c) Diffraction (d) Dispersion 1.

Practice Set 3

51

YCT

21.

22.

Which one of the following is a simple 31. harmonic motion? (a) Ball bouncing between two rigid vertical walls (b) Particle moving in a circle with uniform speed (c) Wave moving through a string fixed at both 32. ends (d) Earth spinning about its own axis In the given figure, force of friction on body B is: 33.

(a) Towards left (c) Either left or right 23.

24.

25.

26.

27.

28.

29. 30.

(b) Towards right (d) No sufficient data

34.

The streamline flow of an incompressible and non-viscous liquid is described by (a) Boyle's law (b) Poiseuille's law (c) Archmedes principle (d) Bernoulli's principle The eccentricity of earth's orbit is 0.0167. The ratio between its maximum velocity to minimum velocity in earth's orbit will be : 3 5 11 (b) (c) (d) 1.03 (a) 2 3 7 The law of gravitation gives the gravitational force between : (a) the earth and a point mass only (b) the earth and the sun only (c) any two bodies having some mass (d) any two charged bodies only Kinetic energy needed to project a body of mass m from earth's surface to infinity is : 1 (a) mgR (b) mgR 4 1 (c) mgR (d) 2 mgR 2 ) ) ) A force F = 4i + 5j + 6k is applied on a body ) ) ) r and it is displaced by x = 3i + 7j + 8k . How much work is done? (a) 48 (b) 60 (c) 95 (d) 120 Nuclear forces are (a) Short range forces (b) Long range force (c) Electromagnetic forces (d) None of the above What is conserved in an inelastic collision? (a) Kinetic energy (b) Potential Energy (c) Total energy (d) Momentum Given four forces (i) Gravitational (ii) Electrostatic (iii) Magnetic (iv) Frictional Which of these are non-conservative? (a) All (b) Only Electrostatic (c) Only Magnetic (d) Both Magnetic and Frictional

Practice Set 3

35.

36.

37.

38.

39.

40.

41.

42.

43. 44.

52

The ionic product of water at 500C is 4 × 10-14. Point out the correct statement : (a) pH is zero and liquid is acidic (b) pH is 7 and liquid is neutral (c) pH is 6.7 and liquid is neutral (d) pH is 7.3 and liquid is basic Which of the following has the strongest acid strength? (a) HClO (b) HClO2 (c) HClO3 (d) HClO4 Two solutions with equal osmotic pressure are called : (a) normal solutions (b) isotonic solutions (c) hypotonic solutions (d) hypertonic solutions The law of reciprocal proportions was given by (a) Richter (b) Charles (c) Avogadro (d) Lavoisier Consider the reaction A+B C+D 4 moles of A react with 4 moles of B and 2 moles of C are termed at equilibrium the equilibrium constant is: (a) 2 (b) 1 (c) 2 (d) 4 Which one of the following is the most effective electrolyte in causing coagulation of a negatively charged As2S3 sol : (a) MgCl2 (b) KCl (c) K3 [Fe (CN)6] (d) Na2SO4 The point defect which lowers the density of materials is— (a) Frenkel (b) Schottky (c) Both (a) and (b) (d) None of the above For which of the following, units of rate of reaction and rate constant are identical: (a) first order reaction (b) zero order reaction (c) second order reaction (d) third order reaction The Vant Hoff factor of a 0.005 M aqueous solution of KCI is 1.95. The degree of ionization of KCl is: (a) 0.94 (b) 0.95 (c) 0.96 (d) 1.0 Which one of the following salts, on hydrolysis, will gives a solution with pH less than? (a) KCl (b) NaCl (c) NH4Cl (d) CsCl Movement of colloidal particles under the influence of an electric field is known as : (a) Electrodialysis (b) Electrophoresis (c) Brownian movement (d) Tyndall effect When initial concentration of a reactant is doubled in a reaction its half life period is not affected. The order of the reaction is (a) First (b) Second (c) Zero (d) None of the above An oxidising agent is a substance which can (a) Accept electrons (b) Donate electrons (c) Accept protons (d) Donate protons Which one of the following compound contain both ionic and covalent bond? (a) Carbon tetrachloride (b) Calcium chloride (c) Ammonium Chloride (d) Water YCT

45.

46.

47.

48.

49. 50.

51.

52.

53.

54.

55.

"A given compound always contains exactly the same proportion of elements by weight". This statement belongs to the : (a) Law of conservation of mass (b) Law of multiple proportions (c) Law of creation of mass (d) Law of definite proportions Which one of the following particles does not obey the Pauli's exclusion principle ? (a) Electrons (b) Neutrons (c) Photons (d) Protons Ethyl alcohol is industrially prepared from ethylene by : (a) Permanganate oxidation (b) Catalytic reduction (c) Absorbing in H2SO4 followed by hydrolysis (d) All the three Titanium is purified by (a) Electroytic refining (b) Zone refining (c) Smelting (d) Van Arkel de Boer's method Fehling's solution is reduced by : (b) Sodium formate (a) Sodium acetate (c) Sodium benzonate (d) Potassium succinate The oxidation number of suphur in S8, S2F2 and H2S respectively are : (a) 0, +1 and -2 (b) +2, +1 and -2 (c) 0, +1 and +2 (d) -2, +1 and -2 Which of the following groups gives WolfKishner reaction? (a) nitro (b) C = C (c) cyano (d) carbonyl Which one of the following electronic transitions requires the least energy? (a) σ → σ* (b) n → σ* * (c) π → π (d) n → π* Chlorination of benzene under photochemical condition takes place by (a) Nucleophilic substitution (b) Electrophilic substitution (c) free radical mechanism (d) Carbocation intermediate HBr The reaction: CH2=CH-COOH  → product is an example of:/ (a) Nucleophilic addition (b) Electrophilic addtiion (c) Concerted addition (d) Electrophilic substitution In which one of the following compounds, the SN1 reaction will be most difficult? (a) CH3CH2Cl (b) CH2 = CH – Cl (c)

56.

57.

(d)

58.

59.

60.

61.

62.

63.

64.

65.

66.

67.

68.

Which one of the following will give haloform reaction ? (a) CH3COCH2COOEt 69. (b) CH3CH3CHOHCH2CH3 (c) CICH2COCH2CH2 (d) CH3 COOEt Diborane combines with ammonia at 120°C to 70. give: (a) B2H6. NH3 (b) B2H6. 2NH3 (c) B2H6. 3NH3 (d) B2H6. 4NH3

Practice Set 3

53

In which of the following neutralization reactions, the value of heat of neutralization will be highest ? (a) NaOH + HCl (b) NaOH + CH3COOH (c) NH4OH + CH3COOH (d) NH4OH + HCl When propionic acid is treated with aq. sodium bicarbonate, CO2 is liberated. The CO2 comes from (a) methyl group (b) carboxylic acid (c) methylene group (d) bicarbonate Which of the following 0.1M solutions will show the highest boiling point? (a) Potassium sulphate (b) Potassium nitrate (c) Potassium chloride (d) Urea Which of the following causes food poisoning due to consumption of raw eggs ? (a) Salmonella typhi (b) Escherichia coli (c) Bacillus megathrium (d) Clostridium Botullium "Cart wheel" structure is exhibited by – (a) Ribosome (b) Lysosome (c) Centriole (d) Nucleolous One of the following is the smallest organelle in a cell : (a) Lysosome (b) Spherosome (c) Peroxisome (d) Ribosome A biochemist measure the amount of DNA in cells growing in laboratory. The amount of DNA would be found to double in _______ . (a) Between prophase and anaphase of mitosis (b) After G2 phase of the cell cycle (c) During the 'M' phase of the cell cycle (d) Between prophase-I and prophase-II of meiosis Cristae are associated with which of the following? (a) Cytoplasm (b) Protoplasm (c) Mitochondria (d) Ribosomes Oxysomes are located inside the : (a) Mitochondria (b) Plastids (c) Lysosomes (d) Golgi body Which of these are analogous organs ? (a) Wings of insects and feathers of birds (b) Forelimbs of horse and hands of human beings (c) Wings of bats and feathers of birds (d) Fins of fish and oars of penguins Species diversification represents : (a) Macro evolution (b) Mega evolution (c) Micro evolution (d) Mutation Irreversibility of evolution was stated by : (a) Dollo (b) Cope (c) Gloger (d) Gause Of the following who challenged the Lamarckian theory? (a) Charles Darwin (b) Hugo de Vries (c) A. Weismann (d) H. Spencer YCT

71.

72.

73.

74.

75.

76.

77.

78.

79.

80.

81.

82.

83.

84.

85.

Sudden inheritable genetic change is : (a) Natural selection (b) Variation (c) Mutation (d) All of these Bile is produced by : (a) Liver (b) Gall bladder (c) Pancreas (d) Intestine R. Q. is less than one when : (a) Glucose is oxidised (b) Organic acids are oxidised (c) Fats are oxidised (d) Sucrose is oxidised Oxygen dissociation curve of Haemoglobin is (a) Hyperbolic (b) Hypobolic (c) Sigmoid (d) Linear Urea formation take place in which organ of body (b) liver (a) kidney (c) lungs (d) large intestine Muscular coordination in mammals is controlled by : (a) Cerebrum (b) Cerebellum (c) Corpus callosum (d) Medulla oblongata Relationship between producers and consumers of different orders are called : (a) Food chain (b) Food web (c) Food cycle (d) Both (b) and (c) "Green House Effect" is related to which of the following cycle? (a) Phosphorus (b) Carbon dioxide (c) Nitrogen (d) Sulphur Which of the following is an example of ex-situ biodiversity conservation ? (a) Biosphere reserve (b) Gene bank (c) Hot spots (d) Sanctuary Which one of the following is ordinarily not an air pollutant : (a) CO2 (b) CO (d) Hydrocarbons (c) SO2 Find the odd man out. (a) LD50 (b) LC50 (c) EC50 (d) LE50 Major source of atmospheric methane in India is : (a) Wheat fields (b) Fruit orchards (c) Rice fields (d) Sugarcane fields Cytochromes are found in : (a) Mitochondrial matrix (b) Mitochondria cristae (c) Lysosomes (d) Mitochondrial outer wall What determines the diffusion of water from one cell to other? (a) OP (b) WP (c) DPD (d) TP Apoplast-symplast concept was given by : (a) E. Munch (b) Rabinowitch (c) Zelitch (d) Bose

Practice Set 3

86.

Which of the following water is largely available to plants? (a) Capillary water (b) Combined water (c) Gravitational water (d) Hygroscopic water 87. Plants of a community investing greater energy towards vegetative parts are called as : (a) 'r' – strategist (b) 'k' – strategist (c) ecotype (d) ecade 88. The National Parks of our country emphasize at protecting the(a) flora only (b) fauna only (c) flora and the fauna both (d) entire ecosystem 89. Prokaryotic and eukaryotic cells differ mainly in respect to : (a) Size of a cell (b) Shape of a cell (c) Chemical composition of protoplasm (d) Organization of nuclear material 90. Photosystem II occurs in : (a) Cytochrome (b) Grana (c) Stroma (d) None of the above 91. Which of the following are essential or ATP synthesis? (a) N and P (b) Fe and Mo (c) Mg and Mn (d) Fe and P 92. Which of the following animals is ferocious in nature? (a) Lions (b) Deer (c) Fish-eating Crocodiles (d) All of these 93. Ribosomes are the sites of which reaction? (a) Photosynthesis (b) Fat synthesis (c) Protein synthesis (d) Respiration 94. Among the following the best biofertilizer for rice is : (a) Notothylas species (b) Anthoceros species (c) Azolla pinnata (d) Bacillus polymyxa 95. Immobilization protects an enzyme against which of the following? (a) High pH (b) Protease (c) Low temperature (d) High temperature 96. The term protoplasm was first proposed by : (a) Watson (b) Purkinje (c) Nirenberg (d) Robertson 97. Plants that grow in a marshy saline habitat are called : (a) Xerophytes (b) Hydrophytes (c) Mangroves (d) Mesophytes 98. The respiratory quotient of flower petals is : (a) One (b) Less than one (c) More than one (d) Zero 99. Fluid mosaic model was given by : (a) Beadle and Tatum (b) Robert Brown (c) Watson and Crick (d) Singer and Nicolson 100. Main function of centromere is linked with : (a) cell enlargement (b) nucleolus formation (c) spindle organization (d) cytokinesis

54

YCT

SOLUTION : PRACTICE SET- 3 ANSWER KEY 1. (a) 11. (d) 21. (c) 31. (c) 41. (b) 51. (d) 61. (a) 71. (c) 81. (d) 91. (d)

2. (c) 12. (d) 22. (b) 32. (d) 42. (a) 52. (d) 62. (c) 72. (a) 82. (c) 92. (a)

3. (a) 13. (a) 23. (d) 33. (b) 43. (a) 53. (c) 63. (d) 73. (c) 83. (b) 93. (c)

4. (c) 14. (d) 24. (d) 34. (a) 44. (c) 54. (a) 64. (b) 74. (c) 84. (c) 94. (c)

5. (a) 15. (d) 25. (c) 35. (b) 45. (d) 55. (c) 65. (c) 75. (b) 85. (a) 95. (d)

6. (c) 16. (b) 26. (b) 36. (a) 46. (c) 56. (a) 66. (a) 76. (b) 86. (a) 96. (b)

7. (d) 17. (b) 27. (c) 37. (b) 47. (c) 57. (b) 67. (a) 77. (a) 87. (a) 97. (c)

8. (a) 18. (c) 28. (a) 38. (b) 48. (d) 58. (a) 68. (c) 78. (b) 88. (c) 98. (a)

9. (b) 19. (b) 29. (d) 39. (b) 49. (b) 59. (b) 69. (a) 79. (b) 89. (d) 99. (d)

10. (d) 20. (a) 30. (d) 40. (c) 50. (a) 60. (a) 70. (c) 80. (a) 90. (b) 100. (c)

SOLUTION 1. (a) i Vd = Electric current is a scalar quantity, because the sum n.e.A of electric currents does not obey the vector law rather i obeys the algebraic law. J= A 2. (c) Current density electric field and drift velocity varies The value of power in AC circuit is inversely with Area. P = Vrms × Irms × cos φ where (Vrms and Irms are root values of square mean of 8. (a) potential and current respectively) 3. (a) L1 = 0.1H and L2 = 0.1H ⇒ M = K L1 L2 for ideal coupling K = 1 The angle between electric field and an equipotential M = 0.1H × 0.1H ⇒ M = 0.1H surface is always 90º. This is because when the potential becomes constant the negative potential 4. (c) Radio waves are readily diffracted around building gradient also becomes zero. hence necessitating the where as light waves are negligible diffracted around need for electric field to be always normal with surface. buildings because the wavelength of radio waves is 9. (b) longer than that of light waves. Given data. 5. (a) Un polarized intensity I0 = 100w/m2 If the direction of velocity has both parallel and Rotation Polaroid B = 60 perpendicular components to the Direction of magnetic Intensity of transmit light from 2nd Polaroid field, the perpendicular component tends to move it in a I 100 circle and parallel components tends to move it along I1 = 0 = = 50 2 2 the direction of magnetic field. Hence the trajectory is a There for, Intensity of light transmitted from 2nd helix. Polaroid 6. (c) Magnetic I2 = I1 Cos2θ Motion of field o conductor = 50 Cos2 (90 - 60) 90 = 50 Cos230° o 90

2

 3 = 50 ×    2  3 = 50 × = 37.5 w/m2 4 I1 = 50 w/m2

Direction of induced current (Flemings Right hand Rule)

7. (d) When a steady current flows in a metallic conductor of non-uniform cross section the current flowing through the conductor is constant. It does not depend on area of conductor. Practice Set 3

I 2 = 37.5 w/m2 55

YCT

4 = (fA – fB) 10. (d) Chromatic aberration is colour distortion in an image As by waxing beat frequency reduced So fA > fB viewed through a glass lens. 4 = fA – fB Dispersion in lens causes chromatic aberration. fA = 256 (given) The refractive index of the lens elements changes 4 = 256 – fB depending on the wavelength of light. Therefore fB = 256 – 4 =252 different colours applied passes through a lens at 18. (c) different speeds. The time required for an exponentially decreasing 11. (d) variable (as the amplitude of a damped oscillation) to Newton's law states that- The product of distance of drop from an initial value to 1/e or (0.368) of that value object and image from respective foci is equal to the (where 'e' is the base of natural logarithms), is called product of focal length. relaxation time. 19. (b) f1 × f 2 = x 1 × x 2 Given 12. (d) According to Rayleigh scattering law, the intensity of First resonating position l1 = 4.6 cm scattering light is inversely proportional to the fourth Second resonating position l2 = 14 cm Frequency of tuning fork = 2000 Hz power of the wavelength. Q λ = 2(l2 – l1) 1 Therefore, scattering intensity IS ∝ 4 = 2(14 – 4.6) λ = 18.8 cm 13. (a) Speed of sound at the room temperature f V = fλ m= o fe 18.8 V = 2000 × f 100 ⇒ 10 = o V = 376 m/sec 20 Where m, f0 and fe are magnifying power, focal length 20. (a) of objective and eyepiece of astronomical telescope. x   t y = 10sin 2π  − f o = 200cm  0.03 200   14. (d) By comparison with standard equation • Since the frequency of any light, wave depends on the  t x source frequency and it doesn't depend on the medium. y = a sin 2π  −  T λ • So whenever the light goes from one medium to another or vice versa, the frequency of light and Amplitude a = 10, phase of light does not change. Wavelength λ = 200 • However, the velocity of light, the wavelength of Then, light and intensity depend on the medium and hence a 10 ∴ = = 1: 20 changes. Therefore option (d) is correct. λ 200 15. (d) 21. (c) A mineral consisting of silicon dioxide occurring in Simple harmonic motion is a type of periodic motion in colorless and transparent or colored hexagonal crystals which a particle moves back and forth through an or in crystalline masses. equilibrium position. In this motion its acceleration is A Quartz crystal when placed in an electric field always towards a fixed point in that line and is oscillates at a constant frequency and is used to control proportional to its distance from the point. devices which require precise regulation. Quartz is • Wave moving through a string fixed at both ends is positive uni-axial crystal. an example of SHM. 16. (b) n' = Frequency heard by the observer when source approaches the stationary observer. n" = Frequency heard by observer, when source moves away from the stationary observer. displacement equation of SHM is given by. Vs = listener's velocity = ω × l x = Asin ( ωt + φ ) = 20×1.5 = 30 m/s 22. (b) V 330 n' = ×n = × 440 = 484 Hz When force F acts on body A friction force acts in V − Vs 330 − 30 backward direction on the Body A  V   330  n '' =  × n = =   × 440 =403 Hz  330 + 30   V + Vs  17. (b) Beat = Difference in the frequency of two tuning fork when we load a tuning fork its frequency decreasesPractice Set 3

56

YCT

3. Partially elastic collision. 30. (d) Non-Conservative forces:- Forces whose work depends on the path followed between the given initial and a final configurations. Example - Frictional forces and non-elastic material stresses, magnetic forces etc. Conservative forces:- Forces whose work is independent of the path taken. Work done in a closed loop is zero. Example:- Gravitational force, electrostatic forces, etc. 31. (c)

Friction force on body B acts in forward direction to have motion. 23. (d) A streamline flow means the flow in which all fluid particles have same velocity at a particular point all the time. In case of streamline flow of incompressible fluid, the flow is ideal and the total energy per unit volume of the fluid is same at all points. The above statement is the description of Bernoulli's theorem. Hence we can say the flow of streamlined incompressible fluid is described by Bernoulli's theorem. 24. (d)





a(1 − e)

Solution in neutral  H +  = OH  = Kw = 4 ×10−14

GM(1 − e) a(1 + e)

in soltion pH = − log 2 ×10−7

1+ e U max 1+ e 1+ e ⇒ = 1− e = × U min 1− e 1− e 1− e 1+ e 1 + e 1 + 0.0167 1.0167 = = = = 1.0339 1 − e 1 − 0.0167 0.9833 25. (c) According to the law of gravity, there is a force by attraction between any two body of mass. mm F = G 1 2 2 (where m1, m2 are mass of the body, r is r the distance between them, G is universal gravitational constant) 26. (b) Any object through with escaping velocity then the object will never come back at the ground, so we can assume it will reach at infinite distance. 2 1 1 K.E = mv 2 esc = m 2gR = mgR 2 2 27. (c) rr W = F.x W = 4iˆ + 5jˆ + 6kˆ 3iˆ + 7ˆj + 8kˆ

(

(

)(

)

)

= 12 + 35 + 48 = 95 28. (a) Nuclear forces act between the protons and neutrons of atom. They are very strong force. They are called short range force since they operate over a very small distance of almost zero meters which is approximately the size of atom. 29. (d) There are three types of collision – 1. Perfectly elastic collision : In this type of collision the total kinetic energy and momentum is conserved. 2. Inelastic collision : In this type of collision the momentum is conserved and there is a loss of kinetic energy. Practice Set 3

0

neutral aqueous soltion in  H +  = OH 

Maximum velocity in earth's orbit U max = GM(1 + e) and minimum velocity U min =

(50 C)



Kw =  H +  ×  OH  = 4 × 10−14

= - log2+log 10-7 = - log2 + 7 log10 = - 0.3010 + 7 = 6.7 So, pH = 6.7 of neutral solution 32. (d) The higher the oxidation number of the elements in differrent oxyacid of the same element. the more acidic wil be. Among the given acid, this is because the conjugate base ClO −4 ion is the weakest base. Among oxyacids of halogen the decreasing order of the acid strength is : HClO4 > HClO3 > HClO2 > HClO 33. (b) Solutions which have the same osmotic pressure at the same temperature are callled isotonic solution i.e. for isostatic solution thus if two membrane at the same temperature the solvent molecules do not flow in any direction. T1=T2 on P1=P2 34. (a) The law of reciprocal proportions was given by ''Richter'' in ''1792'' it states as followsThe ratio of the masses of two element A and B which combine separately with a fixed mass of the third element 'c' is either the same or some simple multiple of the ratio of the masses in which A and B combine directly with each other. Ex - 2 grams of hydrogen react with 16 grams of oxygen and 32 grams of sulphur to form H2O and H2S respectively this according to this law, the ratio of the quantities reacting S and O will also be an absolute multiple – 32 /16 : 32 / 32 = 2 :1 35. (b) A + B  C + D initial mol 4 4 0 0 At equilibrium 4-2 4-2 2 2 C D [ ][ ] = 2 × 2 =1 equilibrium constant K = [ A ][ B] 2 × 2

57

YCT

45. (d) 36. (a) The more positively charged ion is the most effective Proust propounded the law of definite proportions. electrolyte for coagulation negatively charged sol. According to this rule any-the weights of the constituent Hence MgCl2 is the most effective electrolyte because elements in a chemical compound always have a definite the ratio remains. KCl, K3 [Fe (CN)6] and Na2SO4 the ion is positive. Example- H2O the ratio of the weights of hydrogen and 37. (b) Schotlky defect the lowers the density of a solid oxygen 2 : 16 = 1 : 8. The ratio of the weights of because 1 pair of cation and anion are found missing in hydrogen and oxygen in water obtained from any source this point defect lowers the density of materials is called is always 1 : 8. 46. (c) schottky deffect. Pauli exclusion principle states that no two identical 38. (b) The unit of rate of reaction and rate constant are electrons (fermions) can have the same quantum state those that do not obey this principle are called bosons. identical is called zero order of reaction. Photons, gravitons and gluons are an example of 39. (b) bosons. So, photons particles does not obey the Pauli's The vant Hoff factor is the ratio between the actual concentration of particles produced when the substance exclusion principle. as calculated from its mass. For most non electrolytes 47. (c) Manufacture of ethyl alcohol on an industrial scale, dissolved in water the Vant Hoff factor is essentially I. made by different method then ethylene, high pressure + − KCl  K + Cl and on absorption of ethylene in concentrated H2SO4 at at initial 1 0 0 80ºC ethyl hydrogen sulfate and ethyl sulfate are in equilibrium 1–α α α formed. Vant Hoff factor i = 1+ α 800 C → CH 3 CH 2 OSO 3 H CH 2 = CH 2 + H 2SO 4  high pressure 1.95 = 1+ α ethyl hydrogen α = 0.95 sulphate 40. (c) heat CH CH OSO H + H O   → C H OH + H 2SO 4 KCl, NaCl, CsCl is perfectly ionized and formed neutral 3 2 3 2 2 5 solution which contain value of pH is equal to 7. But group obtained by diluting the mixture with water and NH4Cl is weak base and strong acid salt that partially then distillation aqueous solution of ethyl alcohol is ionized and formed acidic solution which contain pH obtained. value is less than 7. 48. (d) 41. (b) Van Arkel de Boer's method -: The Van arkel-de Boer When a colloidal solution is placed in an electric field, process also known as the iodide process for the crystalthe colloidal particles move towards the anode. The bar process, was the first industrial process for the movement of colloidal particles under the influence of commercial production of pure ductile "titanium''. electric field is called electric particles movement. The "zirconium" and some other metal. It was developed by movement of colloidal particles towards the cathode is Anton Eduard Van Arkel and Jan Hendrik de Boer's in called positive particles movement and towards the 1925. Now it used in the production of small quantities of ultrapure titanium and zirconium. anode is called negative particles movement. 49. (b) 42. (a) A red precipitate of cuprous oxide is formed on heating 1 In first order reaction t ½ ∝ n −1 , here n = 1, so, t½ does aldehyde with Fehling's solution hence sodium formate a is heated with Fehling solution. not depend on initial concentration. 50. (a) When initial concentration of a reactant is doubled in a Oxidation number of 'S' in S8 = 0 reaction, its half life period is not affected. The order of Oxidation number of 'S' S2F2 = +1 the reaction is first. The half life period t1/2 of the first Oxidation number 'S' H2S = –2 order reaction is independent of the initial cncentration S F → 2x–2=0 ⇒ x=+2/ =+1 2 2 2 of the reactant. H2S → +2+x=0 ⇒ x = – 2 0.693 51. (d) t 1/ 2 = k Reduction of carbonly group to hydrocarbon group by 43. (a) basic hydrazine (NH2-NH2) it is called Wolf-Kishner An oxidising agent is a substance which can accept reaction. electrons. An oxidising agent or oxidant gains electrons NH 2 NH 2 CH 3 CHO → CH 3 − CH 3 and is reduced in a chemical reaction, also known as KOH / glycol the 'electron acceptor'. The oxidising agent is normally Aldehyde Ethane in one of its higher possible oxidation states because it 52. (d) will give electrons and be reduced. The energy requirement order for excitations of 44. (c) NH4 Cl contains an ionic and a covalent bond. different transition is as follows:Ammonium ion is polyatomic. It is ionic however, the (a) n → π* transition requires lowest energy bond between N and H is covalent because both N and (b) σ → σ* requires highest amount of energy. H are non-metals. Practice Set 3 58 YCT

53. (c) Benzene chlorination occurs through free radical system under photochemical condition. H2 Cl 2  → 2Cl hν

54. (a) HBr CH2=CH–COOH  → product Mechanism

Here, presence of –COOH, shows nucleophilic addition reaction. 55. (c) In SN1 react with great difficulty because carbocation intermediate is not easily formed. 56. (a) -COCH3 containing compounds react with X2 gas in the presence of base produced haloform. CH3COCH2COOEt This compound will give haloform reaction due to containing -COCH 3 group. − 2 EtOOCCH 2COCH3  → EtOOCCH 2COO− + CHX3 haloform rex n haloform

X / OH

57. (b) When diborane is treated with ammonia at 1200C the B 2 H 6 .2 NH 3 is formed. 0

120 C B2 H 6 + 2 NH 3  → B2 H 6 .2 NH 3 58. (a) When acid and base react with each other, so salt is formed this reaction is called neutralization reaction this the stronger the acid and the base, the stronger it is higher is the value of heat of neutralization so, for the reaction NaOH + HCl → NaCl + H 2 O the value of heat of neutralization will be highest. 59. (b) When propionic acid is treated with aqueous sodium bicarbonate (NaHCO3), CO2 is liberated. The CO2 comes from bicarbonate.

NaHCO

3 CH3 CH 2 COOH  → CH3CH2 COONa + CO2 + H 2 O CO2 releases from bicarbonate not from propionic acid. 60. (a) Higher the number of ions in aqueous solution of a compound higher will be its boiling point. Thus number of ions in K 2SO 4 → 2K + + SO −4 − Therefore, the boiling point of potassium sulphate solution will be highest. 61. (a) Food poisoning due to consumption of raw eggs is caused by Salmonella typhi. Salmonella is a genus of rod-shaped, gram-negative bacteria. Raw and

Practice Set 3

undercooked eggs contain may Salmonella, which contaminates the outside of the egg and penetrates through the shell membrane. Symptoms of Salmonella food poisoning include stomach cramps, diarrhea, vomiting, and fever. Salmonella typhi infect the intestinal tract and the blood. 62. (c) "Cart wheel" structure is exhibited by centriole. The cartwheel is a striking structure critical for building the centriole, a microtubule - based organelle fundamental for organizing centrosomes. cilia and flagella. 63. (d) The smallest organelle in the cell is the ribosome. The diameter of the ribosome is about 20 nm. It is made up of small and large subunits. It is the site for the manufacture of proteins within the cell. The small subunit is involved in reading the codons on the mRNA and the large sub-unit helps in the attachment of the amino acids to form a linear chain of the Protein. 64. (b) The amount of DNA will be double after G2 phase of the cell cycle. G2 phase is a period of rapid cell growth and protein synthesis during which the cell prepares itself for mitosis. The cell cycle is divided into mitotic phase (M phase) and interphase. The interphase is divided into G1, S and G2 phase. During interphase the cell grows and rests between two consecutive cell division. 65. (c) Cristae is the inner membrane foldings in mitochondria. It is the finger - like projection of the inner mitochondrial membrane. The inner membrane as well as cristae posses elementary particles or oxysomes. These oxysomes function as ATPase and thus provides the site for ATP synthesis. 66. (a) Oxysomes are located inside the mitochondria. They are also called elementary particles or F–1 particles (F stands for Ferdinand). These particles are present on the cristae of mitochondria. There are about 104 –105 oxysomes, regularly placed at the intervals of 10 nm. Oxysomes are responsible for oxidative phosphorylation, which involves the formation of ATP from ADP and inorganic phosphate. 67. (a) Wings of insects and feathers of birds are analogous organs. Wings of butterfly, bird and bat are analogous organs. Those organs of different animals whose origin and structure are different but perform a common function, are called analogous organs. 68. (c) Species diversification represent the microevolution. It produce small change in the population. Mega evolution is defined as the formation of new species group. Microevolution is diversification with in a species and macroevolution is diversification between species. 69. (a) Irreversibility of evolution was stated by Louis Dollo in 1893. Dollo's law of irreversibility states that once

59

YCT

a complex trait has been lost in evolution, it cannot be regained. An organism never gets back to its original state even if the conditions are favorable. Thus, the organism goes through an irreversible type of evolution. 70. (c) German biologist August Weismann (in the 1880 s) disapproved lamarckism by formulating the germ- plasm theory of inheritance. Weismann reasoned that reproductive cells (germ cells) were separate from the functional body cells (soma or somatic cells). Hence, change to the somatic cells would not affect the germplasm and would not be passed on to the offspring. In order to prove that the misuse or loss of somatic structure would not affect the subsequent offspring, he removed the tail of mice and then allowed them to breed. After twenty generations of this experimental protocol, he found that mice still grew tails of the same length as those who had never been manipulated. 71. (c) Sudden inheritable genetic change is mutation. A mutation is a change in a DNA sequence. These changes occur at many different levels, and they can have widely differing consequences. Mutations can result from DNA copying mistakes made during cell divison, exposure to ionizing radiation, exposure to chemicals called mutagens, or infection by viruses. 72. (a) Bile is produced by liver. It is a fluid that is made and released by the liver and stored in the gall bladder. It breaks down fats into fatty acids and helps in digestion. It is dark green to yellowish brown in colour. Bile is composed of water (97-98%), bile salts (0.7%), bilirubin (0.2%), fats (0.51%) and 200 meq/l inorganic salts, about 400-800 ml of bile is produced per day in adult human beings. 73. (c) R.Q. is less than one when fats are oxidised. Respiratory Quotient (RQ) is the ratio of the volume of carbon dioxide produced over the volume of oxygen consumed during cellular respiration. The instrument used for measuring RQ is Respirometer. ⇒ RQ for carbohydrates is 1. ⇒ RQ for proteins is less than 1. ⇒ RQ for organic acids is more than 1. 74. (c) The percentage of hemoglobin that has taken up oxygen is called its percentage saturation. It depends on the partial presure of oxygen in the blood i.e. PO2. This relationship between the partial pressure of oxygen (PO2) and the percentage saturation of hemoglobin is graphically depicted by a curve called the oxygen hemoglobin dissociation curve. It is always of the shape of sigmoid i.e. S- shaped. 75. (b) The chemical formula of Urea is NH2CONH2. Urea is produced in the liver by ornithine cycle or Urea cycle. In this cycle ammonia and CO2 react with each other resulting in the formation of Urea, some amino acids such as citrulline and ornithine are formed in this cycle.

Practice Set 3

Liver cell synthesize Urea with the help of enzyme like arginase urease etc. This Urea is excreted by the kidneys from the blood along with Urine. 76. (b) Cerebellum control equilibrium and coordination of voluntary muscular movement. Corpus callosum is characteristic feature of mammalian brain found in cerebrum hemisphere. 77. (a) Producers and consumers represent different trophic levels of a food chain. A food chain shows that flow of energy through organisms in an ecosystem producers and consumers are part of the chain. Producers use energy and inorganic molecules to make food. Consumers take a food by eating producers or other living things. 78. (b) "Green House Effect" is related to carbon dioxide (CO2) cycle. The warming of earth's surface and troposphere, caused by the presence of water vapour, CO2, methane, and certain other gases in the air, is known as greenhouse effect. The carbon dioxide cycle plays a key role in regulating Earth's global temperature and climate by controlling the amount of CO2 in the atmosphere. 79. (b) Gene bank is an example of ex-situ biodiversity conservation. Gene banks are a type of biorepository that preserves genetic material. Ex-situ conservation is the technique of conservation of all levels of biodiversity outside their natural habitats through techniques like zoo captive breeding, aquarium, botanical garden and gene bank. 80. (a) CO2 is an important trace gas in earth's atmosphere. It is an integral part of carbon cycle. CO2 is not a pollutant at a normal level in the atmosphere but if its level is increased in the environment, they are pollutant in the atmosphere. 81. (d) LD50, LC50 both represents amount and concentration respectively, at the same time EC50 is strongly related to concentration. EC50 means half maximum effective concentration, In a pharmacological context. LE50 is a laser measure and not related with toxicology. 82. (c) Methane is chiefly produced in marshy place such as rice field. In India rice has largest cultivation area. So they are chief source of methane in India. The paddy cultivation occurs in water logged condition and favours the release of methane after the bacteria has respired aerobically. 83. (b) Cytochromes are found in mitochondria cristae. They are redox-active proteins containing a heme, with a central Fe atom at its core, as a cofactor. Cytochromes are the main electron carriers present in the electron transport chain, and involved in ATP synthesis during aerobic respiration. Mitochondria is covered by a bilayered membrane. The inner membrane is folded in the form of shelf-like inward projections called cristae, and it covers the inner space.

60

YCT

84. (c) The force which determines the flow of water from one cells to another is diffusion pressure deficit. Diffusion pressure deficit (DPD) is the amount by which two solution differ in their diffusion pressure. It is defined or the amount by which the diffusion pressure of water or solvent in a solution is lower than that of pure water or solvent. It allows easy absorption of water due to diffusion pressure gradient water moves from lower DPD to higher DPD. 85. (a) Apoplast-symplast concept was given by E.Munch. The term apoplast was coined by Munch in 1930 to distinguish the living symplast from the dead apoplast. ⇒ The apoplast is the free space outside the plasma membrane. It is interrupted by the casparian strip by the spaces between the plant cells in the roots and the pores of the plant. 86. (a) Capillary water is largely available of plants. It is the water that remains in the soil after gravitational water is drained out. This water occurs as a film around soil particles and in the pores between them, and is the main source of plant moisture. It can move in any direction within the soil, by capillary action. ⇒The water which percolates deep in the soil due to gravity is called gravitational water. ⇒ The water absorbed from the atmosphere and help very tightly by the soil particles is, called hygroscopic water. 87. (a) The plants of the community which invest more energy in the growing part are called 'r' – strategist. 88. (c) National park is conservation areas where certain species of fauna (wild animal) and flora (group of specific plants) are protected. National park provide a safe space for wild life to breed and survive. Without national park certain animal (fauna) species are at greater risk of becoming extinct. A landscape needs to be protected in order to provide habitat for wild life. 89. (d) Prokaryotic and eukaryotic cell differ mainly based on Organization of nuclear material. In prokaryotic cell, nucleus is absent and in eukaryotic cell nucleus is present. The nucleus is where eukaryotes store their genetic information. Most noteworthy, eukaryotes have a membrane - bound nucleus while the prokaryotes are without it. 90. (b) Photosystem II is the first protein complex in the lightdependent reactions of oxygenic photosynthesis. It is located in the grana of the thylakoid membrane of plants, algae and cyanobacteria. 91. (d) Fe and P, are essential for ATP synthesis Mitochondria are the major source of ATP synthesis in cells during aerobic respiration. ATP is synthesis from its are found in the cristae and the inner membrane of mitochondria. Iron (Fe) is present in the iron-sulpher cluster and heme groups of the electron transport chain proteins that generate a proton gradient that allows ATP synthase to Practice Set 3

synthesize ATP phosphorus (P) is involved in the energy transfer mechanism, including the generation of ATP. 92. (a) Lion is a ferocious animal. It is one of the four big cats of the genus Panthera and is a member of the felidae family. Wild lions are currently found in Sub-Saharan Africa and Asia. 93. (c) Ribosomes are called the site of protein synthesis. Protein molecules are made up of one or more types of polypeptide chains. Polypeptide chains are formed by polymerization of 20 types of amino acid units. The synthesis of the polypeptide chains of proteins take place outside the nucleus in the cytosol of the cell. 94. (c) Azolla pinnata is the best biofertilizers for rice field . It have been used to control mosquito larvae in rice field. 95. (d) Immobilization protects an enzyme against high temperature. Immobilization improves many properties of enzymes such as performance in organic solvent. It allows one to reuse the enzymes for an extended period of time and enables easier separation of the catalyst from the product. 96. (b) Purkinje was a physician and physiologist. In 1839. He introduced the term protoplasm for the fluid substance of a cell. He was one of the best known scientist of his time. Protoplasm is composed of a mix of organic and inorganic substances like monosaccharides, water, ions, amino acids, as well as polysaccharides, nucleic acids, proteins and lipids. 97. (c) Plants that grow in a marshy saline habitat are called Mangroves. A Mangrove is a shrub or small tree that grows in coastal saline. The term is also used for tropical coastal vegetation consisting of such species. e.g. Rhizophora 98. (a) The respiratory quotient of flower petals is One. The respiratory quiotient (RQ) also known as the respiratory ratio is defined as the volume of carbon dioxide released over the volume of oxygen absorbed during respiration. RQ is one when hexose sugars are consumed in respiration. 99. (d) Fluid mosaic model was given by Singer & Nicolson in 1972. According to this model, plasma membrane have bimolecular lipid layer in the centre and protein layer on outer side. This model explains the structure of the plasma membrane of cells as a mosaic of components such as phospholipids, proteins, cholesterol and carbohydrates. 100. (c) Main function of centromere is linked with spindle organization. The centromere is to provide the foundation for assembly of the kinetochore, which is a protein complex essential to proper chromosomal segregation during mitosis.

61

YCT

PRACTICE SET - 4 1.

2.

3.

4.

5.

6.

7.

Read the following statements and choose the correct answer on the basis of codes given below the statements: Assertion (A): An electric current is a scalar. Reason (R): It does not obey the law of vector addition. Codes: (a) Both A and R are true and R is the correct explanation of A (b) Both A and R are true but R is no the correct explanation of A (c) A is true but R is false (d) A is false but R is true In a step up transformer, the turn ratio is 1 : 2. A Leclanche cell (emf = 1.5 V) is connected across the primary. The voltage across the secondary is: (b) 1.5 volt (a) 3 volt (c) 0.75 volt (d) 0 volt Two circuits have a coefficient of mutual inductance of 0.09 henry. E.M.F induced in the secondary circuit by a change of 0 to 20 ampers in 0.006 seconds in the primary, is: (a) 200 volt (b) 300 volt (c) 120 volt (d) 180 volt

Two charges q1 and q2 are placed close to each other, if a third charge q3 if brought near to it, the force now exerted by q1 on q2 (a) increases (b) decreases (c) remains the same (d) reduces to zero 9. In Young's double slit experiment, if the screen is moved away from the plane of slits, then (a) Angular fringe width will decrease (b) Angular fringe width will increase (c) Angular fringe width will remain constant (d) Angular fringe width first increases then decreases 10. The dispersive powers of lenses used in an achromatic combination are in ratio 5 : 3. If focal length of the concave lens is 15 cm, the nature and focal length of other lens will be− (a) Convex, 9 cm (b) Concave, 9cm (c) Convex, 25cm (d) Concave, 25 cm 11. The number of cardinal points in a thin lens is : (a) zero (b) two (c) three (d) six 12. Rayleigh criterion of just resolution is (a) when maximum of one image lies on the minimum of another image. (b) when maximum of one image lies on the The phenomenon of adiabatic demagnetization maximum of other image. is used to (c) when minimum of one image lies on the (a) Demagnetize a magnet minimum of other image. (b) Produce very low temperature (d) no condition exists (c) Generate electricity 13. An astronomical telescope of 10 fold angular (d) Purify a gas magnification has a length of 44cm. The focal A magnetic needle laying parrallel to a magnetic length of the objective is: field requires W units of work to turn it through (a) 4 cm (b) 40 cm 60°, the torque needed to maintain the needle in (c) 44 cm (d) 440 cm this position will be. 14. Law of refraction was given by (b) W (a) 3 W (a) Maxwell (b) Hygen (c) 3/2W (d) 2W (c) Newton (d) Snell The incorrect statement is 15. A convex mirror forms an enlarged image of an object (a) Electromagnetic waves are transverse (a) For no values of object distance (b) Electromagnetic waves travel in vacuum with the speed of light (b) When object distance is less than twice the focal length (c) The speed of electromagnetic waves in same in all the media (c)When object distance is more than twice the focal length (d) Electrmagnatic waves are emitted by an accelerated charge. (d)For all values of object distance The energy which an electron acquires when 16. The law applicable for determining the accelerated through a potential difference of 1 apparent change in frequency when a source volt is and an observer are in motion is (a) 1 joule (b) 1 electron-volt is (a) Doppler's law (b) Huygen's law (c) 1 erg (d) 1 watt (c) Newton's law (d) Galileo's law

Practice Set 4

8.

62

YCT

17.

18.

The difference in the frequencies of two sounding 26. object is more than 20 Hz, then the beats (a) Are not formed at all (b) Are heard with increased clarity (c) Cease to be distinguishable (d) Are inaudible A mass of 1.0 kg is suspended from a heavy spring whose force constant K = 1000 N/m. The mass of spring is 3.0 kg. The time period T will be: (a) 2π

1 1000

(b) 2π

4 1000

27.

2 3 (d) 2π 1000 1000 Velocity of sound in air is 332 m/s. Its velocity in vacuum will be (a) More than 332 m/s (b) Less than 332 m/s (c) Equal to 332 m/s (d) None of these Which of the following body allows only a part of light to pass through it (b) Brick (a) Oiled paper (c) Wood (d) Air What is the percentage change in time period of a simple pendulum, if the length of simple pendulum increase by 3%? (a) 3% (b) 6% (c) 3.5% (d) 1.5% Work- energy theorem for linear motion of any particle is based on the : (a) Newton's first law of motion (b) Newton's second law of motion (c) Newton's third law of motion (d) Law of conservation of linear momentum (c) 2π

19.

20.

21.

22.

28.

29.

30.

31.

The velocity of oil in a 5 cm internal diameter 32. pipe is 0.63 m/s. The velocity in a 3 cm internal diameter pipe that connects the first one is: (a) 1.75 m/s (b) 1.50 m/s (c) 3.0 m/s (d) 9.0 m/s 24. The time period of revolution of a satellite is given by : Where symbols have their usual meaning. 33. 23.

(a) T =

4πR 3 GM

(b) T =

4π 2 R 3 GM

34.

2π2 GM R (d) None of the above 25. The atmosphere consisting of a large number of 35. gases is held to the earth due to : (a) winds (b) clouds (c) earth's magnetic field (d) gravity (c) T =

Practice Set 4

63

Choose the correct statement for centre of mass reference frame : (a) The momentum of system of particles is always zero in this frame (b) The momentum of the system of particles can never be zero in this frame (c) The momentum of the system of particles is never conserved in this frame (d) The momentum of the system of particles may or may not be zero in this frame Two bodies of mass m and 4 m are moving with equal kinetic energy. The ratio of their linear momentum will be : (a) 1 : 2 (b) 1 : 4 (c) 4 : 1 (d) 2 : 1 The ratio of weights of a man inside a lift when it is stationary and when it is going down with a uniform acceleration a is 3 : 2. The value of a is g 3 2 (a) g (b) (c) g (d) g 2 3 3 A force of 10 N acts on a particle in vertically upward direction and it is displaced downward direction by a distance 5.0 m. The work done is: (a) +10 J (b) +50 J (c) –50 J (d) –10 J Which of the following forces is an example of the non-conservative force? (a) Gravitational force (b) Elastic force (c) Electrical force (d) Frictional force Which one of the following salts on hydrolysis will give a solution with pH more than 7? (a) K2SO4 (b) KNO3 (c) CH3COONa (d) NH4Cl Approximate degree of hydrolysis of 0.05 M solution of sodium acetate at 2980K, given that dissociation constant of acetic acid is 1.85 × 10-5 m2 and ionic product of water is 1.10 × 10-14 m2 (both at the same temperature), will be: (a) 10.9 × 10-5 (b) 11.2 × 10-4 -5 (c) 12.0 × 10 (d) 9.9 × 10-5 La chateliers principle is applicable on – (a) At reversible system (b) At irreversible system (c) At both the system (d) At neutralisation reactions The equilibrium concentrations of x, y and yx2 are 4, 2 and 2 respectively for the equilibrium 2x + y yx2' The value of equilibrium constant, Kc, is: (a) 0.625 (b) 6.25 (c) 0.0625 (d) 62.5 For the Stabilization of a 100 c.c. Colloidal Solution 0.25 gm starch is added which prevents Precipitation on addition of 1ml NaCl solution. What will be the gold number of starch? (a) 25 (b) 2.5 (c) 0.25 (d) 250 YCT

36.

37.

38.

39.

An acidic buffer solution can prepared by mixing which of following pair of solutions? (a) (NH4)2 SO4 and H2SO4 (b) CH3COONa and CH3COOH (c) NH4Cl and NH4OH (d) NaCl and HCl The combustion enthalpies of carbon and carbon monoxide are –393.5 and –283kJ mol–1 respectively. The enthalpy of formation of carbon monoxide is— (a) 110.5 kJ mol–1 (b) 676.5 kJ mol–1 (c) –676.5 kJ mol–1 (d) –110.5 kJ mol– Half-life period of a first order reaction is 100 sec. The rate constant of the reaction is (a) 6.93 × 10–4 sec–1 (b) 6.93 × 10–3 sec–1 –4 –3 (c) 0.693 × 10 sec (d) 69.3 sec–1 Number of components, phases and degree of NH3(g) + HCl(g), when freedom in NH4Cl(s) PNH ≠ PHCl are, respectively :

45.

46.

47.

48.

4

40.

41.

42.

43.

44.

(a) 1, 2, 1 (b) 2, 2, 2 (c) 1, 1, 2 (d) 2, 1, 2 The half-life period of a substance is 100 sec. at a certain concentration. When its concentration is doubled, the half-life period becomes 200 sec. The order of the reaction is (a) Zero (b) 1 (c) 2 (d) 3 Which of the following elements does not form oxyacids ? (a) Chlorine (b) Nitrogen (c) Sulphur (d) Fluorine The platinum electrodes were immersed in a solution of CuSO4 and current was passed through it. After some time it was found that colour of the solution of CuSO4 disappeared with the evolution of gas at the electrode. The colourless solution of gas at the electrode. The clourless solution consists of (a) Platinum sulphate (b) Cuprous sulphate (c) Sulphuric acid (d) Cuprous hydroxide Which of the following statements is correct? (a) For spontaneous process, ∆ G > 0 (b) Entropy is an extensive property (c) Enthalpy values of neutralization of all types of acids with bases are same (d) During Joule-Thomson expansion, the temperature of all gases decrease above their inversion tempertaure The combination of atoms take place due to (a) Acquire stability by increase in energy (b) Acquire stability by decrease in energy (c) Acquire high energy state (d) Acuire instability

Practice Set 4

49.

50.

51.

52.

53.

64

∇ 2 in∇ 2 ψ +

8π2 m ( E − V )

ψ = 0 is known as: h2 (a) Laplacian operator (b) Hamiltonian operator (c) Angular momentum operator (d) Energy operator The binding power of haemoglobin with oxygen is pH dependent. This is called (a) Hill constant (b) Bohr effect (c) Trans effect (d) None of the above "In an atom no two electrons can have same values for all the four quantum numbers." This is called : (a) Aufbau Principle (b) Hund's Rule (c) Heisenberg Uncertainty Principle (d) Pauli's Exclusion Principle Malachite and bauxite are the ores respectively of (a) Fe and Al (b) Cu and Fe (c) Cu and Al (d) Cu and Li Which of the following acids can be used as catalyst in Friedel-Crafts reaction? (a) AlCl3 (b) BF3 (c) Both (d) None of these Salicylic acid reacts with acetic anhydride to form : (a) Oil of wintergreen (b) Aspirin (c) Mustard oil (d) Salol Consider the following statements about SN2 reactions : 1. They are bimolecular reactions. 2. They proceed with complete inversion of configuration. 3. They involve the formation of an intermediate 4. No rearrangement is involved in them. Select the correct answer from the codes given below . (a) Only 1, 2 and 3 are correct (b) Only 2, 3 and 4 are correct (c) Only 1, 3 and 4 are correct (d) Only 1, 2 and 4 are correct Which one of the following statements regarding Reimer-Tiemann reaction is false ? (a) Reaction of phenol with CHCl3 and KOH (b) :CCl2 acts as an electrophile in the reactions (c) Reactionn of phenol with CCl4 and NaOH (d) Reaction of phenol with formaldehyde Toluene reacts with bromine in presence of U. V. light to bromide or bromine (a) Benzoyl bromide (b) Benzyl bromide (c) m-bromotoluene (d) p-bromotoluene YCT

54.

55.

56.

57. 58.

59.

60.

61.

62.

63.

64.

65.

66.

Which one of the following compounds will form a hydrocarbon on reaction with Grignard reagent? (a) CH3CHO (b) CH3COCHO3 (d) CH3COOCH3 (c) CH3CH2OH Which of the following haloalkanes will undergo hydrolysis most readily ? (a) (CH3)3CF (b) (CH3)3CCl (c) (CH3)3CBr (d) (CH3)3Cl When acetic acid is mixed with sodium acetate, the pH value : (a) Increases (b) Decrease (c) Becomes 7 (d) Does not change Acetylene is obtained by the electrolysis (a) Sodium succinate (b) Sodium fumarate (c) Both (d) None of these Isopropyl alcohol is heated at 3000C with copper catalyst to form : (a) acetone (b) dimethyl ether (c) acetaldehyde (d) ethane A hydrocarbon, C5H8 upon ozonolysis yielded only one product. The hydrocarbon is : (a) cyclopentene (b) 2-Methyl-1, 3-butadiene (c) 2-Pentyne (d) 1, 4-Pentadiene In the reaction 2H 2 O H 3 O + + OH bonding (a) A weak base (b) A weak acid (c) Both a weak acid and a weak base (d) Neither an acid nor a base Cry-I endotoxins obtained from Bacillus thuringiensis are effective against – (a) Nematodes (b) Boll worms (c) Mosquitoes (d) Flies In metazoa, groups of cells similar in structure and function are called: (a) tissue (b) organ (c) system (d) body Krebs cycles completed in : (a) Nucleus (b) Chloroplast (c) Cytoplasm (d) Mitochondria The cellular event that corresponds to Mendel's law of independent assortment is : (a) Anaphase I (b) Anaphase II (c) Metaphase I (d) Metaphase II In which phase of cell cycle multiplication of mitochondria and chloroplast take place? (a) G0 (b) S (c) G2 (d) M Homologous structures are : (a) Similar in origin, but dissimilar or similar in function. (b) Similar in origin and function.

Practice Set 4

67.

68.

69.

70.

71.

72.

73.

74.

75.

76.

77.

65

(c) Dissimilar in origin but similar in function. (d) Dissimilar in origin and function. If an allele has adaptive value of 0.55, what will be its selection coefficient? (a) 0.45 (b) 0.64 (c) 0.1 (d) None of the above Haeckel's recapitulation theory was based on : (a) Genetic Variations (b) Germ layers (c) Germplasm (d) None of the above 'Camels' are found in Asia but related to 'Llamas' in the Andes. This is an example of : (a) Survival of the fittest (b) Emergent evolution (c) Isolation (d) Discontinuous distribution Mutation was described by : (a) de Vries (b) Lamarck (c) Darwin (d) None of these Which of the following is not present in the bile juice? (a) Digestive enzymes (b) Cholesterol (c) Phospholipids (d) Bile salts What is the net gain of ATP molecules when one molecules of glucose is broken down into two molecules of pyruvic acid during glycolysis with in a cell : (a) two (b) four (c) thirty-eight (d) forty In healthy adult man, the smallest type of leucocytes are : (a) Lymphocytes (b) Basophils (c) Monocytes (d) Eosinophils Kidneys are vital organs of the body because they help in : (a) regulation of body fluids (b) regulation acid base balance (c) removal of metabolic wastes (d) all the above functions Matrix of mucoid connective tissue that surrounds the two arteries and one vein of the umbilical cord is : (a) Barton's jelly (b) Wharton's jelly (c) Lymph (d) Karton's jelly The efficiency of an ecosystem is best depicted by pyramid of : (a) Biomass (b) Energy (c) Number (d) Volume The gas responsible for causing acid rain is : (a) SO2 (b) O3 (c) CO2 (d) NH3

YCT

78.

79.

80.

81.

82.

83.

84.

85.

86.

87.

88.

Nandan Kanan Zoo is known for the : (a) Nilgiri Tiger (b) Hippopotamus (c) White Tiger (d) Whale Which of the following air-polluting gases is emitted exclusively from refineries or pulp mills: (a) CO2 (b) SO2 (c) H2S (d) NO2 What is a xenobiotic ? (a) Insecticide (b) Fungicide (c) Herbicide (d) All of the above Partially Decomposed litter, partially or fully is known as : (a) calcicol (b) humus (c) latesol (d) podosol Excess of which of the following in the aquatic system leads to eutrophication : (a) Nitrate and chloride (b) Chloride and phosphate (c) Sulphate and Chloride (d) Phosphate and Nitrate The organelle which is associated with photorespiration is – (a) Lysosome (b) Glyoxysome (c) Ribosome (d) Peroxysome Middle lamellae is made up of : (a) Suberin (b) Calcium pectate (c) Cellulose (d) Lignin Loss of water from the tips of leaves is called : (a) Guttation (b) Transpiration (c) Respiration (d) Bleeding Biomagnification phenomenon is related to : (a) CO2 (b) NOx (c) DDT (d) SO2 Two plants show normal chromosome bivalents in meiosis. When the two are crossed, the progeny shows-a-quadrivalent. This suggests that : (a) Both the plants are translocation heterozygotes (b) Both the plants are translocation homozygotes (c) One plant is normal and the other is translocation heterozygote (d) Both plants are tetrasomics In the leaves, yellow to yellow-orange colour is because following pigment : (a) Chlorophylla (b) Chlorophyllc (c) Xanthophills (d) Carotenoids

Practice Set 4

89.

Mountain Zone of Western Himalayan Floristic region extends from : (a) 1,000 to 5, 000 ft. above sea level (b) 5,000 to 10,000 ft. above sea level (c) 5,000 to 11,675 ft. above sea level (d) 6,000 to 11,675 ft. above sea level 90. Suicidal bag is another name of (a) Mitochondria (b) Ribosome (c) Lysosome (d) Chloroplast 91. Phytoalexin was first reported by : (a) Muller (1940) (b) Koch (1882) (c) Millardet (1885) (d) Prevost (1807) 92. Secondary treatment of sewage is : (a) biological degradation of organic materials by micro-organisms under controlled condition (b) chlorination of sewage to kill the pathogenic micro-organisms (c) separation of colours from the sewage (d) separation of heavy metals from the sewage 93. The unit of measurement in transmission electron microscopy is : (a) .001 µm (b) .00001 µm (c) .01 µm (d) .0001 µm 94. Ac-Ds system in maize was discovered by : (a) McClintock (1950) (b) Muller (1975) (c) Morgan (1933) (d) Metz (1934) 95. Which of the following organism does not fix atmospheric nitrogen : (a) Nostoc (b) Azotobacter (c) Clostridium (d) Microcystis 96. During which phase of meiosis does crossing over take place: (a) Zygotene (b) Leptotene (c) Pachytene (d) Diplotene 97. Which one of the following generally increases during senescence ? (a) Chlorophyll (b) Protein (c) Photosynthesis (d) Respiration 98. The maximum stratification of plants is found in : (a) Temperate forests (b) Tropical rain forests (c) Tropical shrubby forests (d) Alpine forests 99. Lampbrush chromosomes are seen during: (a) Meiotic metaphase (b) Mitotic metaphase (c) Mitotic prophase (d) Meiotic prophase 100. What plant part is best suited for study of mitosis? (a) Anther (b) Root tip (c) Pieces of bark (d) Shoot apex

66

YCT

SOLUTION : PRACTICE SET- 4 ANSWER KEY 1. (a) 11. (d) 21. (d) 31. (c) 41. (d) 51. (d) 61. (b) 71. (a) 81. (b) 91. (a)

2. (d) 12. (a) 22. (b) 32. (a) 42. (d) 52. (d) 62. (a) 72. (a) 82. (d) 92. (a)

3. (b) 13. (b) 23. (a) 33. (a) 43. (d) 53. (b) 63. (d) 73. (a) 83. (d) 93. (a)

4. (b) 14. (d) 24. (b) 34. (c) 44. (b) 54. (c) 64. (c) 74. (d) 84. (b) 94. (a)

5. (a) 15. (a) 25. (d) 35. (a) 45. (a) 55. (d) 65. (c) 75. (b) 85. (a) 95. (d)

6. (c) 16. (a) 26. (a) 36. (b) 46. (b) 56. (d) 66. (a) 76. (b) 86. (c) 96. (d)

7. (b) 17. (c) 27. (a) 37. (d) 47. (d) 57. (b) 67. (a) 77. (a) 87. (a) 97. (d)

8. (c) 18. (c) 28. (b) 38. (b) 48. (c) 58. (a) 68. (d) 78. (c) 88. (d) 98. (b)

9. (c) 19. (d) 29. (c) 39. (b) 49. (a) 59. (a) 69. (d) 79. (b) 89. (c) 99. (d)

10. (a) 20. (a) 30. (d) 40. (a) 50. (b) 60. (c) 70. (a) 80. (d) 90. (c) 100. (b)

SOLUTION 1. (a) C Electric current is a scalar quantity and does not obey electromagnetic waves v = (where u is the refractive µ the law of vector addition. Rather, Simple algebraic index of the medium) is not the same in all medium. All rules are followed for addition of currents, so both A electromagnetic waves are transverse in nature. and R are correct. 7. (b) 2. (d) Leclanche cell works on direct current and thus gives Electron volt is amount of energy that an electron out direct current. The ratio given here won't work since acquires when accelerated through a potential the transformer work on alternating current not on direct difference of 1 volt. It is obtained by multiplying 1 current. Thus voltage developed in secondary would be electronic charge by 1 volt. E = qv = e ×1 = 1.6 ×10−19 J zero volt. 3. (b) 8. (c) According to the superposition principle, The force dI From the equation, e = −M between any two charged particles is independent of all dt other charges in the universe. dI The force of q1 or q2 is not affected by q3. Note that, e =M however, resultant force on q2 will be affected. dt 9. (c) 20 e = 0.09 × D 0.006 Band width β =   λ d e = 300 volt Where D = Distance between screen and slits 4. (b) d = Distance between slits Adiabatic demagnetization is a technique of liquefaction λ = Wave length of gases in which temperature is reduced. When screen move away D increases then band width Adiabatic demagnetization is a process by which the increases. removal of a magnetic field from certain material serves λ Angular fringe width is βθ = to lower their temperature. d 5. (a) By the formula when screen moved away or near then Work W = M B Cosθ angular fringe width remains constant there is no change in fringe width. Torque τ = MBSinθ 10. (a) τ MB Sinθ = Given data W MB Cosθ ω1 5 = τ = W tan θ ω 3 2 θ = 600 f1 = −15cm τ = W tan 600 N − m f2 = ? τ = 3 W N−m ω1 f =− 1 6. (c) ω2 f2 Electromagnetic waves have the same velocity in any ( −15 ) 5 medium. The velocity of electromagnetic waves is equal =− 8 to the velocity of light 3 × 10 m/sec. But the speed of 3 f2 Practice Set 4

67

YCT

f2 = 9cm (focal length is positive so it is a convex lens)

11. (d) The cardinal point consist the three pairs of points located on the optical axis of a rotationally symmetric, focal, optical system. These are the focal points, the principal points and the nodal points. • Nodal points are points arrange in way where a ray aimed at one of them will be refracted by the lens it appears to have come from the other. • Principal points are the points where the principal planes cross the optical axis. • So a thin lens has 6 cardinal points where as a thick lens has two cardinal points. 12. (a) Rayleigh criterion of just resolution when maximum of one image lies on the minimum of another image and vice versa. To obtain a good image, point sources must be sufficiently far apart that their diffraction patterns do not overlap.

19. (d) ound waves require medium for propagation. It is a mechanical wave and thus needs a medium to travel. It will not travel at all in vaccum. So, the velocity will be zero. Hence, option (d) is correct. 20. (a) Oiled paper allows only some part of the light to pass through it while brick and wood will not. Air is transparent material as it allows light to completely pass through it. 21. (d) Time period of simple pendulum T = 2π ℓ1 = ℓ +

l g

3ℓ 103ℓ = 100 100 ∆ℓ ℓ 1 − ℓ 3 = = ℓ ℓ 100

∆T 1 ∆ℓ %= T 2 ℓ ∆T 1 3 = × T 2 100 ∆T 13. (b) = 1.5% T fo m = 10 = ⇒ f o = 10 f e 22. (b) fe Work-energy theorem : Work done by all forces acting f o + f e = 44 on a particle is equal to the change in kinetic energy of the particle. ⇒ 10 f e + f e = 44 ⇒ f e = 4cm By Newton’s second Law of motion ⇒ f o = 40cm 1 14. (d) v2 = u2 +2aS ⇒ (v2 – u2) = aS Snell gave the law of refraction according to which. 2 sini n 2 Multiplying both side by m = where i → Angle of incidence sinr n1 1 m (v2 – u2) = maS r → Angle of refraction 2 n1 and n2 are the Refractive index of the medium. 1 15. (a) m (v2 – u2) = FS (F = ma) A convex mirror is a diverging mirror as this mirror 2 diverges light rays when they strike its reflecting We know FS = W surface. Virtual, erect and diminished images are 1 always formed with convex mirrors, irrespective of the 2 1 2 W = mv − mu distance between the object and the mirror. 2 2 16. (a) Due to the relative motion between the source and the 23. (a) listener, the difference in frequencies of sound is found. According to Flow of continuity of liquids This effect is called 'Doppler's effect'. Since A1v1 =A2v2 17. (c) πr12 v1 = πr22 v2 When the difference in the frequencies of two sounding 2 2 object is more than 20 Hz, then the beats cease to be 5 3 distinguishable. When two (or more) sounds are present × 0.63 =     v2 having a frequency difference less than about 20 Hz 2 2 then beats can be heard. 25 × 0.63 or v 2 = = 1.75m / s 18. (c) 9 M 24. (b) T = 2π K 4π 2 R 3 M = mass + spring mass/3 T= GM 3 If the height of satellite from earth = h M = 1+ = 2 3 So the time taken to take one round to earth 2 2π ( R + h ) T = 2π T= 1000 v

Practice Set 4

68

YCT

⇒T=

2π ( R + h )  GM / ( R + h ) 

1/ 2

4π ( R + h ) 2

⇒ T2 =

T2 =

GM

=

2π ( R + h )

3/ 2

( GM )

1/ 2

3

, if h < R, then

4π 2 R 3 4π 2 R 3 ⇒T= GM GM

25. (d) Gravity is the reason for many gases present in the atmosphere to remain around the earth. 26. (a) A frame of reference carried by the centre of mass of an isolated system of particle (a system is not subjected to any external force) is called centre of mass frame of reference. In this frame of reference1. Position vector of centre of mass is zero. 2. Velocity and momentum of centre of mass is also zero. So, the option (a) is correct. 27. (a)

p2 2m K1 p12 m 2 p12 × m × 4 = = =1 K 2 p22 m1 p 22 × m p1 1 = p2 2 27. (b) when lift is stationary then, W1 = mg when lift is going down then, W2 = m(g − a) W1 mg 3 = = W2 m(g − a) 2 g 3g − 3a = 2g ⇒ 3a = g ⇒ a = 3 29. (c) Given F = 10 N ↑ S = -5 (down ward direction) Then the work done w = F.S W = 10 × (–5) K=

W = −50J 30. (d) Conservative Force: When the work done by the force is independent of path taken and only depends on the initial and final position. It is called conservation force. The work done by a conservative force on a closed path is zero. Ex-Gravitational force, electrostatic force, magnetic force elastic force, spring force etc. Non- Conservative force: - When the work done by the force depends on the path taken then the force is called non-conservative force. The work done by a non-conservative force on a closed path is non-zero. Ex-Frictional force, non elastic material stress. Practice Set 4

31. (c) CH3COONa → CH3COO− + Na + 2 CH 3 COO −  → → CH 3 COOH + OH − A salt of weak acid and strong base on hydrolysis gives a solution with pH more than 7. 32. (a)

H O

here, C = 0.05 Ka = 1.85×10–5 Kw = 1.1×10–14 Kh.Ka = Kw ⇒ Kh =

kh =

Ch 2 = Ch 2 1− h

Kw Ka

{Q h NaOH > Ca ( OH )2 > Zn ( OH ) 2

Hence, Zn(OH)2 is the weakest base among the given. 61. (b) "Omnis cellula e cellula" was given by Rudolf Virchow in 1855. The complete quote is "Omnis cellula e cellula" he said again and again every cell is born of a previous cell. Life comes from life. This theory is also known as cell lineage theory. 62. (b) Ribosomes are found in both Prokarytoes and Eukaryotes. The ribosome is the site for protein synthesis. Thus, it is also known as protein factory. They are complexes of proteins and three (Prokaryotes) or four (eukaryotes) rRNA molecules called subunits. The main function of ribosomes is the site of mRNA translation; once the two subunits are joined by the mRNA from the nucleus, the ribosome translates mRNA into a specific sequence of amino acids. 63. (d) Endoplasmic reticulum is responsible for the mechanical support, protein synthesis and enzyme transport. → Endoplasmic reticulum (ER) is involved in posttranslational modifications and transportation of proteins, Lipids to their target location. Thus ER stabilizes the cytoplasmic matrix and provides mechanical support to the cell. → Rough endoplasmic reticulum (RER) possesses ribosomes on its outer surface and is actively involved in synthesis of proteins.

Practice Set-6

64. (c) Germ cells in vertebrate gonads are produced by mitosis and meiosis. The cells divides mitotically to produce diploid cells. The diploid cells undergo meiosis to produce haploid gametes. 65. (b) An organism that does not possess a true nucleus (nucleus with nuclear membrane) and membranebounded organelles are called as Prokaryotes. Monerans are Prokaryotes. E.g- bacteria (Mycobacterium), Cyanobacteria (Nostoc and Mycoplasama and Oscillatoria). Chlorella and Chlamydomonas are genera of single - called algae and they are eukaryotic. 66. (b) Phylogeny is the study of relationships among different groups of organisms and their evolutionary development. It attempts to trace the evolutionary history of an organism or all life on the planet by the hereditary characteristics and DNA and protein molecule of them. 67. (d) The characters that shows that man is more close to chimpanzee that to other hominoid apes are sex chromosomes, autosomes and DNA from mitochondria. Ever since researchers sequenced the chimpanzee genome in 2005, they have known that humans share about 99% of DNA with chimpanzees, making them our closest living relatives. 68. (a) Origin of life as a result of chemical evolution has been explained by Miller in detail through his experiment. The Miller – Urey experiment was a chemical experiment that stimulated the condition thought at the time to be present on the early earth and tested the chemical origin of life under those conditions. 69. (a) In 1859, Darwin published his detailed theory in his book titled 'On the Origin of Species' by means of natural selection. The theory of inheritance of acquired character was propounded by Lamarck. Mendel was the father of genetics. 70. (c) Cenozoic Era is called 'Age of Mammals'. It is the third of the major eras of Earth's history, beginning about 66 million years ago and extending to the present. It is marked by global cooling. The extinction of the dinosaurs allowed mammals to diversify and grow in sizes during this era. Mammals are the largest land animals surviving at this time. 71. (a) Chloride ion is essential for the activity of salivary amylase. Na+ ion control heart beat. Fe++ found in the hemoglobin. SO4- ion does not play any role in the body. Salivary amylase or ptyalin catalyzes the hydrolysis of starch into sugars. Chloride is the allosteric effector of salivary amylase. 72. (c) Breathing is controlled by medulla oblongata. It is part of hind brain connected by pons to the midbrain and is continuous posteriorly with spinal cord. Lungs, ribs and trachea involved in respiratory organs and helps in breathing for exhalation and inhalation. 94

YCT

73. (b) Human heart is Myogenic. In myogenic heart, the muscles present in the heart initiates cardiac movement. The human heart's contraction is activated by a specially modified heart muscle known as sinoatrial node (SA node) that is located in the right atrium. SA node is known as the pacemaker because it initiates a wave of contraction and controls the heart beat. 74. (d) The Na+ - K+ exchange pump mainly Occurs in the convoluted tubule of the kidney. This process is active because energy is require for its absorption and movement. By active movement the ions go back into the blood. 75. (d) Vocal cords occur in larynx. → Vocal cords are present in the top area of the larynx. It is also called vocal folds, and are two bands of smooth muscle tissue.The vocal cords vibrate and air passes through the cords from the lungs to produce the sound of voice. 76. (d) A zooplankton feeding on a phytoplankton is both, primary consumer and secondary producer Zooplankton is present on the second trophic level in a food chain the aquatic ecosystem. They are primary consumers as they eat Phytoplankton (Producers) Secondary production in zooplankton is growth, the rate of biomass increase per time. It reflects the net balance between metabolic gains in biomass and the integral of all metabolic losses. 77. (a) The gases associated with green house effect are – CO2, N2O, CFCl3, CH4 and CFCL. for further explanation see the Q. 220 & 210. 78. (b) The Asiatic lion is a population in Gujarat. It is listed in Endangered Red list, because of the small population. Since 2010 the lion population in around Gir National Park only is Asiatic lion. The lion population increased from 411 individuals in 2010 to 523 in 2015 and all of them live in or around Gir National Park. In 2020 the lion population is also increased from 523 to 674. 79. (b) Cadmium pollution is associated with Itai-itai. Itai-itai disease was the name given to the mass cadmium poisoning of Toyama Prefecture, Japan, starting around 1912. Cadmium poisoning can also cause softeing of the bones and kidney failure. The cadmium was released into rivers by mining companies. 80. (a) 2-4-D is a synthetic auxin. Full name of 2.4 - D is the 2, 4- dichlorophenoxyacetic acid and is used as one of the most common herbicides to control broadleaf weeds. It has also good potential to enhance storage life and quality of fruits. Practice Set-6

81. (c) The maximum damage to the forest cover on earth is being done by biotic factor. Based on data collected on forest damage in connection with National Forest Inventory (NFI) approximately 50% of Finland's wood production forest area had suffered damage of varying degrees b/w 2009 & 2013. The most damage is caused by wind, snow, freezing and other climatic factors. 82. (c) Man and Biosphere programme (MAB) was launched by UNESCO in 1971. It is an intergovernmental scientific programme that aims to establish a scientific basis for the improvement of relationship between people and environment. UNESCO-United Nations Educational, Scientific and Cultural Organisation. 83. (b) Plant physiology is a subdiscipline of botany concerned with the functioning, or physiology, of plants. Stephen hales is considered the father of "plant physiology" for the many experiments he conducted in 1727, though Julius von sachs unified the pieces of plant physiology and put them together as a discipline. 84. (a) The first eukaryotic cells appeared on earth about 1400 million years ago. It was also during the proterozoic eon that the first symbiotic relationships between mitochondria (for nearly all eukaryotes) and chloroplasts (for plants and some protists) and their host evolved. The first advanced single-celled, eukaryotes and multicellular life appeared. 85. (c) International Crop Research institute of semiarid Tropics (ICRISAT) is located at Hyderabad (India). It is an international organization which conducts agricultural research for rural development. 86. (a) The science dealing with the relation of plants with different tribal and aboriginal societies is called Ethnobotany. It is the study of how people of a particular culture and region make use of indigenous (native) plants. 87. (c) The main function of golgi bodies is glycosidation of protein. Glycosidation is the reaction of a Saccharides with a hydroxyl or amino functional group to form glycoside. Nucleotide sugar transporters that function together to complete the synthesis of glycans from founding sugars covalently attached to protein or lipid in the endoplasmic reticulum. 88. (a) Epinasty is not an example of incluced nastic movement. The nastic movements are those movements in plants that occurs as the response to a stimulus but do not follow any specific direction of the stimulus. These are direction-independent movements and stimulus position has no effect on their direction. There are following types of nastic movements– 95

YCT

(1) Seismonastic movements 95. (d) (2) Thermonastic movements Nepenthes Khasiana is an endangered tropical pitcher plant of genus Nepenthes. It is the only Nepenthes (3) Nyctinastic movements species Native to India but Nepenthes Khasiana Plant is (4) Thigmonastic movement. a carnivorous plant endemic to the Khasi hills of 89. (d) Ultraviolet (UV) radiation is electromagnetic radiation Meghalaya. This plant is rare and endangered pitcher with a wavelength from 10nm to 400 nm, shorter than plant occur in India. that of visible light but longer than X-rays. UV radiation 96. (c) is present in sunlight constituting about 10% of the total Lysosomes are produced by golgi apparatus Golgi body output of the sun. UV rays is harmful which cause skin is an organelle present in eukaryotic cells. They process the protein and lipid as they are synthesized within the cancer and genetic change in human and plant. cell. Lysosomes are acidic in nature and are responsible 90. (c) Centrioles are absent in plant cells. Centrioles are the for the degradation of large molecules and worn out cell part of centrosome located near the nucleus in the organelles. animal cell. It is involved in the formation of spindle fibre during cell division. Centrioles also maintain the chromosome number during cell division. Plant cells have a cell wall, chloroplasts, and other specialized plastids, and a large central vacuole.

97. (c) The first electron acceptor in Photosystem I of cyclic phosphorylation is Ferredoxin which is a non-haem iron protein indicated by "fd". Photosystem - I (PS-I) is an integral membrane protein complex that absorbs solar energy and mediate electron transfer from plastocyanin to ferredoxin. Ferredoxin receives electrons from PS-I and reduces to NADP+ to form NADPH. 98. (c) Y-shaped model of energy flow in an ecosystem represent both grazing and detritus food chains. It explains how the grazing and detritus food chain connect with each other. It was originally proposed by H.T Odum.

91. (d) Plants utilize nitrogen in the form of nitrate. Plants can absorb nitrate or nitrite from the soil via their root hairs. Nitrogen is vital because it is a major component of chlorophyll. Nitrate moves freely towards plant roots as they absorb water. Nitrates are used by the plants for making amino acids, which are needed to make proteins. • Nitrification is the biological oxidation of ammonia or ammonium to nitrite followed by the oxidation of the nitrite to nitrate. 92. (a) An ecosystem having highest primary producitivity is tropical forest. They are forested landscapes in tropical region, bounded by the tropics of cancer and Capricorn but possibly affected by prevailing winds. 93. (a) The basic repeating structural (functional) unit of chromatin is the nucleosome, which contains eight 99. (d) Cell theory was given by Schleiden and Schwann in histone proteins and about 146 base pairs of DNA. The nucleosome core particle consists about 140 - 160 1839. According to cell theory, living organisms are made up of cells, that they are the basic base pairs (approx. 146 bp). So, the correct answer is structural/organisational unit of all organisms and that '146 bp DNA'. all cell come from pre existing cell. 94. (a) 100. (b) Primary precursor of I.A.A. is tryptophan. Tryptophan During germination of barley seeds the enzyme α(Trp) is an amino acid with an indole ring and is plant amylase is secreted by aleurone layer. It help in seeds growth regulator. nutritional requirement during seed germination. This I.A.A. (indole -3- acetic acid) is the most common layer of cell is sometimes referred to as the peripheral naturally occuring plant hormone of the auxin class. endosperm. Practice Set-6 96 YCT

PRACTICE SET - 7 1.

2.

3.

4.

5.

6.

7.

8.

9.

10.

If the position of a particle y is given by y = 3t3 + t2 + 5, then what is the acceleration at t = 2 ? (a) 5.39 (b) 5.38 (c) 37 (d) 38 Which one does not represent an electromagnetic radiation: (a) α–rays (b) γ–rays (c) x–rays (d) Infrared rays 11. An electromagnetic wave of frequency n in air enters a medium of refractive index µ. The frequency of the wave in the medium will be n (a) nµ (b) µ 12. µ (c) (d) n n The self- inductance of a coil may be measured with the help of : 13. (a) Carey-Foster brige (b) Anderson bridge (c) Meter bridge (d) When's bridge The dipole moment of current loop does not depend upon : (a) Current in the loop 14. (b) Area of the loop (c) Number of turns in the loop (d) Shape of the loop Which of the following is not a source of 15. electric field? (a) Static charge (b) Current (c) Capacitor (d) Changing magnetic field In ordinary bicycle dynamo : (a) Only magnet moves 16. (b) Only armature moves (c) Both magnet and armature moves (d) Both magnet and armature remain at rest, only wheel moves A conductor has a positive charge of 19.2 × 10– 17. 19 Coulombs. How many electrons are in short on the conductor ? (a) 16 (b) 12 (c) 20 (d) 18 Diffraction and refraction indicate (a) Wave nature (b) particle nature 18. (c) both wave and particle nature (d) None of the above Which of the following statements is/are true of the human eye lens? Choose the correct answer from the code given below: (a) It is a convex lens

Practice Set-7

97

(b) it is a concave lens (c) it is a convex lens of variable focal length (d) forms a real and inverted image of a distant object at the retina Code: (a) only (b) (A) and (C) only (c) (A) (C) and (D) only (d) (B) and (C) only Two thin similar lenses of focal length + f each are placed at a distance x apart. For what value of x the equivalent focal length of the combination will be negative (a) x = 2f (b) x 2f The sun appears red at sunrise or sunset. This can be explained on the basis of: (a) Rayleigh scattering (b) Raman effect (c) Compton effect (d) Phonon scattering An achromatic lens is formed from two thin lenses in contact, having powers of +12 and –8 diopters. The focal length of the combination is (a) +25 cm (b) –25 cm (c) +10 cm (d) –15 cm The distance travelled by light in a glass of refractive 1.5 in a nanosecond will be : (a) 50 cm (b) 40 cm (c) 30 cm (d) 20 cm On reflection from a convex surface, the path followed by light is that for which time taken is− (a) Maximum (b) Minimum (c) Neither maximum nor minimum (d) None of these The intensity of sound after passing through a slab decreases by 20%. On passing through two such slabs, the intensity will decrease by: (a) 50% (b) 40% (c) 36% (d) 30% The number of beats per second, resulting from two vibrations y1 = A cos 768 πt and y2 = A cos 764 πt are: (a) 2 (b) 4 (c) 2π (d) 4π The differential equation that describes damped mechanical oscillations, is (a) M

d2x dx + Rm =0 2 dt dt

(b) M

d2x dx + Rm + Sx = 0 2 dt dt YCT

(c)

d2x + Sx = 0 dt 2

d2x S 1 dx + x= M dt dt 2 M Velocity of sound will be highest in (a) Air (b) Water (c) Vacuum (d) Steel A plane wave is represented, in usual symbols, x   by the equation, y = a sin π  200t  . Its 150   frequency is : (a) 2 Hz (b) 100 Hz (c) 150 Hz (d) 200 Hz A body of mass m that is undergoing simple harmonic motion passes through its equilibrium position, its velocity is (a) Zero (b) Maximum (c) Half of its maximum value (d) None of the above A body of mass 2 kg is moving with a constant velocity of 4m/sec on friction less table. The force required to keep the body moving with the same velocity will be – (a) 8 N (b) Zero (c) 12 N (d) 2 N For an incompressible and non-viscous liquid flowing in stream line through a tube of unequal area of cross section. A with a velocity v v (a) = Constant (b) v × A = Constant A v2 (c) = Constant (d) v × A2 = Constant A The gravitational potential energy of the earth is given by : where M, R represent the mass and the radius of the earth and G represents the universal constant of gravitation. 1 GM 2 3 GM 2 (b) − (a) − 5 R 5 R (d) R m

19.

20.

21.

22.

23.

24.

2

27.

28.

29.

30.

31.

32.

33.

34.

2

GM 6 GM (d) − R 5 R Kepler's second law– a real velocity remains constant– is equivalent to which of the 35. following conservation laws : (a) Energy (b) Linear momentum (c) Angular momentum (d) None of the above If the weight of an object of 10 kg is reduced to 36. zero on equator, then the angular speed of the earth will be : (Radius of Earth = 6400 km, g = 10 m/s2)

(c) −

25.

26.

Practice Set-7

1 1 Rad / s (b) Rad / s 100 200 1 1 (c) Rad / s (d) Rad / s 400 800 Potential energy per unit volume u of a strained wire is 1 (a) u = stress × strain (b) u = stress × strain 4 1 3 (c) u = stress × strain (d) u = stress × strain 2 4 One kg-weight is equal to : (a) 9.8 newton (b) 1.0 newton (d) 98 newton (c) 980 newton If the work done by a force in moving a particle from point A to point B does not depend on the path, then the force is of which type? (a) Central (b) Non-conservative (c) Conservative (d) Inverse square law force There will be an increase in potential energy of the system if work is done upon the system by. (a) Any conservation or non-conservative forces (b) A non-conservative force (c) A conservative force (d) None of the above In a buffer solution of CH3COOH and CH3COONa, the ratio of salt to acid concentration is increased 10 times, the pH of the solution should increase by: (a) 10 (b) 7 (c) 4 (d) 1 Gold numbers of protective colloids A, B, C and D are 0.50, 0.01, 0.10 and 0.005 respectively. The correct order of their protective power is(a) D < A < C < D (b) C < B < D < A (c) A < C < B < D (d) B < D < A < C Strongest acid among HCl, HBr & HI is(a) HCl (b) HBr (c) HI (d) None of these A certain compound has the molecular formula X4O6, If 10.0 g of compound contains 5.62 g of X, the atomic mass of X is (a) 32 amu (b) 37 amu (c) 42 amu (d) 48 amu Which one of the following pairs is not correctly matched ? (a) Arrhenius– Electrolytic theory of dissociation (b) Ostwald – Dilution law (c) Onsager – Theory of strong electrolytes (d) Henderson – Hydration energy Which of the following defects lower the density of the crystal? (a) Frenkel defect (b) Schottky defect (c) Substitution defect (d) Interstitial defect (a)

98

YCT

37.

38.

39.

40.

41.

42.

43.

44.

45.

46.

Main oxidation state of Thorium is: (a) +2 (b) +3 (c) +4 (d) +6 The equilibrium pressure of CO2 for the reaction CaCO3 CaO+CO2 is Pe at a particular temperature. CaCO3 is heated in a closed vessel in an atmosphere of CO2 at a pressure higher than Pe. Select the correct answer from the following : (a) CaCO3 decomposes very fast (b) CaCO3 reacts with CO2. (c) CaCO3 will not decompose. (d) None of the above. Aluminium oxide contains 52.9% aluminium and carbon dioxide contains 27.2% carbon. Assuming that the law of reciprocal proportion is true, the percentage of aluminium in aluminium carbide will be (a) 1.123% (b) 0.375% (c) 74.97% (d) None of these Half life period of third order reaction is (a) inversely proportional to square of initial concentration (b) inversely proportional to initial concentration (c) directly proportional to initial concentration (d) independent of initial concentration Which of the metal reacts with water at ordinary temperature (a) Cu (b) Fe (c) Mg (d) Na Which one of the following alloys is not properly matched with regard to its composition? Alloy Composition (a) Brass Cu and Zn (b) Gun metal Cu, Sn and Zn (c) Nichrome Ni, Fe and Cr (d) Solder Pb and Zn The cryoscopic constant of any liquid is(a) The drop in freezing point with disolution of 1gm solute in per kg solvent. (b) The fall in freezing point with the disolution of 1 mole solute per kg of solvent (c) The elevation in boiling point of 1 molar solution (d) The factor brought in use for the calculation of elevation in boiling point Which of the following compounds is chemically inert : (a) SF4 (b) SF2 (c) SF6 (d) XeF2 Magnetic quantum number in 4s orbital is : (a) 2 (b) 4 (c) –1 (d) 0 Which of the following statements is false ? (a) In aqueous solution, sodium is a better reducing agent than lithium. (b) Barium is more electropositive than calcium.

Practice Set-7

47.

48.

49.

50.

51.

52.

53.

99

(c) Chlorine is a stronger oxidant that iodine. (d) The metallic character of bismuth is greater than that of antimony. A sudden lare jump between the values of second and third ionisation energies of an element would be assoicated with which of the following electronic configuration? (a) 1s2, 2s2, 2p6, 3s1 (b) 1s2, 2s2, 2p6, 3s2 (c) 1s2, 2s2, 3p6, 3s2 3p1 (d) 1s2, 2s2, 2p6, 3s2, 3p2 The presence of which one of the following groups on benzene nucleus activates it towards electrophilic substitution? (a) -CN (b) -CHO (c) -COOR (d) -OCOR Which of the following additions will increase percent ionization of CH3COOH ? (a) HCl (b) NaOH (c) CH3COONa (d) NH4Cl A new Carbon-Carbon bond formation is possible in : (a) Clemmensen Reduction (b) Wurtz Reaction (c) Friedel-Craft Reaction (d) Cannizzaro's Reaction Ozone is made up of how many atoms of oxygen? (a) 4 (b) 3 (c) 2 (d) 1 Which one of the following compounds will not give Cannizzaro reaction? (a) C6H5CHO CH3 | (b) CH 3 − C − CHO | CH3 (c) C6H5 – CO – CHO (d) C6H5 – CH2 – CHO Which of the following compound does not react with NaHCO3? OH NO2

O2N (a)

NO2

COOH (b)

YCT

63.

OH NO2 (c)

SO2OH

64.

(d) 54.

55.

56.

57.

58.

59.

60.

61.

62.

Strecker reaction is used for synthesis of : (a) Ketone (b) Amino acid (c) Aldehyde (d) Alcohol In E2 elimination reactions, the two leaving atoms/groups with respect to each other must be (a) The trans and coplanar (b) anti and coplanar (c) only anti (d) only coplanar Which one of the following alcohols does not react with Na2Cr2O7/H2SO4? (a) (CH3)3 C-OH (b) (CH3)2 CH – OH (c) CH3CH2OH (d) All of these Which one of the following compounds will react with Grignard's reagent ? (a) H2C = CH – CH = CH2 (b) H3C – CH2 – C ≡ CH (c) H3C – C ≡ C – CH3 (d) H3C – CH2 – CH = CH2 For the electrophilic substitution in benzene, identify the group which is deactivating but ortho and para directing. (a) – COOH (b) – NO2 (c) –CONH2 (d) – Cl According to Henry's law the solubility of gas in a given volume of liquid increases with increase in: (a) Temperature (b) Pressure (c) Both pressure and temperature (d) Neither temperature nor pressure Nucleoplasm and cytoplasm are separated by : (a) single layered membrane (b) two layered membrane (c) three layered membrane (d) multi-layered membrane The covering membrane of the vacuole is known as : (a) Tonoplast (b) Cell wall (c) Plasma layer (d) none of these The cell organelle which is rich hydrolytic enzyme is called : (a) Nucleus (b) Ribosome (c) Golgi body (d) Lysosome

Practice Set-7

65.

66.

67.

68.

69.

70.

71.

72.

73.

74.

75.

100

During meiosis, Chiasma are observed at pachytene stage. Presence of one Chiasma reduces the possibilities of another occuring in near vicinity. This phenomenon is called. (a) Coupling and repulsion (b) Position effect (c) Cis- trans effect (d) Interference Which of the following is in exception to Cell theory ? (a) Bacteria (b) Protists (c) Viruses (d) Protozoans What is called as "Reverse Genetics" approach? (a) From gene to enzyme (b) From DNA to phenotype (c) From phenotype to DNA (d) From RNA to DNA The concept of 'Genetic drift' was presented by (a) Sewall Wright (b) G.G Simpson (c) Julian Huxley (d) Hardy Weinberg The statement "Nothing in Biology makes sense except in the light of evolution" was given by : (a) Charles Robert Darwin(b) Aldous Huxley (c) T.Dobzhansky (d) H.G.Wells Co-discoverer of Darwinism was : (a) Malthus (b) Wallace (c) Ruskin (d) Robert Dinosaurs disappeared from the earth about – (a) 100 million years ago (b) 200 million years ago (c) 10 million years ago (d) 65 million years ago Pyloric extract contains the hormone : (a) gastrin (b) secretin (c) pancreozymin (d) cholecystokinin Which of the following occurs in mitochondria: (a) Glycolysis (b) Glycogenolysis (c) Tricarboxylic acid cycle (d) Histolysis of tissues The cells secreting histamine – (a) Adipocytes (b) Mast cells (c) Fibrocytes (d) Phagocytes Uric acid is excreted by : (a) pigeon (b) frog (c) rabbit (d) man The D-cells and D2F-cells of pancreas respectively secrete : (a) Pancreatic polypeptide and somatostatin (b) Pancreatic somatostatin and polypeptide (c) Insulin and somatostatin (d) Glucagon and somatostatin The term 'ecology' was introduced by: (a) Haeckel (b) Odum (c) Tansley (d) Ramdeo Mishra YCT

76.

77.

78.

79.

80.

81.

82.

83.

84.

85.

86.

87.

The chemical, that contributes to the destruction of ozone layer above the earth's surface, is : (a) Carbon monoxide (b) Mercury (c) Chlorofluorocarbon (d) Sulphurdioxide A high density of protected animals in a National Park can result in : (a) Emigration (b) Predation (c) Mutualism (d) Intraspecific competition The poisonous symptom of DDT is : (a) Necrosis of liver (b) Haemolysis (c) Albinism (d) All of these Which one of the following is an example of a plant adapted to highly saline coastal condition ? (a) Sonneratia acida (b) Anacardium occidentale (c) Taxus baccata (d) Gnetum gnemon Which of the following is not an insecticide of biological origin : (a) Pyrethrum (b) Bt toxin (c) Malathion (d) Azadirachtin In which year and where latest International Botanical Congress was held ? (a) Paris 2005 (b) Vienna 2005 (c) America 2006 (d) India 2006 The primary receptor of CO2 in C3 plants is : (a) RUBP (b) PGA (c) PEP (d) QAA Origin of life as a result of chemical evolution was properly explained by : (a) Oparin (b) Miller (c) Haeckel (d) Fox Plants with latex and unisexual flowers are found in : (a) Ranunculaceae (b) Rutaceae (c) Amaranthaceae (d) Euphorbiaceae During the past 150 years, the carbon dioxide content in our atmosphere has increased approximately between : (a) 270 to 340 ppm (b) 20 to 35 ppm (c) 0.2 to 0.3 ppm (d) 0.027 to 0.034 ppm Mitosis was discovered by : (a) R. Brown (b) W. Flemming (c) Strasburger (d) A. Weismann The isotopes that have proved useful in photosynthesis research are : (a) 18O and 14O (b) 11C and 32P 14 18 (c) C and O (d) 32P and 18O

Practice Set-7

88.

A transitional zone where two or more different communities meet and integrate is called : (a) Ecotone (b) Ecocline (c) Ecad (d) Niche 89. The cork formed by the activity of phellogen is also known as : (a) Bark (b) Phellem (c) Phelloderm (d) None of above 90. Fat is hydrolysed by enzyme lipase to yield : (a) Fatty acids and amino acids (b) Glycerol and fatty acids (c) Glycerine and water (d) Glycerol and amino acids 91. The most important element associated with protoplasm and proteinaceous materials of plants is : (a) Oxygen (b) Sulphur (c) Potassium (d) Nitrogen 92. Chromosome doubling can be achieved by use of: (a) IAA (b) GA (c) Zeatin (d) Colchicine 93. A flash of light in the dark inhibits flowering in : (a) SDP (b) LDP (c) LSDP (d) DNP 94. Which of the following process is related to soil formation? (a) Laterization (b) Podosolization (c) Gleization (d) All of the above 95. Stalked particles attached to the inner membrane of mitochondria are called : (a) Oxysome (b) mitoribosome (c) granules (d) chondriospheres 96. How much ATP is generated by oxidation of one molecule of FADH2? (a) 1 (b) 1.5 (c) 2 (d) 3 97. Ultraviolet radiation is injurious to plants because it : (a) breaks down carbohydrates (b) increases respiration (c) causes dehydration (d) causes gene mutation 98. Which cell organelle removes hydrogen peroxide? (a) Peroxisome (b) Microsome (c) Spherosome (d) Golgi body 99. The end product of glycolysis is : (a) C2H5OH (b) pyruvic acid (c) CO2 (d) glucose 100. Which of the following ecosystem has least productivity? (a) Tropical rain forests (b) Tropical moist forests (c) Grasslands (d) Oceans

101

YCT

SOLUTION : PRACTICE SET- 7 ANSWER KEY 1. (d) 11. (c) 21. (b) 31. (d) 41. (d) 51. (b) 61. (a) 71. (c) 81. (b) 91. (d)

2. (a) 12. (a) 22. (b) 32. (c) 42. (d) 52. (d) 62. (d) 72. (b) 82. (a) 92. (d)

3. (d) 13. (a) 23. (b) 33. (c) 43. (b) 53. (c) 63. (d) 73. (a) 83. (a) 93. (a)

4. (b) 14. (d) 24. (c) 34. (a) 44. (c) 54. (b) 64. (c) 74. (b) 84. (d) 94. (d)

5. (d) 15. (a) 25. (c) 35. (d) 45. (d) 55. (b) 65. (b) 75. (a) 85. (a) 95. (a)

6. (b) 16. (c) 26. (d) 36. (b) 46. (a) 56. (a) 66. (a) 76. (c) 86. (b) 96. (c)

7. (b) 17. (a) 27. (c) 37. (c) 47. (b) 57. (b) 67. (c) 77. (d) 87. (c) 97. (d)

8. (b) 18. (b) 28. (a) 38. (c) 48. (d) 58. (d) 68. (b) 78. (d) 88. (a) 98. (a)

9. (a) 19. (d) 29. (c) 39. (c) 49. (b) 59. (b) 69. (d) 79. (a) 89. (b) 99. (b)

10. (c) 20. (b) 30. (c) 40. (a) 50. (c) 60. (b) 70. (a) 80. (c) 90. (b) 100. (d)

SOLUTION 7. (b) A dynamo machine consists of a stationary structure y = 3t + t + 5 called the stator, which provides a constant magnetic dy field and a Set of rotating windings called the armature = ( 9t 2 + 2t ) which turns within that field when the wheel rotates. dt 8. (b) d2y = 18t + 2 , t = 2 Total charge = 19.2 × 10-19 coulombs dt 2 Q = ne = 18 × 2 + 2 = 36 + 2 = 38 Where Q = charge, n = number of electron and e = 2. (a) charge on an electron. • Electromagnetic waves are crated as a result of e = 1.6 × 10-19 C vibrations between an electric field and a magnetic −19 Q 19.2 × 10 field. n= = = 12 −19 • EM waves are radio waves, microwaves, infrared, e 1.6 × 10 light, ultraviolet, X- rays , and Gamma rays. n = 12 Hence, α - rays does not represent EM waves. 9. (a) 3. (d) The frequency of electromagnetic waves does not Diffraction and refraction represent the wave nature of change with the change of medium. Because no light whereas Compton effect and photoelectric effect medium is needed for the propagation of show the particle nature of the light. 10. (c) electromagnetic waves. The human eye is a convex lens whose focal length 4. (b) The self-inductance of a coil may be measured with the adjusts automatically which can form inverted and real image of any object on the retina. help of Anderson bridge. 11. (c) • The carey foster bridge is a bridge which is used to The focal distance of the combination lensmonitor medium resistences or slight variations 1 1 1 d between two high resistences . = + − , [ f1 = f 2 ] and (d = x) F f f f • Meter bridge: It is an electrical instrument based on the 1 2 1 f2 principle wheatstone bridge and used to measure a 1 2 x 2f −x = − 2 = unknown resistence. F f f f2 5. (d) Dipole moment is the measure of the separation of f2 ⇒ F = (Since F is – ve) positive and negative electrical charges within a system. 2f −x SI unit of electric dipole moment is Coulomb–meter Therefore x>2f . It depends upon area, number of turn of the loop and not 12. (a) depend upon shape of loop and strength of magnetic field. According to Rayleigh's law, the intensity of the 6. (b) scattering is least because the wavelength of red color is Current flowing in a conductor magnetic field and not highest, due to which the sun appears red during sunrise the electric field because flow of current is neutral and there are no extra charges present. Electric field will be and sunset.  I ∝ 1   s  produced when extra charges are there. λ4   Practice Set-7 102 YCT 1. (d)

3

2



13. (a) Total power of lens P = + 12 + (–8) = + 4 D Focal lenth of combination 100 ∴f = = +25 cm 4 14. (d) c 3 ×108 – v= = = 2 × 108 m / s µ 1.5 ⇒ s = v × t = 2 ×108 ×10−9

s = 0.2m = 20 cm. Fermat principle: When a light ray moves from one fixed point to another fixed point, by any number of reflections or refractions, the total optical path followed by the light ray should be stationary it will either be minimum or maximum. • For reflection and refraction at curved surfaces, the total optical path followed by the light ray should be a maximum. 16. (c) Let the initial value of intensity be 100 dB Remaining intensity = (100 – 20) = 80 dB Again decrease in intensity on passing the second strip 80 × 20 · = 16 100 Therefore Total decrease in Intensity · 20 + 16 = 36dB 36 × 100 = 36% decrease ⇒ 100 17. (a) Ideal equation y = A cos ωt = A cos 2π f t ................(i) •

y1 = A cos 768πt 2π f = 768 π ⇒ f = 384 Hz Let f1 = 384 Hz Again comparing y2 = A cos 764πt from eqn (i) f2 = 382 Hz therefore the no of beats = f1 − f 2 = 384 − 382 = 2 18. (b) The differential equation that describes damped mechanical oscillations, is d2x dx + Rm + Sx = 0 2 dt dt

where M = mass Rm = damping coefficient S = stiffness 19. (d) The velocity of sound is Highest in solids. It depends on density of medium through which it travels. The higher the density of medium the faster is the propagation of sound. Among the given option steel is a solid and has highest density among all.

Practice Set-7

γRT M Where γ = adiabatic index T = temperature (K) M= molecular mass R = Universal gas constant. 20. (b) x   y = a sin π  200t −  150    2t 2x  Compare with y = a sin π  −  T λ  2t 1 200t = ⇒ = f = 100 Hz T T 21. (b) When a body is not under the influence of any additional force, it is in balanced or equilibrium position. While passing through the equilibrium position, the body speed is maximum and acceleration is zero. 22. (b) Given that ν = Constant dν Acceleration (a) = =0 dt ⇒a=0 Force (F) = mass (m) × acceleration (a) = m × 0 = 0 Then force (F) = 0 The answer to the question can be conferred from Newton's first Law. 23. (b) From the principle of continuity for an ideal liquid, A.V = Constant. 24. (c) GMm Gravitational potential energy U = − R where mass of earth = M, Radius of earth = R, −GM 2 U= {m = M} R m is the mass of object on earth Gravitational constant = G The gravitational potential energy of the object is defined as the work done is moving that object from infinite to a particular point in the effect of gravity 25. (c) Kepler's second law is based on the principle of conservation of angular momentum. dA L = dt 2m L = Angular momentum 26. (d) When body is at equator it feels weightlessness then angular speed, V=

15. (a)

M

Sound cannot travel in vacuum since there are no molecules to propagate the waves.

ω=

g R

103

YCT

ω=

539.52 17.52 x ≅ 32

10

x=

3 6400 × 10 1 ω= rad/second 800

27. (c) The potential energy per unit volume 'u' of a strained wire is equal to strain energy stored in the wire. It is given by, 1 u = × stress × strain 2 28. (a) 1 kg weight · 1 × 9.8 = 9.8 Newton [from W = mg ] 29. (c) If the workdone by a force in moving a particle does not depend on the path, then this force is called the conservative force. 30. (c) If work is done on the system by a conservative force, then the potential energy of the system increases. 31. (d) So buffering action is maximum when the ratio of salt to acid is equal to 1. As we know, By Henderson's Equations(Salt ) pH = pKa + log ( Acid ) then pH = pKa + log10 ∴ pH = pKa + 1 32. (c) Gold number of a protective colloid is the minimum mass of it in milligram which must be added to 10 ml of standard red gold. Smaller the gold number, greater is the protective power thus protective power of A, B, C and D are in the order A (0.50) < C (0.10) < B (0.10) < D (0.005) 33. (c) Hydrogen halids how the behaviour of acids in aqueous solutions. Hydrogen halide molecules in aqueous solution on reacting with water, hydronium ions (H3O+ ion) get ionized into halide ions. HCl + H 2 O → H 3 O + Cl – Hydrofluoric acid (HF) is a weak acid in aqueous solution hydrochloric acid (HCl), hydrobromic acid (HBr) and hydrogen iodide (HI) is a strong acid, strength of these acids from (HF) to HI. HI > HBr > HCl > HF 34. (a) X % in log of compound X4O6 compound = mass percentage of x in mole of 5.62 4x × 100 = ×100 10 4x + 16 × 6 (4x + 96) × 5.62 = 40x 22.48x + 539.52 = 40x 539.52 = 40x – 22.48x 539.52= 17.52x Practice Set-7

35. (d) Arrhenius theory of electrolytic dissociation. Ostwald's law of dilution, The theory of strong electrolytes by Onsager but Henderson did not give the theory of hydration but buffer solution. 36. (b) Schottky defect reduces the density of the crystal, this defect was pointed out by German scientist Schottky in 1930. Schottky defect is shown by those crystals in which size of cation and anion is comparable. In this defect a pair of catione and anion vanishes from crystal resulting space behind. Usally crystals having C.N 6 or 8 show theis defect. In this defect density of crystal decreases and crystal remains electrically neutral. example - CsCl, KBr, KCl, AgBr 37. (c) Thorium exhibits an oxidation state in almost all of its compound the Th4+ ion forms many complex ions. Thorium dioxide (ThO2), a very refractory substance has many industrial application thorium electronic configuration [Rn] 6d2 7s2. 38. (c) CaCO3 CaO+CO2 This equilibrium pressure of CO2 at constant temperature is constant and equal to equilibrium pressure. If CaCO3 is heated in a closed vessel in the atmosphere of CO2, there will be no decomposition of CaCO3 : 39. (c) In Al2O3 Percentage of ''Al'' = 52.9% Percentage of 'O' = 100-52.9 = 47.1% In CO2 Percentage of 'C' = 27.27% Percentage of 'O' = 72.73% In Al2O3 Q 47.1 gm O2 reacts with 52.9 gm of ''Al'' with 52.9 ∴ 1 gm O2 is reacts = 1.21 ≈ 1 gram Al 47.1 Therefore in CO2 Q 72.73 gm O2 reacts with 52.9 gm of carbon 27.27 1 gm O2 obtained = 0.37 gm of Carbon 72.73 Therefore 1 gm of oxygen will react with 'Al' and 'C' in ratio 1.12 = 3 : 01 0.32 Part of Al ∴ amount of 'Al' = × 100 Total 3 = × 100 = 75 % 4 Hence option (c) 74.97 % correct.

104

YCT

40. (a) We know that

1 a n −1 Let's the third order n = 3 1 t1/ 2 ∝ 3−1 a 1 t1/ 2 ∝ 2 a The equation a shows hat the Half-life of a third order reaction is inversely proportional to the square of the initia concentration. 41. (d) Sodium belongs to alkali metal alkali metal reacts strongly with water and form their hydroxide and release hydrogen gas at normal temperature 2Na + 2H2O → 2NaOH + H2 (g) Keeping the sodium in the moist -air form a layer of hydroxide on it which fades the shine on the surface of alkali metal, this hydroxide react with CO2 and form sodium carbonate. 2NaOH + CO2 → Na2CO3 + H2 To avoid this reaction, sodium is kept under the kerosene oil. 42. (d) Soldering iron contains Sn (Tin) and Pb (Lead) 43. (b) The cryoscopic constant (molal depression constant) of a liquid is one mole of solute per kilogram of its mass equal the decrease in freezing point due to dissolution. It is expressed by kf each solvent has a certain and the constant molal depression constant is k or the cryoscopic constant is kf. Value of kp in degree per mole or the degree is expressed in units of kilogram per mol (k 1kg mol-). 44. (c) In the molecule of SF6, the six atoms of fluorine protect the sulphur atom from the attack by the reagents to such an extent that even thermodynamically most favorable reactions like hydrolysis do not occur. Therefore SF6 is chemically inert. 45. (d) Magnetic quantum number of 4s orbital n=4 l=0 m=0 1 S= ± 2 The value of magnetic quantum number is zero. 46. (a) Lithium is most powerful reducing agent and sodium is the least powerful reducing agent amongst the alkali metals in aqueous solutions due to greater hydration energy. 47. (b) 1s2, 2s2, 2p6 3s2 which is stable electronic configuration of noble gas Neon (Ne). Octet is completely filled energy is required as this stable electronic configuration is to be broken. t1/ 2 ∝

Practice Set-7

48. (d) Presence of OCOR-Presence of group activates benzene towards electron transfer reaction because ketonic. The lone pair of electrons on oxygen increase the electron density on benzene, there by activating the benzene ring. 49. (b) CH3 COOH CH3 COO− + H +

Na + + OH −

NaOH

CH3 COOH + NaOH

strong base

Weak acid

CH3 COONa + H 2O

H+ and OH − weakly ionize to form H2O the same reason on adding CH3COOH to NaOH the ionisation potential of CH3COOH increases. 50. (c) Carbon-carbon bond forming reaction's are friedel craft's reaction and Reimer- tiemann reaction. CH3

+ CH3Cl

Anhydrous AlCl3

+ HCl

51. (b) Ozone or triple oxygen, is an inorganic molecule with the chemical formula O3. It is a pale blue gas with a distinctively pungent smell. Ozone is an excellent oxidising agent as it breaks down into oxygen gas and hascent oxygen. 52. (d) α | C6 H5 C H 2 CHO contains α-hydrogen atom so it does not give Cannizzaro reaction. 53. (c) OH NO2

Ortho-nitrophenol does not react with NaHCO3 because ortho-nitrophenol is weak acid which does not give out CO2 on reaction with NaHCO3. 54. (b)

The German chemist Adolph Strecker who first used synthesize amino acid from aldehyde and ketone. He used this method to produce alumina from acetaldehyde, ammonia and hydrogen cyanide. 55. (b) In E2 elimination reaction two leaving groups/atoms are anti and coplanar. Where E2 stands for bimolecular elimination involves a one step mechanism in which reaction rate is second order because the E2 mechanism results in the formation

105

YCT

of a 'pi' bond the two leaving groups (often a hydrogen 64. (c) and a halogen) need to be antiperiplanar and coplanar or Cell theory is given by Schleiden and Schwann in 1839. group. The exception of cell theory is the virus. Virus are nonliving crystalline entities which replicate in other living cells. They do not have a cellular structure. 65. (b) Reverse genetics is an experimental molecular genetic technique that enables researchers to elucidate gene 2-Bromo 2-Methyl propane 2-methyl propene function by examining changes to phenotypes (of cells 56. (a) or organisms) caused by genetically engineering specific t-butyl alcohol [(CH3)3C – OH] does not react with nucleic acid sequences (within DNA or RNA). Na2Cr2O7/H2SO4 why butyl alcohol hydrogen is found 66. (a) and it reacts with Na2Cr2O7 on reaction, aldehyde and The concept of 'Genetic Drift' was presented by Sewall carboxylic acid are formed. Wright in 1929. Genetic drift is a mechanism of evolution. It refers to random fluctuations in frequencies CH3 of alleles from generation to generation due to chance Na2Cr2O7/H2SO4 CH3 – C – OH No reaction events. It effects is seen in small populations. 67. (c) CH3 The statement "Nothing in Biology makes sense except in the light of evolution" was given by T. Dobzhansky. 57. (b) Hydrocarbon on reaction with Grignard's reagent OH Theodosius Grygorovych Dobzhansky was a prominent group will get replaced by the R- group of RMgX Ukrainian- American geneticist and evolutionary biologist. He believed that some how mankind would leading to the formation of hydrocarbon. eventually evolve into higher levels of harmony and R Mg X H 3C − CH 2 − C ≡ CH → RX creativity. The above statement is a 1973 essay first Butyne Hydrocarbon published in American Biology Teacher. 58. (d) 68. (b) Halogen (Cl) are electron with drawing group due to Co-discoverer of Darwinism was Wallace. Alfred Russel electronegativity hence they deactivate benzene ring for Wallace was a British naturalist, explorer, geographer, electrophilic substitution but are O, P -directing due to anthropologist, biologist and illustrator. He was a resonance. The correction is (d). scientific contemporary of Darwin and co-developed the 59. (b) theory of natural selection and evolution with him. According to Henry's law the solubility of gas in a given Wallace wrote over 20 books and published more than volume of liquid increases with increase in Pressure. 700 articles and letters on a wide variety of topics. He Increase in solubility ∝ Increase in pressure (Henry's died in 1913 at the age of 90. law) 69. (d) 60. (b) Dinosaurs disappeared from the earth about 65 million Surrounding every nucleus is a double (two) layered year ago. The Mesozoic Era is the age of dinosaurs, that membrane called the nuclear membrane or nuclear lasted from 245 to 65 million years ago. The Era is envelope. This membrane separates the nucleoplasm divided into Triassic period (245-200 million years ago), from the cytoplasm. The nuclear membrane consists of Jurassic period (200-145 million years ago) and two lipid bilayers - the inner nuclear membrane. It also cretaceous period (145-65 million years ago) Dinosaurs has many small holes called nuclear pores that allow went extinct at the end of the cretaceous period. materials to move in and out of the nucleus. 70. (a) 61. (a) Gastrin is a peptide hormone that stimulates secretion A vacuole is covered by a membrane called tonoplast. of gastric acid (HCl) by the parietal cells of the stomach Tonoplast (vacuolar membrane) refers to the cytoplasmic and helps in gastric motility. It is released by G cells in membrane surrounding the large vacuole of a plant cell. the pyloric antrum of the stomach, duodenum, and the Tonoplast separates the vacuolar contents from the pancreas. Its release is stimulated by peptides in the cytoplasmic contents. It is a semi-permeable membrane. lumen of the stomach. 62. (d) 71. (c) Lysosome are also called 'suicidal bags' of cell because Cellular respiration is divided into two part–1. they contain lytic enzyme or destructive enzyme and Glycolysis, 2. Tricarboxylic acid cycle digestive enzymes used for digestion and removal of Glycolysis occur in cell cytoplasm in the absence of O2. excess organelles, and engulfing of viruses and bacteria. Tricarboxylic acid cycle called TCA cycle or krebs Lysosomal enzyme works better under acidic condition, cycle occur in mitochondria. collectively termed as hydrolytic enzyme. 72. (b) The cells secreting histamine is mast cells or mastocyte. 63. (d) During meiosis chiasma are observed at Pachytene Histamine is generated in white blood cells (leukocytes). stage. Presence of one chiasma reduce the possibilities Mast cells are especially numerous at sites of potential of another occuring in near vicinity. This phenomenon is injury like the nose, mouth and feet, internal body surfaces and blood vessels. called Interference.

Practice Set-7

106

YCT

73. (a) In Animal mainly three major nitrogenous wastes emit for example Uric acid, ammonia and Urea. Animal that excrete Uric acid are called Uricotelic animal such as insects, Moth. desert dwellers, snakes birds etc. In these excretory substance in the form of Uric acid and Urates, the body conserves water. 74. (b) The D-cells and D2 F-cells of Pancreas respectively secrete pancreatic somatostatin and polypeptide. The endocrine portion of the pancreas taken from many small clusters of cells called islets of Langerhans or, more simply, islets. Delta cells (D-cells) secrete the hormone somatostatin and polypeptide, which is also produced by a number of other endocrine cells in the body. 75. (a) The term 'ecology' was introduced by Ernst Haeckel. He was a German zoologist, naturalist, eugenicist, philosopher, physician, professor, marine biologist and artist. He was the follower of Charles Darwin. He used the term oekologie to mean "the relationship of the animal to both its organic and its inorganic environments. The word comes from the Greek oikos, meaning "a place to live". ⇒Ecology is the branch of biology that deals with the interrelationships amongst organisms and interaction between organisms and their environment. 76. (c) The chemical that contributes to the destruction of ozone layer above the earth's surface is chlorofluorocarbon (CFCs). Ozone shield is found at the height of 16-45 km in stratosphere of Earth that absorb ultraviolet radiation of sun and protect animals and human. 77. (d) A high density of protected animals in a National Park can result in intraspecific Competition, mean competition within same species of animal. If the number of organisms gets increased, then the competition for food, shelter and other resources will also get increased. 78. (d) The poisonous symptom of DDT is necrosis of liver, haemolysis and albinism. The direct DDT exposure toxic effects in humans include developmental abnormalities, reproductive disease, neurological disease and cancer. DDT is banned for agricultural use in India. DDT has high degree of persistence in the environment. 79. (a) Sonneratia acida is an example of a plant adapted to highly saline coastal condition. This is a halophytic plant. Sonneratia is a genus of plants in the family Lythraceae. Sonneratia species are mangrove trees. It has got intertidal habitats due to its ability to cope with salty environment. 80. (c) Malathion is not an insecticide of biological origin. Malathion is an organophosphate insecticide which act as an acetylcholinesterase inhibitor. It is commonly used to control mosquitoes and a variety of insects that attack fruits, vegetables, landscaping plants and shrubs. Practice Set-7

81. (b) Latest International Botanical Congress was held in Vienna, 2005. 82. (a) The C3 Cycle is a cyclic process of reduction of carbon di-oxide in which the first stable product is a three carbon compound which is 3-Phsphoglyceric arid or 3PGA. In the cycle the carbon di-oxide receptor molecule is RUBP and the enzyme catalyzing this render is RUBP carboxylation. 83. (a) In 1924, Oparin and JBS Haldane explained the theory of origin of life as a result of chemical evolution in different stages of property. In this theory, organic molecules could be formed from abiogenic materials. Chemical evolution is considered the first step in the development of life on Earth. 84. (d) Plants with latex and unisexual flowers are found in Euphorbiaceae family. It is the spurge family, comprises some 7500 species and 275 genera of flowring plants distributed primarily in the tropics. The flowers are typically unisexual and are borne in a characteristic cluster known as cyathia. Many species contain milky latex and some are useful as a source of oil or wax. 85. (a) During the past 150 years, the carbon dioxide content in our atmosphere has increased approximately between 270 to 340 ppm. 86. (b) Mitosis was discovered by W. Flemming in 1882. Mitosis is the common method of cell division in which parent cell divides into two daughter cells with an equal number of chromosomes and thus, called as equational division. It occurs in somatic cells of both, unicellular and multicellular organisms. 87. (c) The isotopes that have proved useful in photosynthesis research are 14C and 18O. C14 labelled carbon dioxide was used by Calvin and Coworkers to trace the path of carbon in dark reaction, whereas water containing O18 was used by Ruben and Kamen to confirm that the source of oxygen released in photosynthesis is water not carbon dioxide. 88. (a) An ecotone is created by the juxtaposition of different habitats or ecosystem. An ecotone is a transition area b/w two biomes. It is transitional zone where two communities meet and integrate. It may be narrow or wide, and it may be local. 89. (b) The cork formed by the activity of phellogen is also known as phellem. In hypodermis or outer cortical cells, a layer becomes meristematic which is known as cork cambium or phellogen. The phellogen also cuts of cell both on its outer side and inner side. The cells on inner side are phelloderm or secondary cortex. The phellem or cork cells are dead and have deposition of a fatty substance called suberin (ie, cork cell are suberized). Suberin is impervious to water and thus cork cells are buoyant (i.e float on water). Phellem, phellogen and phelloderm collectively constitute periderm.

107

YCT

96. (c) 90. (b) Fat is hydrolysed by lipase enzymes to yield Glycerol 2 ATP is generated by oxidation of one molecules of and fatty acid. They hydrolyse the easter bond present FADH2. The 2 ATP are oxidized and formed in 1 in the lipid, which is formed by the acidic group of fatty molecules of FADH2 in glycolysis and photorespiration. acids and hydroxyl group of glycerol. FADH2 produces 2 ATP during electron transport chain Hydrolysed because it gives up its electron to complex II, bypassing Fat → Glycerol + Fatty acids complex I. lipase 97. (d) 91. (d) The most important element associated with protoplasm Ultraviolet radiation is injurious to plants because it and proteinaceous materials of plants is Nitrogen. causes gene mutation. Plants are highly sensitive to UVNitrogen is essential for plants to synthesize amino radiation because of their sessile nature. It damages cell acids, which are the building blocks for protein membrane and all organelles within the cell including chloroplasts, mitochondria and DNA within the nucleus. synthesis. 98. (a) 92. (d) Chromosome doubling can be achieved by use of Peroxisome is membrane bound organelle occurring in Colchicine. As microtubules function in chromosome the cytoplasm of eukaryotic cells. Peroxisome are segregation, Colchicine induces polyploidy by involved in removal of H2O2 (Hydrogen-Peroxide). preventing the segregation of chromosomes during Peroxisomes contain more than 50 enzymes including meiosis that results into half of the gametes containing copious amounts of catalase. Catalase that breaks down double the chromosome number than usual. the very toxic hydrogen peroxide to water and oxygen. 93. (a) A flash of light in the dark inhibits flowering in SDP. SDP refers to short day plants. These plants begin flowering when the days are shorter than their critical length. The require long period of darkness and short period of light to flower. SDP form flowers only when day length is less than about 12 hours, Examples of SDP include rice, soyabean, chrysanthemum, onion, 99. (b) christmas cactus, etc. The end product of glycolysis is pyruvic acid. The 94. (d) process of glycolysis, anaerobically breaks down one Soil is a changing and developing mixture of organic molecule of glucose into two molecules of pyruvic acid. and non-living substances, which cover the land No oxygen is consumed nor any carbon dioxide is surface. This helps in maintaining the vegetation cover. released. The factors of soil formation include basic rock, relief, The final product of glycolysis is pyruvate in aerobic time, climate and biological factors. Soil is formed due settings and lactate in anaerobic conditions. Pyruvate to the process of Laterization, Podsolisation and enters the krebs cycle for further energy production. Gleization. 95. (a) Stalked particles attached to the inner membrane of mitochondria are called oxysomes. They are elementary particles present in the mitochondria. They are helpful in production of ATP molecules. A number of knoblike, stalked structures arising from the inner surface of inner membrane at mitochondria project into the matrix and these are called elementary particles or F1 particles or oxysomes.

100. (d) Oceans have least productivity. Productivity of whole biosphere is about 110 billion tons out of which oceans contribute only 55 billion tons because due to the presence of small autotrophic plants, sunlight not available beyond a depth, photosynthetic capacity of the marine ecosystem is far less than advanced plants. Practice Set-7

108

YCT

PRACTICE SET - 8 1.

2

3.

4.

5.

6.

7.

8.

If velocity - time graph is parallel to time axis, the(a) Body is moving with uniform velocity (b) Its acceleration is zero (c) Its displacement can be found by finding the area of the graph (d) All the above Electromagnetic waves are produced by: (a) a static charge (b) a moving charge (c) an accelerating charge (d) chargeless particle If N is the number of turns in a coil, the value of self inductance varies as (a) N0 (b) N (c) N2 (d) N-2 The critical current which can flow through a super conductor aluminium wire of diameter 1 mm is (critical magnetic field for aliminium is 7.9×103 A/m) (a) 7.9 π A (b) 15.8 π A (c) 15.8 A (d) 7.9 π2 A An electron revolves 6× 1015 round per second in a circular orbit. The current in the loop is (a) 0.96 mA (b) 0.96µA (c) 28.8A (d) none of these The amount of work done in rotating a magnet of moment M through 1800 from its position along the magnetic meridian is: (b) 1/2 MB (a) zero (c) MB (d) 2MB Carbon resistors are available in resistance values ranging from : (a) 1 kΩ to 100 MΩ (b) 1 MΩ to 100 MΩ (c) 1 Ω to 20Ω (d) 1 kΩ to 100 kΩ The ratio of the intensities in the scattered light

11.

(b) ω1f1 + ω2 f 2 = 0 (c) ω1f1 − ω2f 2 = 0 (d) ω1f 2 + ω2f1 = 0 12.

0

9.

10.

(c) frequency of γ − rays is less than that of x-rays 13.

14.

15.

16.

17.

6000 A is (a) 3:2 (b) 9:4 (c) 27:8 (d) 81:16 Rotation of plane polarised light can be measured by : (a) Manometer (b) Galvanometer 18. (c) Polarimeter (d) Viscometer The amount of light entering the eye is controlled by the (a) pupil (b) iris (c) cornea (d) eye lens

Practice Set-8

γ − rays and x-rays both are electromagenetic waves. The differences between them is : (a) X-rays are emitted from nucleus, while γ − rays are the characteristics of atom (b) X-rays are the characteristics of atom and γ − rays are emitted from nucleus. (d) velocity of γ − rays is higher than that of x-rays

0

of two colors of wavelengths 4000 A and

The condition for achromatic doublet of two lenses may be expressed as: (ω1, ω2 and f1, f2 are the respective dispersive powers and focal lengths of two lenses.) (a) ω1ω2 + f1f 2 = 0

109

A man cannot see objects distant more than 40 cm. What is the power of the lens needed to allow him to see very distant objects? (a) – 3.5 D (b) + 3.5 D (c) – 2.5 D (d) + 2.5 D The refractive index of water is 1.33. What will be the speed of light in water (a) 1.33×108 m/s (b) 2.25×108 m/s 8– (c) 3×10 m/s (d) 4×108 m/s For study of ultraviolet light we use (a) plane transmission grating (b) concave grating (c) glass prism (d) none of the above A person hears an explosion 17.7 s after it took place. If the atmospheric–temperature is 140 C, the approximate distance of the place of explosion from the person is: (a) 6.0 km (b) 3.0 km (c) 2.0 km (d) 4.0 km Tuning fork A of frequency 384 Hz gives 6 beats per second when sounded with another tuning fork B. On loading B with a little wax, the number of beats per second becomes 4. Frequency of B is(a) 390Hz (b) 388 Hz (c) 378 Hz (d) 380 Hz Which one of the following instruments work on the principle of damped oscillation? (a) Ammeter (b) Voltmeter (c) Helmhotz galvanometer (d) Ballistic galvanometer YCT

19.

20.

21.

22.

23.

24.

25.

26.

27.

28.

29.

Light waves are : (a) electromagnetic waves in nature (b) cosmic rays (c) infra-red rays (d) in visible in atmosphere In a standing wave of frequency 1000 cycles per second the distance between a node and the adjacent antinode is 10 cm. The velocity of the wave is : (b) 400m/s (a) 200m/s (c) 50m/s (d) 100m/s Which of the following is not essential for the simple harmonic motion? (a) Inertia (b) Restoring force (c) Material medium (d) Gravity A body of mass 0.25kg is falling down with the acceleration 9.2 metre/second2. The friction on the body due to air is : (a) Zero (b) 15 newton (c) 1.5 newton (d) 0.15 newton Viscosity of a gas is due to transport of (a) momentum (b) energy (c) mass (d) none of these The intensity of gravitational field at a point situated inside at a distance x from the centre of the sphere of radius R is proportional to: (a) 1/R (b) 1/R2 (c) x (d) x2 For a comet the minimum and maximum distances from sun are 7 × 107 km and 14 × 109 km respectively. If maximum speed of the comet is 6 × 102 km/sec. its minimum speed will be (in km/s) : (a) 3 (b) 4 (c) 6 (d) 8 If normal force is doubled, then the coefficient of friction will be : (a) unchanged (b) half (c) two times (d) three times A women whose mass is 60 kg on the earth surface is in an spacecraft at an altitude of two times the earth radius. Her mass there is : (a) 6.7kg (b) 15kg (c) 20kg (d) 60kg The Lagrangian for a particle is given 1 • by L = m(r 2 + r 2 θ 2 ) – V(r) Identify the cyclic 2 coordinate. (a) r only (b) θ only

30.

31.

32.

33.

34.

35.

36.

37.

(c) both r and θ (d) rθ A particle is situated on the circumference of 38. the wheel of a moving car. The path of the particle will be : (a) Circular (b) Parabolic (c) Straight line (d) Cycloid

Practice Set-8

Mechanical advantage in a machine where a resistance W is over come by applying an effort P is : W (a) W×P (b) P P (c) (d) W–P W Which one of the following indicators can not be used in the titration of weak base with strong acid: (a) Methyl red (b) Methyl orange (c) Phenophthalein (d) Bromophenol blue Which of the following is the strongest base ? (a) NH3 (b) AsH3 (c) SbH3 (d) BiH3

110

The formation of SO2 in the above figure is according to : (a) Law of reciprocal proportions (b) Law of multiple proportions (c) Law of definite proportions (d) Law of conservation of mass Which of the following is a colloid (a) Muddy water (b) Milk (c) Blood (d) All of these The normal boiling point of a liquid is that temperature at which vapour pressure of the liquid is equal to: (a) Zero (b) 380 mm of Hg (c) 100 mm of Hg (d) 760 mm of Hg Solubility product of : (a) Pbs is greater than ZnS (b) ZnS is more than PbS (c) PbS is equal to ZnS (d) There is no solubility product of either PbS of ZnS The unit of rate constant for a second order reaction is (a) mole–1 litre time–1 (b) mole–1 time–1 (c) mole litre–1 time–1 (d) time–1/s Mixing of two gases gives: (a) true solution (b) supension (c) colloidial solution (d) stable emulsion YCT

39.

40.

41.

For the reaction, 2SO2+O2→ 2SO3, ∆= - 45.2 K Cals The most favourable conditions for the formation of sulphur trioxide would be : (a) high temperature and low pressure (b) high temperature and high pressure (c) low temperature and high pressure (d) low temperature and low pressure In metallurgical processes, the flux used for removing acidic impurities is : (a) Silica (b) Sodium Chloride (c) Limestone (d) Sodium Carbonate Consider the following reaction : Cr(OH)3 + ClO− + OH − → ?+ Cl− + H 2 O The missing ion is (a) Cr2 O72 −

42.

43.

44.

45.

46.

47.

48.

49.

50.

51.

52.

(b) CrO 24 −

(d) Cr2O3 (c) Cr3+ Most common oxidation state of Lanthanide is (a) +2 (b) +3 (c) +4 (d) +5 "No two electrons in an atom can have the same set of all the four quantum numbers" - is 53. the statement of : (a) Pauli's principle (b) Heisenberg's principle (c) Hund's rule (d) Aufbau principle For the reaction, 4NH 3 + 7O 2 6H 2 O( g ) + 4NO 2 , the 54. (g) (g) (g) value of KP is– (a) KC / RT (b) KC. RT (c) KC (RT)2 (d) KC/(RT)2 Neutron was discovered by : (a) Rutherford (b) Langmuir (c) Chadwick (d) Austin The phenomenon in which an electron of higher energy level returns within 10-8 seconds to a lower energy level by radioactive decay is called: (a) Fluorescence 55. (b) Phosphorescence (c) Photosensitisation (d) Light scattering The compound obtained by heating a mixture 56. of a primary amine and chloroform with KOH is (a) An Amide (b) An alkylhalide (c) An amide and Nitrocompound (d) None of the above 57. Which reaction is possible(a) Al + ZnSO 4 (b) Zn + ZnSO 4 (c) Fe + ZnSO 4

Practice Set-8

Graphite has a layered lattice and the layers are joined together by : (a) electrovalent bonds (b) covalent bonds (c) Vander Waals forces (d) metallic bonds Carbylamine reaction is given by : (a) Primary amine (b) Secondary amine (c) Tertiary amine (d) Quaternary ammonium salt Which one of the following compounds gives iodoform test? (a) CH3COOCH3 (b) CH3COCl (c) CH3CHOHCH3 (d) CH3CONH2 Which one of the following compounds will be the most reactive in SN1 reaction ?

(d)

Cu + ZnSO 4 111

Which of the following statements is false for alkali metals? (a) Lithium is the strongest reducing agent (b) Na is a amphoteric in nature (c) Li+ is exceptionally small (d) All alkali metals give blue solution in liquid ammonia Which one of the following statements is not correct ? (a) Mandelic acid on oxidation with strong oxidizing agents gives benzoic acid. (b) Sulphonic acids show Friedel-Crafts reaction. (c) Cinnamic acid on oxidation with chromic acid gives a mixture of benzaldehyde and benzoic acid. (d) Acetylation of salicylic acid with acetyl chloride in presence of pyridine and glacial acetic acid gives aspirin. Mixture of C2H5OH and CH3CHO can be separated by using (a) NaHSO3 (b) NH2OH (c) HCN (d) NH2 – NH2 The treatment of CH3 MgX with CH3C ≡ CH produces

H H CH 3 − C = C − CH3 (c) (d) CH4 When acetaldehyde is heated with Fehling's solution it gives a precipitate of : (a) Cu (b) Cu2O (c) CuO (d) Cu + Cu2O + CuO

YCT

58.

The reactivity of alcohols towards sodium 69. decreases in the following order: °

°

°

°

°

°

(a) 1 > 2 > 3

59.

60.

61.

62.

63.

64.

65.

66.

67.

68.

°

°

°

°

°

°

(b) 3 > 2 > 1

70.

(c) 2 > 1 > 3 (d) 1 > 3 > 2 Which one of the following groups makes the benzene ring more active for electrophilic substitution reactions? (a) –OH (b) –Cl (c) – NO2 (d) – CN An aqueous solution of FeCl3 will be (a) Acidic (b) Neutral (c) Basic (d) Amphoteric Who discovered that all cells arise from preexisting cells by cell division? (a) Rudolf Virchow (b) Robert Brown (c) T. Schwann (d) M. Schleiden Centriole is found in : (a) animal cells (b) red algae (c) prokaryotes (d) flowering plants The ribosome's are composed of proteins and : (a) m-RNA (b) s-RNA (c) t-RNA (d) r-RNA Meiosis and mitosis are unlike each other because in meiosis : (a) Homologous chromosomes pair and exchange parts (b) Chromosome number is halved (c) The four nuclei formed are not identical (d) All are correct Peroxisomes are rich in – (a) DNA (b) RNA (c) Catalytic enzymes (d) Oxidative enzymes The phenomenon of Bradytely is exhibited by which of the following animal? (a) Neoceratodus only (b) Latimeria only (c) Sphenodon only (d) All of the above Ontogeny recapitulates phylogeny is the brief diecription of : (a) Mutation theory (b) Abiogenesis (c) Darwinism (d) Biogenetic law Of the following who started the study of Geographical distribution of animals? (a) P.L. Sclater (b) A.R. Wallace (c) C. Darwin (d) H. Spencer

Practice Set-8

71.

72.

73.

74.

75.

76.

77.

78.

79.

80.

112

The book 'Origin of Species' was published in : (a) 1809 (b) 1858 (c) 1859 (d) 1956 First amphibian appeared during the period : (a) Permian (b) Carboniferous (c) Devonian (d) Silurian Diastema is : (a) type of tooth (b) a part of tooth (c) a structure in eye (d) a gap between teeth The source of oxygen in glucose formed in photosynthesis is : (a) Water (b) Water and carbon dioxide (c) Air (d) Carbon dioxide Intercalated disc are found in : (a) visceral muscle (b) skeletal muscle (c) cardiac muscle (d) All of the above Kidney tubules originate from : (a) Ectoderm (b) Mesoderm (c) Endoderm (d) None of the above Steroidogenesis takes place in : (a) Sertoli cells (b) Leydig cells (c) Germ cells (d) none of these Biotic potential means : (a) carrying capacity of the medium (b) capacity of individuals to migrate in and out of the population (c) growth curves of population (d) reproductive potential of individuals In stratosphere, the concentration of which of the following ozone destroying free radicals, has greatly increased as a result of human activity? (a) Only HO(b) Only NO(c) Both Cl- and Br(d) Both NO- and OHIn India wildlife conservation board was established in : (a) 1887 (b) 1952 (c) 1973 (d) 1985 PAN is : (a) Peroxyamyl nitrate (b) Polyoxyamyl nitrate (c) Peroxyacetyl nitrate (d) Polyoxyacetyl nitrate Which of the following plant is capable of absorbing CO2 and NO2 : (a) Robinia pseudoacacia (b) Phaseolus vulgaris (c) Daucus carota (d) Coleus indica YCT

81.

82.

83.

84.

85.

86.

87.

88.

89.

90.

Arboreal animal are more common in : (a) Tropical rain forests (b) Temperate forests (c) Alpine forests (d) Coniferous forests The major constituent of the cell organelle centriole is : (a) Actin (b) Myosin (c) Tubulin (d) Intermediate Filament Prochlorophytes possess : (a) Chl. a only (b) Chl. b only (c) Chl. a and b both (d) Chl. c Which of the following has lowest number of chromosomes n = 2 : (a) Solanum nigrum (b) Haplopappus gracilis (c) Poa annua (d) Argemone Mexicana Which one of the following is a natural cytokinin? (a) Indole-3-Acetic Acid (b) Zeatin (c) Kinetin (d) Indole-3-Butyric Acid Which one of the following is an endemic plant? (a) Nepenthes Khasiana (b) Cynodon dactylon (c) Riccia discolor (d) Vernonia cinerea The prokaryotic genetic system contains : (a) DNA and histones (b) DNA or histones (c) DNA without histones (d) Neither DNA nor histones Which of the following Kreb's cycle enzyme is localized in the inner mitochondrial membrane? (a) Isocitric dehydrogenase (b) Fumarase (c) Malate dehydrogenase (d) Succinic dehydrogenase Ecotone is a : (a) core region of a forest (b) type of grassland (c) open area of a lake (d) transition zone between two distinct plant communities The longest phase of meiosis is : (a) Prophase-I (b) Prophase-II (c) Metaphase-I (d) Metaphase-II

Practice Set-8

91.

The number of RNA molecules in 60s subparticles of 80s ribosomes are : (a) Five (b) Four (c) Three (d) Two 92. Which one is an example of seasonal ecotype : (a) Xanthium strumarium (b) Tectona grandis (c) Melia azedarach (d) Cassia siamea 93. Mitosis and meiosis always differ in regard to the presence of : (a) Chromatids (b) Homologous chromosome (c) Bivalents (d) Centromeres 94. O3 is a pollutant gas in : (a) Tropopause (b) Lower stratosphere (c) Troposphere (d) Mesosphere 95. In the photosynthetic electron transport system, the hydrogen transporter is : (a) Pheophytin (b) Plastoquinone (c) Cytochrome (d) Plastocyanin 96. Electron microscope was invented by : (a) H. Rohrer (b) G. Binnig (c) Antonie Van Leeuwenhoek (d) Max Knoll and Ernst Ruska 97. Pneumatophores are commonly found in : (a) Tinospora cordifolia (b) Vanda roxburghii (c) Ipomoea batatus (d) Rhizophora mucronata 98. A plant hormone promoting femaleness is : (a) Gibberellin (b) Abscisic acid (c) Auxin (d) None of the above 99. Which one of the following plants is an example of short-day plant? (a) Mirabilis jalapa (b) Beta vulgaris (c) Xanthium Strumarium (d) Lycopersicum esculentum 100. 'Green house effect' with respect to global climate refers to : (a) Cool and moist condition (b) Warming effect (c) Increased rain fall (d) Desertification

113

YCT

SOLUTION : PRACTICE SET- 8 ANSWER KEY 1. (d) 11. (d) 21. (d) 31. (c) 41. (a) 51. (c) 61. (a) 71. (d) 81. (a) 91. (c)

2. (c) 12. (b) 22. (d) 32. (a) 42. (b) 52. (c) 62. (a) 72. (d) 82. (c) 92. (d)

3. (c) 13. (c) 23. (a) 33. (a) 43. (a) 53. (b) 63. (d) 73. (c) 83. (c) 93. (c)

4. (a) 14. (b) 24. (c) 34. (d) 44. (a) 54. (b) 64. (d) 74. (b) 84. (b) 94. (c)

5. (a) 15. (b) 25. (a) 35. (d) 45. (c) 55. (a) 65. (d) 75. (b) 85. (b) 95. (b)

6. (d) 16. (a) 26. (a) 36. (b) 46. (a) 56. (d) 66. (d) 76. (d) 86. (a) 96. (d)

7. (c) 17. (a) 27. (d) 37. (a) 47. (d) 57. (b) 67. (d) 77. (c) 87. (d) 97. (d)

8. (d) 18. (d) 28. (b) 38. (a) 48. (a) 58. (a) 68. (b) 78. (b) 88. (d) 98. (c)

9. (c) 19. (a) 29. (d) 39. (c) 49. (c) 59. (a) 69. (c) 79. (c) 89. (d) 99. (c)

10. (a) 20. (b) 30. (b) 40. (c) 50. (a) 60. (a) 70. (c) 80. (b) 90. (a) 100.(b)

SOLUTION 1. (d) When an object moves with a constant velocity, its acceleration is zero. The area covered by the velocity-time graph is equal to the displacement of the object. 2. (c) Electromagnetic waves are produced whenever electric charges are accelerated. This makes it possible to produce electromagnetic waves by letting an alternating current flow through a wire an antena 3. (c) The expression for the self inductance of any coil µ πr L = o .N 2 2 L ∝ N2 where N →Number of turns in a coil. 4. (a) diameter of wire d = 1 mm = 1×10–3m radius of wire = 0.5×10–3m Magnetic field H = 7.9×103A/m Critical current IC = 2πrH = 2×3.14×0.5×10–3×7.9×103 IC = 7.9 πA 5. (a) i = f .e = 6 × 1015 × 1.6 × 10−19

compared to carbon resistance. The carbon resistance draws the power of 1 watt to 1/8 watt. Range of carbon resistance 1Ω to 20Ω 8. (d) Since, Intensity of light 1 I∝ 4 λ 4

so,

4

I1 λ 2 4  6000  81 3 = =  =  = I 2 λ14  4000  2 16  

9. (c) Polarimeter used for measuring optical rotation. It consists of two Nicol prisms, one called the polarizer (neat the eye) and the other is known as an analyzer. In between the polarizer and analyzer a glass tube containing the solution of an optically active compound is placed. So option (c) is correct. 10. (a) The amount of light entering the eye is controlled by the pupil. The iris contracts and relaxes to regulate the size of the pupil. When it is dark, the pupil expands to allows more light to enter the eye. When it is bright, the pupil contracts to prevent more light from entering the eye. 11. (d) For the achromatic doublet (ω1, ω2 and f1, f2 are the respective dispersive powers and focal lengths of two lenses.) ω1 ω =− 2 f1 f2

= 9.6 × 10−4 ⇒ 0.96 × 10 −3 ampere i = 0.96 mili ampere. 6. (d) Work done, W = MB (cos θ1 – cos θ2) ⇒ ω1f 2 = −ω2f1 here θ1 = 0o and θ2 = 1800 ⇒ ω1f 2 + ω2 f1 = 0 ⇒ W = MB (cos 0 – cos 180) 12. (b) = MB {1 – (–1)} = 2 MB X- rays are the characteristics of atom and γ − rays 7. (c) Many material like Nichrome, Brass, Platinum and emitted from nucleus. Tungsten are metal used to produce resistance. But most The atomic number of the largest must be high for Xof these materials have very low electrical resistance as rays.

Practice Set-8

114

YCT

13. (c) The lens should be given to the person which forms the image of an object at infinity at distance 40 cm in front of the eye. 1 1 1 = − f v u 1 1 1 =− − f 40 −∞ f = – 40 100 100 Power, P = =− = −2.5D f 40 14. (b) The speed of light in medium. speed of light in vacuum ( C ) Refractive index (n) = speed of light in medium ( v )

v=

C 3 × 108 = = 2.25 × 108 m / s n 1.33

15. (b) For study of ultraviolet light we use concave grating. The main advantage of a concave grating is that it can be used as the primary dispersive and focusing element in a instrument. The concave grating reduces the number of optical elements required, increasing throughput and instrument efficiency. 16. (a) Speed of the sound at atmospheric pressure and temperature tºC v t = 332 + 0.6 × t

CXDR the galvanometer is said to be critically damped. With more resistance it is under damped and with less it is over damped. When the galvanometer is critically damped it will make one swing and return slowly to its zero position. 19. (a) Light waves are electromagnetic waves and they do not require a medium to travel. It can easily travels in any medium or vacuum. • It includes the wavelength in the range of 380 nm to 740 nm. • The electromagnetic radiation (visible light) propagates by mass less elementary particles called photons that represents the quanta of electromagnetic field, and can be analyzed as both waves and particles. 20. (b) Nodes and antinodes are formed in stationary waves and in a stationary wave the distance between two successive node is one half of wave length. Therefore the distance between node and antinode is one fourth of a wave length. ∴λ = 10 × 4 = 40 cm ∴The velocity of wave v = nλ = 1000 × 40 = 40,000 cm/s = 400 m/s 21. (d) we know that, Fnet = Fspring

According to question, t = 14 0C

F = − ky

(k is spring constt)

v t = 332 + 0.61× 14

ma = − ky

(Q F = ma)

= 340.5 m / s Therefore Distance = speed of sound × time 340.5 ×17.7 6 km

k a = −  y m

k  2 Q ω =  m 

a = −ω2 y Therefore, for a body doing simple harmonic motion, inertia, restoring force and physical medium are necessary, whereas gravity is not necessary. 22. (d) Force, F = m (g – a) a = 9.2 m/sec2 g = 9.8 m/sec2 = 0.25(9.8 − 9.2) = 0.25 × 0.6 = 0.150 Newton

17. (a) Given, nA = 384 No of Beats = 6 ⇒ ± 6 = nB – nA ⇒ ± 6 = nB – 384 ⇒ nB = 384 ± 6 So possible values of frequency of tuning fork B = 390 Hz or 378 Hz After applying wax, nB = 384 ± 4 23. (a) Possible values = 388 or 380 The term Viscosity represents the internal resistance ∴ On applying wax frequency decreases so 378Hz can't offered by one layer of fluid to the adjacent layer in a become 388 or 380 after applying wax. flow. In case of Gases the main reason of viscosity is So frequency of B = 390 Hz. molecular collision or molecular momentum transfer. 18. (d) Hence the viscosity of gases increases with increase in Damping of Ballistic Galvanometer: temperature. A ballistic galvanometer will oscillate if it has not In liquid viscosity is due to intermolecular bonding and properly damped. Galvanometers are damped by adding it decreases with increase in temperature. a shunt of right amount of resistor in parallel with them. 24. (c) The proper amount of resistance at which the motion just ceases to be oscillatory is called critical external The intensity of the gravitational field over the solid damping resistance (CXDR). When shunted by its sphere Practice Set-8 115 YCT

I=

GM x 3

so I ∝ x

R Other terms are fixed (M = mass of the sphere) (R = Radius of the sphere) 25. (a) The conservation of angular momentum J=I×ω if I = constant then J ∝ ω Q v = rω v J∝ r v max v min we get = rmax rmin then

v min =

6 × 102 × 7 × 107 14 × 109

= 3 km/s

26. (a) The coefficient of friction is a property of the two surfaces in contact. It we double the normal force, we double the amount of friction, but the coefficient of friction is the same, since the nature of the two surfaces in contact has not changed. 27. (d) The mass of the woman on the earth is m = 60 kg and the radius is twice the radius of the earth that is R = 2d. we know that the mass of an object remains so the same in every situation, whether it is on the earth or in the therefore its mass will be 60 kg. 28. (b) The Lagrangian for a particle is given 1 •2 by L = m(r + r 2 θ 2 ) – V(r) 2 The relative motion only depends on magnitude of the seperation and θ is a cyclic co-ordinate with the conserved angular momentum. 29. (d) The path of the particle will be cycloid because the moving particle gets the inertia that is necessary to make it cycloid. 30. (b) Mechanical Advantage – In a simple machine when the effort (P) balances a load (W) the ratio of the load to the effort is called mechanical Advantage. Mechanical advantage in a machine. W load = = effort P 31. (c) Phenolphthalein has acidic nature so it can not be used as indicator in (weak base + acid) titration due to weak base and strong acid solution is acidic nature.

Practice Set-8

32. (a) The hydride of nitrogen family contain one lone pair on the central atom. as we move down the group the size of the central atom increases, there by their orbital become more diffused, hence the nucleophilicity of the lone pair is less than over bigger atom, there by decreasing the basicity, the most basic hydride is NH3. basicity in the decresing order NH3>PH3>AsH3>SbH3>BiH3 as we move down the group, the stability of hydride decreases so NH3 is most stable. 33. (a) It explains the law of reciprocal proportions, suppose that the three elements are hydrogen, Sulphur and oxygen hydrogen combines with oxygen to form water H2O while sulphur react with oxygen to form sulphuric oxide, Hydrogen and sulphur also react with each other to form hydrogen sulphide. 34. (d) The substances whose solution do not diffuse through the parchment membrane or diffuse very slowly are called colloids. Blood is a colloidal solution in which the size of the solute particles is between 1 to 200 nm but the size of the solute of the solvent is smaller (10–7 – 10–8) cm than the particles per gram of solute, due to the large size of the particles on the kelvin of solute. It will be a soft diffused bun from the parchment. Muddy water is a classic suspensin, with relatively large solid particles suspended in water. Milk is an emulsion colloid in which both dispersion and dispersed medium are liquid. 35. (d) Normal boiling point of a liquid is the temperature at which the vapour pressure is equal to 760 mm of Hg. 36. (b) The solubility product of PbS is less than that of ZnS it happens. Solubility product of PbS at 25°C = 8.4 × 10 –28 Solubility product of ZnS at 25°C = 1.1 × 10 –21 37. (a) Unit of rate constant for a second order reaction is mole1-n litren-1 time-1 Second order of reaction n = 2 ∴ mole1–2 litre2–1 time–1 mole–1 litre time–1 38. (a) A colloid cannot be produced by mixing of two gases because two gases can mix in any proportion to from homogenous solution. 39. (c) 2SO2+O2 → 2SO3, ∆= - 45.2 K Cals This reaction is an exothermic reaction in which the reaction will move forward on lowering the temperature of the reaction while the volume of sulphur trioxide is increasing from three to two of course the volume is less. Which means the pressure is increasing, which means that at low temperature and high pressure the reaction will move forward.

116

YCT

Since secondary and tertiary amines (aliphatic or 40. (c) Flux in metallurgy, any subsance introduced in the aromatic) do not give this reaction. It is used as a test smelting of ores to promote fluidity and to remove for primary amines and also for this distinction of primary amine from secondary and tertiary amines. objectionable impurities in the form of slag. In metallurgical process, the flux used for removing C2 H 5 NH 2 + CHCl3 + 3KOH → C2 H 5 NC + 3KCl + 3H 2O acidic impurities is lime stone. Ethyl isocyanide Other materials used as fluxes are silica, dolonite, lime, 48. (a) borax and fluorite. More reactive element replace the less reactive element 41. (a) from their solution. By balancing of reaction Aluminum (Al) is above in reactive series than zinc 2Cr(OH)3 + 3ClO − + 2OH − → Cr2 O 7 −2 + 3Cl− + 4H 2 O (Zn) so it easily replaces zinc from ZnSO4. K > Na > Ca > Mg > Al > Zn > Fe > Pb > Cu 42. (b) 2Al + 3ZnSO 4 → Al2 (SO 4 )3 + 3Zn The common oxidation state of the element of the Rest all reaction is not possible because they are below lanthanide series is '+3'. The principle and common oxidation state '+3', some element also show +2 and +4 the Zn in reactivity series so less reactive than Zn. So Al + ZnSO4 reaction is possible a balanced equation is oxidation states. mention above. 43. (a) The Pauli exclusion principal states that in a single 49. (c) atom in two electrons will have an identical set or the Graphite has a layered structure held by Vander waals forces and the distance between the two layers is 340 same quantum numbers (n, l, me and ms). layer is composed of planar hexagonal rings There are two salient rules that the Pauli exclusion nm. Each 2 of sp hybridsed carbon atoms. Each carbon atom in a principle follows hexagonal ring makes three sigma bonds with three - Only two electrons can occupy the same orbital. neighbouring carbon atoms and the fourth electron - The two electrons that are present in the same orbital forms a pi (π) -bond. must have opposite spins or they should be antiparallel. Graphite cleaves easily between the layers, hence it is 44. (a) very soft and slippery. We know that ∆n 50. (a) KP = KC (RT) Carbylamine reaction:- When aliphatic/ aromatic ∆n = Product – Reactant primary amines are heated with chloroform (CHCl3) 6H2O + 4NO2(g) 4NH3 + 7O2 and alc. KOH foul-smelling alkyl isocyanides or ∆n = 10–11 carbylamine are obtained, secondary or tertiary amines ∆n = –1 do not give this test. then, CH3–NH2+CHCl3+Alc KOH→CH3– N ≡ C + KCl + H2 KP = KC (RT)–1 Methyl Chloroforme Methyl K amine isocyanide KP = C RT 51. (c) 45. (c) Iodoform test gives only –COCH3 containing group The British physicist sir James Chadwick discovered compounds and secondary alcohol. 'neutrons' in the year 1932 he was awarded the Nobel So CH3CHOHCH3 is secondary alcohol then Iodoform prize in physics in the year 1935 for this discovery. test is given by CH3CHOHCH3. James Chadwick fired alpha radiation at beryllium sheet 52. (c) form a polonium source. As iodine is the best- leaving group, so C is most 46. (a) When a fluorescence substance absorbs energy due to reactive it is resonance stabilized. the incidence of X-ray, light waves and electrons, it starts to emit visible light (longer wavelength) and slowly reaches the group state from the Partially excited state within 10-8 seconds. This phenomenon is called 53. (b) fluorescence. Na is amphoteric in nature this statement is false. In 47. (d) Carbylamine reaction -: When a primary amine chemistry amphoteric is a molecule or ion that can (aliphatic or aromatic) is warmed with chloroform and react both with an acid as well as a base but it only alcoholic KOH, it form an isocyanide or carbylamine react with acid to give hydrogen gas it does not react having offensive smell this reaction is called with base. carbylamine reaction. 2Na + 2HCl → 2NaCl + H2

Practice Set-8

117

YCT

So, the aqueous solution of FeCl3 will be acidic in 54. (b) Sulphonic acids does not Friedel-Crafts reaction nature. because this reaction is an organic coupling reaction 61. (a) involving an electrophilic aromatic substitution that is Rudolf Virchow discovered that all cells arise from preused for the attachment of substituents to aromatic existing cells by cell divison. The classical cell theory rings. was proposed by Schwann and Schleiden in 1839. The 55. (a) first part of this theory states that all organisms are A mixture of C2H5OH is separated by NaHSO3 (sodium made of cell. The second part states that cells are the hydrogen sulphide) soluble in water, sodium hydrogen basic units of life. The third part, which asserts that cells sulphide react with aldehyde to give an addition come from Preexisting cells that have multiplied, was compound, this is a reversible reaction. described by Rudolf Virchow in 1858 OSO H OSO H CH CH CH 62. (a) C C C = O + NaHSO OH ONa Centrioles are paired barrel-shaped organelles located H H H in the cytoplasm of animal cells near the nuclear 56. (d) envelope. It plays a role in organizing microtubules that CH3 – C ≡ C – H + CH3 – MgX → CH4 + CH3 – C serve as the cells skeletal system, and help determine ≡ C – Mgx the location of the nucleus and other organelles within When propyne (prop-l-yne) is reacted with Grignard the cell. reagent, an alkane (CH4) is formed. CH3 has a negative 63. (d) ion and 4C- attracts the proton ion towords it and form The chemical composition of ribosome is protein + (CH4). rRNA. Polyribosome is group of ribosome's associated 57. (b) with a messenger RNA molecule, involved in peptide Acetaldehyde is heated with Fehling's solution then synthesis. Protein + rRNA + mRNA = Polyribosome reducing the Fehling's solution and gives a red coloured 64. (d) precipitate of Cu2O. Fehling Sol. Meiosis is the type of cell division in which → RCOOH + Cu 2 O R − CHO  homologous chromosomes separate after an exchange of Acetaldehyde Carboxylic fragments. As a result of meiosis four haploid cell are acid produced which are all genetically different from each Fehling's solution is a tartaric acid complex of cupric ions. Acetaldehyde is oxidised to acetate ion and cupric other. This is due to the process of Crossing over or exchange ion reduced to cuprous oxide. of segments between homologous chromosomes. 58. (a) The reactivity of alcohol towards sodium metal follows 65. (d) the same sequence, the order in which the acidity of Peroxisomes are rich in oxidative enzymes. It is a alcohols occurs, the order of acidity of alcohol membrane bound organelle occuring in the cytoplasm (Primary) > Secondry > tertiary alcohol against sodium of eukaryotic cells. They contain enzymes that oxidize reactivity. certain molecules normally found in the cell, notably 1° > 2° > 3° fatty acids and amino acids. Those oxidation reactions produce hydrogen peroxide, which is the basis of the 59. (a) Hydroxyl group (–OH) makes the aromatic ring more name peroxisome. Its main oxidative enzymes are reactive compared to benzene, they are. activating catalase and urate oxidase. group on the other hand Chlorobenzene, nitrobenzene 66. (d) and cyanobenzene react more slowly than benzene. The The phenomenon of bradytely is exhibited by chloro and nitro groups are deactivating group because Neoceratodus, Latimeria and Sphenodon. Bradytely they make the aromatic ring less reactive. means arrested evolution or evolution at very slow 60. (a) rates, outside the rate distribution usual for a given FeCl3 is a salt of strong acid (HCl) and weak base. It group of plants or animals. The concept was introduced gives Fe (OH)3 and HCl on hydrolysis, Fe (OH)3 is a by the American zoologist and paleontologist G.G. weak base and HCl is strong acid. Simpson (1944). Living fossils exhibit this phenomenon − over geologically long time scales. H2O ↽ ⇀ H+ + OH 3+ − 67. (d) FeCl3 ↽ ⇀ Fe + 3Cl Ontogeny recapitulates phylogeny is the brief − → Fe (OH)3 diecription of Biogenetic law. Ontogeny recapitulates Fe3+ + 3OH  phylogeny, given by Ernst Haeckel. It states that an H+ + Cl − ↽ ⇀ HCl organism repeats it's evolutionary history during embryonic development. FeCl3 + 3H2O → Fe (OH)3 + 3HCl 3

3

2

3

2

3

Practice Set-8

118

YCT

68. (b) Once proposed by Alfred Russel Wallace, known to be the father of Zoogeography, phylogenetic affinities can be quantified among zoogeographic regions, further elucidating the phenomena surrounding geographic distributions of organisms and explaining evolutionary relationships of taxa. Alfred Russel Wallace OM FRS, was a british naturalist, explorer, geographer, anthropologist, biologist and illustrator. 69. (c) The book 'Origin of Species' written by Darwin was published in 1859. In this book with the help of data collected on animals in many islands located in the Atlantic and Pacific Oceans, Darwin has made a successful attempt to prove the origin of the species. 70. (c) The earliest amphibians evolved in the Devonian period from Sarcopterygian fish with lungs and bonylimbed fins, features that were helpful in adapting to dry land. The earliest amphibian discovered to data is Elginerpeton, found in late Devonian rocks of scotland dating to approximately 368 million years ago. 71. (d) A diastema (plural diastemata) is a space or gap between two teeth. Many species of mammals have diastemata as a normal feature, most commonly between the incisors and molars. Diastemata are common for children and can exist as well in adult teeth. 72. (d) Photosynthesis is an oxidation reduction process in which water is oxidized to form oxygen (O2) and carbon dioxide (CO2) is reduced to form sugar (glucose). The oxygen in glucose comes from carbon dioxidelight 6CO 2 + 12H 2 O  → C6 H12 O 6 + 6H 2 O + 6O 2 ↑ Chlorophyll, Enzymes

or interstitial cells are located in the connective tissue surrounding the seminiferous tubules. Adrenal cortex, testes, ovaries and placenta synthesize steroid hormones. 76. (d) Biotic potential means reproductive potential of individual. Biotic potential, the maximum reproductive capacity of an organism under optimum environmental conditions. Biotic potential depends on the frequency of reproduction as well as the number of offspring produced. 77. (c) In stratosphere, the concentration of both Cl– and Br–, ozone destroying free radicals has greatly increased as a result of human activity. In the atmosphere HO–, NO, Cl–, Br–, NO– etc. are found as ozone free radicals. 78. (b) Wildlife conservation board was established in 1952. Due to the rapid decline in wildlife population, the government of India had constituted an advisory body designated as the Indian Board for wildlife (IBWL). The IBWL was chaired by the Prime Minister. 79. (c) The full form of the PAN is Peroxyacetyl Nitrates. It is also called Acetyl peroxy nitrates (PAN). It is a toxic chemical that is important component of smog. It is a gas at normal temperatures and pressure. Its chemical formula is C2H3O5N. 80. (b) Phaseolus vulgaris is a plant capable of absorbing CO2 and NO2. Phaseolus vulgaris is a bean plant. Common beans acquire the nitrogen through an association with rhizobia, which are nitrogen-fixing bacteria. 81. (a) Tropical rain forests are rain forest that occurs in area of tropical rainforest climate in which there is no dry season. Arboreal animal are more common in Tropical rain forests. Arboreal animals live in the trees. 82. (c) The major constituent of the cell organelle centriole is tubulin. Tubulin is a protein that can polymerize into long chains or filaments that form microtubules. Most centrioles are made up of nine circularly arranged triplet microtubules. Centrioles are present in animal cells and the basal region of cilia and flagella in animals and lower plants. 83. (c) Prochlorophytes have both chlorophyll a and b but they do not have phycobilins. This feature separated them from cyanobacteria. Prochlorophytes can be unicellular or filamentous and depending on the filamentous species, they can be either branched or unbranched. Prochlorophytes have a starch-like reserve polysaccharide. They are also able to fix nitrogen.

73. (c) Intercalated discs or lines of Eberth are microscopic identifying features of cardiac muscle. These are unique structural formations found between the myocardial cells of the heart. They play vital roles in transmitting signals between cells. 74. (b) Kidney tubules originate from mesoderm. Intermediate mesoderm forms the kidneys, ureters and the vasculature. Human kidney development begins as early as the third week of embryonic development, with formation of the pronephros, followed by the mesonephros at 4 weeks and the metanephros at 5 weeks gestation. The development of the urinary system involves the transient formation and remodeling of the intermediate mesoderm. 75. (b) Steroidogenesis takes place in Leydig cells. It is a biological process of steroid hormone production. The major classes of steroid hormones are progesteron, corticosteroids, androgens and estrogens. Leydig cells Practice Set-8 119

YCT

92. (d) 84. (b) Haplopappus gracilis is a genus of flowering plant in the Cassia siamea also still commonly referred as Senna compositae (Asteraceae) family. It has lowest number of siamea. Cassia siamea is a popular forestry plant. The chromosomes (n = 2) known for higher plants. It is native flowers appear mainly in the hot season. Cassia siamea to the southwestern United states and northern Mexico, is an example of seasonal ecotype. 93. (c) where it grows in the deserts and plateaus. Mitosis and meiosis always differ in terms of presence 85. (b) of bivalent. Bivalent is always found in meiosis and Zeatin is a natural cytokinin which is discovered by absent in mitosis. During pachytene, the chromosomes Letham (1963) in immature corn kernels from the genus are fused in the form of bivalent or tetrad. Each zea. It promotes growth of lateral buds and when bivalent has four chromatids and a centromere. sprayed on meristems stimulates cell division to 94. (c) produce bushier plants. O3 is a pollutant gas in Troposphere. Ozone can trigger 86. (a) a variety of health problem including chest pain, Nepenthes Khasiana is an endemic plant to North coughing, throat irritation. Troposphere is the lowest Eastern region of India. Nepenthes khasiana is an region of the atmosphere extending from the earth's endangered tropical pitcher plant of the genus surface to the height of about 6-10 km. Nepenthes. It is thought to attract prey by means of blue 95. (b) fluorescence. Plastoquinone is an isoprenoid quinone molecules involved in the electron transport chain in the light 87. (d) Prokaryotic genetic system contains neither DNA nor dependent reaction of photosynthesis. Plastoquinone is histones. DNA stands for deoxyribonucleic acid. The hydrogen transporter in the photosynthetic electron transport system. nucleus or genetic material is absent in prokaryotes. 96. (d) Prokaryotes are unicellular organisms without Electron microscope was invented by Max knoll and membrane bound organelles. Ernst Ruska in 1931 while Antonie van Leeuwenhoek 88. (d) discovered the living cell. Succinic dehydrogenase is found in many bacterial cell Ernst Ruska at the University of Berlin, along with Max and inner mitochondrial membrane of eukaryotes. It is Knoll, built the first transmission electron microscope the only enzyme that participate in both the citric acid (TEM), for which Ruska was awarded the Noble Prize cycle and electron transport chain. for physics in 1986. It forms the complex II of the electron transport chain. 97. (d) It produces FADH2 during oxidation of succinate to Respiratory roots or pneumatophores are the roots of fumarate as FAD2+ gets reduced to FADH2. halophytes and also called "breathing roots" because their role in exchange of gases through numerous lentic 89. (d) Ecotone is a transition zone between two types of cells. These plants show viviparous (fruits germinate communities and represent a zone of special ecological without detaching from their mother plants eg. interest e.g. estuary is ecotone between marine and Rhizophora mucronata). 98. (c) fresh water communities. In day neutral plants, auxin have been found playing a 90. (a) florigenic role. IAA (indole acetic acid) promotes the The longest phase of meiosis is prophase-I. formation of female flowers. Application of gibberellins It is divided into five sub-phases can induce the formation of male flowers. (I) Leptotene (II) Zygotene (III) pachytene 99. (c) (IV) diplotene (V) Diakinesis. Xanthium Strumarium is a short day plant. The It shows the pairing of homologous chromosomes and 1 photoperiod of this plant is 15 hrs. The cocklebur the interchange of hereditary material. 2 Typically consuming 90% of the time for the two (Xanthium strumarium) is a short-day plant that begins divisions. to flower in the late summer and fall, when day length 91. (c) no longer exceeds 15 hours. In experiments by Karl The number of RNA molecules in 60s sub-particles of Hamner and James Bonner, cocklebur plants were 80s ribosomes are three. In 80s eukaryotic ribosome, grown continuously on a 16-hour light/8-hour dark there are two subunits, one large subunit, that is, 60s cycle. 100. (b) and one small subunit, that is, 40s. In animals, 60s subunit is further divided into 28s +5s + The phenomenon of keeping the earth warm due to presence of certain gases in the atmosphere is called 5.8s and 40s is divided in to 18s. In plants, 60s subunit is divided into 25s + 5s + 5.8s and green house effect. 'Green house effect' with respect to global climate refers to warming effect. 40s subunit is divided into 18s. Practice Set-8 120 YCT

PRACTICE SET - 9 1.

2.

3.

4.

A man slides down a snow covered hill along a curved path and falls 20 m below his initial position. The velocity in m/sec with which he finally strikes the ground is (g=10 m/s2) (a) 20 (b) 40 (c) 200 (d) 400 Which of the following statements is not true of X–rays: (a) X–rays are electromagnetic radiation of very short wavelength. (b) X–rays are produced by energy changes in the inner electrons of the atom. (c) X–rays have electric and magnetic fields. (d) In X–rays tube entire energy of the incident electron is converted to X–rays. In electromagnetic wave, electric and magnetic vectors oscillate perpendicular to each other. What is the phase difference of oscillation between the two vectors? (a) Zero (b) π/2 (c) π/4 (d) π Instantaneous current in a circuit is given by i = 2 2 cos (ωt + φ) Ampere. The value of r.m.s current is :

11.

(a) 2 2 Ampere

15.

(b) 4 Ampere

If two lenses having power P1 and P2 form a coaxial lens system with separation d between them, the power of the system will be: (b) P1 + P2 + 0.5dP1P2 (a) P1 + P2 + dP1P2 (c) P1 + P2 − dP1P2

12.

13.

14.

(c) 2 Ampere (d) 2Ampere 5. A straight wire of length L has current I flowing through it. A constant magnetic field B, parallel to the wire, is applied. What is the force on the wire 16. (a) Zero (b) BLI (c) B2LI (d) BL2I 6. A long solenoid of length 40 cm has 500 turns. A current 1.0 amp flows in it. The intensity of magnetic field at its center will be− 17. −3 3 (a) 1.57 × 10 N/amp m (b) 1.57 × 10 N/amp m (c) 1.57 N/amp m (d) 1.57 × 10−1 N/amp m 7. Which of the following produces magnetic field? 18. (a) Force (b) Electric dipole (c) Diamagnetic substance(d) Time varying field 8. Current through a capacitive element is called : (a) Conduction current (b) Transient current (c) Displacement current (d) Steady state current 9. Wave nature of electron is shown by (a) Photoelectric effect (b) Uncertainty principle (c) Electron diffraction (d) Thomson effect 10. A myopic eye can be corrected by using a: (a) convex lens (b) cylindrical lens (c) plano–convex lens (d) concave lens Practice Set-9 121

(d) P1 + P2 − 0.5dP1P2

If the focal length of objective lens is increased then magnifying power of (a) microscope will increase but that of telescope decrease (b) microscope and telescope both will increase (c) microscope and telescope both will decrease (d) microscope will decrease but that of telescope will increase A vessel of depth 0.5 metre is half filled with a liquid of refractive index µ1 and the upper half with another liquid of refractive index µ2. If µ1 = 1.2 and µ2 = 1.3, the apparent depth of the vessel as seen perpendicularly will be approximately (a) 0.3 metre (b) 0.4 metre (c) 0.35 meter (d) 0.45 meter A thin convex lens of focal length 25 cm and aperture 10 cm, is cut to have a hole of aperture 5 cm at the centre. The power of lens now is: (a) 2 D (b) 4 D (c) 1 D (d) 8 D Which one of the following applications is not associated with ultrasonics : (a) Crystal symmetry (b) Signalling (c) Optical communication (d) Depth sounding Two tuning forks when sounded simultaneously give one beat each in 0.25 s. The difference of their frequencies is: (a) 4 (b) 8 (c) 6 (d) 2 Rayleigh scattering is proportional to: (a) ν (b) ν2 (c) ν3 (d) ν4 The differential equation of a damped mechanical oscillator is : (where letters have their usual meanings (a) m

d2 x dt 2

+ kx = 0

2 (b) m d x + r dx + kx = 0 2

dt

dt

2

(c) m (d) m

d x dt

2

d2 x dt 2

+r

dx + kx = F0 sin wt dt

+r

dx + kx = F0 cos wt dt YCT

19.

20.

The frequency of a wave is 5 Hz. It refers to (type of wave) (a) ultrasonics (b) microwaves 27. (c) infrasonics (d) radio waves The expression for frequency of a longitudinal wave in a wire is: (a) f =

1 T 2ℓ m

(b) f =

E ρ

28. 1 E T (d) f = 2ℓ ρ m What remains constant during simple harmonic motion? (a) Restoring force (b) Kinetic energy (c) Potential energy (d) Time period The displacement equation of oscillations of a particle executing simple harmonic motion is x 29. = 5 sin (0.2πt + 0.5π) The time period of particle is (a) 10.0 s (b) 1.0 s (c) 0.8 s (d) 0.5 s If water flows out of hole at p as shown in figure, then x = ? 30.

(c) f = 21.

22.

23.

(a) x = h( H − h)

25.

26.

h ( H − h) 2

1 h( H − h) 2 A wrist watch (A) working on a spring system and a wall clock (B) working on a pendulum system are both set for the same timing on the surface of the earth. Thereafter they are taken to the moon. Then: (a) Both A and B will give the same time as on the earth (b) A will be faster than B (c) B will be faster than A (d) A and B both will give the same time but different from that on the earth The linear velocity of a satellite in a circular orbit is: (a) proportional to its mass (b) independent to its mass (c) proportional to square root of its mass (d) inversely proportional to its mass The weight of a body at the centre of the earth is: (a) zero (c) x = 2 h( H − h)

24.

(b) x =

Practice Set-9

31.

(d) x =

32.

33.

34.

35.

122

(b) infinite (c) same as on the surface of earth (d) none of the above D'Alembert's principle is used for (a) Reducing the problems of kinetics to equivalent statics problems (b) Stability of floating bodies (c) Designing safe structures (d) Solving kinematic problems A ball of mass 2 kg and another ball of mass 4 kg are dropped together from a 60 feet tall building. After a fall of 30 feet each towards earth, their respective kinetic energies will be in the ratio of (a) 1 : 2 (b) 2 : 1 (c) 1 : 4 (d) 1 : 2 Angular momentum of a particle of linear r r momentum p and at a distance r from the

origin is: r r (a) r × p r r r ×p (c) r r

rr (b) r.p r r r×p (d) r r r p

Two bullets P and Q of masses 10 gm and 40 gm have velocities 20 metres/sec. and 5 metres/sec. respectively. If both strike against a mud wall then : (a) Both will pierce equally (b) P will pierce more (c) Q will pierce more (d) Nothing can be said Which of the following salts is not soluble in water : (a) K2CO3 (b) BaCO3 (c) CaCl2 (d) Na2SO4 Which of the following liquids has maximum viscosity? (a) Glycol (b) Ethanol (c) Acetone (d) Water A first order reaction is 50% complete in 80 seconds. The time for 75% of the reaction to be complete is : (a) 160 seconds (b) 120 seconds (c) 240 seconds (d) 80 seconds Mathematical expression of Ostwald's dilution law : 1- α Cα2 (a) K = (b) K = 1− α C 2 C Cα (d) K = 2 (c) K = 1+ α α In the following reaction

B(OH)3 + H 2 O → [B(OH) 4 ]− + H + (a) B(OH)3 is a Lewis Acid YCT

36.

37.

38.

39.

40.

(b) B(OH)3 is a Lewis Base (c) B(OH)3 is amphoteric (d) None of these In the kinetic study of a reaction, A→Products a straight line was observed when a graph 1 was plotted. The between time (t) and C2 reaction is of (a) second order (b) third order (c) zero order (d) first order Solubility of PbI2 is S. The correct expression for solubility product, Ksp is (a) Ksp = S3 (b) Ksp = S2 3 (c) Ksp = 4S (d) Ksp = 4S3 Iron articles rust because of the formation of (a) Ferrous chloride (b) A mixture of ferrous and ferric hydroxides (c) Sodium carbonate (d) Ferric chloride Which one of the following Pairs of compounds illustrates the law of multiple proportion ? (a) H2O, Na2O (b) MgO, Na2O (c) Na2O. BaO (d) SnCl2'SnCl4 Which of the following hypothetical reactions will be favoured by an increase of temperature and pressure : (a) A(s) + 2B(g)     C(g) + D(g), ∆H = –ve (b) A(g) + 2B(g)     2C(g) + 2D(g), ∆H = +ve (c) 2A(g) + B(g)     2C(g) + D(s), ∆H = +ve (d)

41.

42.

43.

44.

45.

46.

47.

48.

(c) C2 H 5 OH

49.

50.

2A(s) + 2B(g)     2C(g) + 2D(g), ∆H = –ve

(s = solid, g = gas) A colloidal solution of silver iodide is prepared by adding excess of potassium iodide to silver nitrate solution. The charge on colloidal sol will be 51. (a) Positive (b) zero (c) negative (d) first negative and finely positive The valency of Fe in K3[Fe (CN)6] is (a) 6 (b) 2 52. (c) 4 (d) 3 Nitrogen has the electronic configuration 1s22s2 2p1x 2p1y 2p1z and not 1s22s2 2p 2x 2p1y 2p z

which is determined by : (a) Aufbau principle (b) Pauli's exculsion principle (c) Hund's rule (d) Uncertainty principle Out of HF, HCl, HBr and HI which one is 53. strongest acid in aqueous solution ? (a) HF (b) HCl (c) HBr (d) HI

Practice Set-9

Lines in an atomic spectrum are due to (a) Nuclear charge. (b) Energy of electrons and nucleus. (c) Excited electrons dropping from higher energy level to lower energy levels. (d) Electrons excited from lower energy levels to higher energy levels. Hydrometallurgy and pyrometallurgy are used for the preparation of (a) sodium and gold respectively (b) gold and iron respectively (c) iron and aluminium respectively (d) sodium and lithium respectively Which one of the following statements is not correct ? (a) Electrophilic substitution occurs at positions 2 and 5 in furan. (b) Pyridine undergoes Friedel-Crafts reaction. (c) Pyridine is more basic than aniline. (d) Pyridine is more basic pyrrole. Which of the following substance does not gives Haloform Reaction? (a) CH 3 CHO (b) CCl3 CHO

123

(d) All of these give this reaction

The radius of an atomic nucleus is of the order (a) 10–8 cm (b) 10–10 cm (c) 10–13 cm (d) 10–6 cm Conversion of phenol to salicylaldehyde is done by which of the following name reaction? (a) Barton reaction (b) Favorskii reaction (c) Vilesmeier reaction (d) Rimmer-Tiemann reaction The reactive intermediate involved in the reaction Br2 RCOO Ag → R – Br, is ∆

(a) Carbanion (b) Radical (c) Radical anion (d) Carbocation The absolute configurations of the chiral centre and double bond in

are respectively (a) R and E (b) R and Z (c) S and E (d) S and Z When cellulose boiled with dil H2SO4' final product is (a) glucose (b) fructose (c) sorbitol (d) gluconic acid YCT

54.

55.

Which of the following is an optically active 66. compound ? (a) n-Propoanol (b) n-Butanol (c) 2-Chlorobutane (d) 4-Hydroxyheptane Which one is the best reagent of accomplish the 67. following conversion? ? CH 3 CH 2 Br → CH 3 − CH 3

56.

(a) NaBH4 (b) Na/ether (c) Zn/C2H5OH (d) Mg followed by H3O+ Identify the name of the following reaction. CH3Br +2Na

57.

58.

59.

60.

61.

62.

63.

64.

65.

68.

Dry +BrCH3 ← Et O → C2H6+2NaBr 2

(a) Kolbe's reaction (b) Hydrogention reaction (c) Wurtz reaction (d) Corey-House reaction Among the following compounds the one that is most reactive towards electrophilic nitration is (a) Toluene (b) Benzene (c) Benzoic acid (d) Nitrobenzene Which has the maximum coagulating power for the precipitation of Ferric hydroxide Sol: (a) K3[Fe(CN)6] (b) NaCl (c) K2CrO4 (d) BaCl2 The value of the solubility product (KSP) of PbSO4 is 1.3×10-8. The solubility of PbSO4 in 0.2 M solution of Na2SO4 will be : (a) 2.6×10-8 g/l (b) 6.5×10-8 g/l -8 (c) 4.0×10 g/l (d) 1.2×10-8 g/l The suitable indicator for the titration of HCl against NH4OH is (a) Methyl red (b) Cresol red (c) Thymol blue (d) Methyl orange Smallest size of the cell which can be seen with naked eye : (a) 1 micron (b) 100 micron (c) 10 micron (d) 1000 micron Who discovered cell for the first time ? (a) Robert Brown (b) Robert Hooke (c) Antonie Van Leeuwenhoek (d) Theodor Schwann DNA of the mitochondria is : (a) naked (b) double stranded (c) circular (d) all of the above The regular sequence of cell cycle is : (a) S, G1, M, G2 (b) G1, S, M, G2 (c) G1, S, G2, M (d) G1, G2, S, M Organelles which are found in the mid-piece of the sperm : (a) Golgi body and lysosome (b) Ribosome and ER (c) Mitochondria and Centriole (d) Mitochondria and ribosome

Practice Set-9

69.

70.

71.

72.

73.

74.

75.

76.

77.

124

In a region if population of various species is small, which of the following will be strongest : (a) Natural Selection (b) Genetic drift (c) Hybridisation (d) Migration The first successful attempt to test in the theory that precursors of biomolecules could be abiotically synthesised under presumed primitive earth condition was made by : (a) Darwin (b) Huxley (c) Miller (d) Oparin Miller synthesized simple amino acids from one of the following mixtures in an experiment: (a) H2, O2, N2, (1 : 2 : 1) and water vapour (b) CH4, NH3, H2, (2 : 1 : 2) and water vapour (c) H2, O3, N2 (2 : 1 : 2) and water vapour (d) CH4, NH3, H2, (1 : 2 : 1) and water vapour Darwin's finches provide evidence of evolution from : (a) Anatomy (b) Morphology (c) Geographical distribution (d) all of the above First vertebrates on earth were(a) Ostracoderm (b) Cephalochordates (c) Urochordates (d) Placodermi During the process of digestion, the raw starch present in the diet is digested : (a) by salivary amylase only (b) by pancreatic amylase only (c) by both the salivary and pancreatic amylases (d) not digested at all Number of NADH2 produced in kreb's cycle : (a) 2 (b) 6 (c) 4 (d) 8 Which one of the following is the largest lymphatic vessel of the human body : (a) Throracic duct (b) Lacteal duct (c) Cysterna chyli (d) Right lymphatic duct The normal brownish colour of faeces results from the presence of the following : (a) stercobilin (b) mesobilirubin (c) urobilinogen (d) biliverdin Calcitonin regulate : (a) Na/K level (b) Cholesterol level (c) Plasma phosphate level (d) Ca++ level The largest ecosystem : (a) Forest ecosystem (b) Marine ecosystem (c) Pond ecosystem (d) Grassland ecosystem Reduced oxygen carrying capacity of blood is manifested by the presence of which pollutant in the air? (a) Nitrogen oxide (b) Sulphur dioxide (c) Carbon monoxide (d) All the above YCT

78.

79.

80.

81.

82.

83.

84.

85.

86.

87.

88.

First National Park in the world is : (a) Yellowstone National Park (b) Corbett National Park (c) Gir National Park (d) Dachigam National Park Cause of Bhopal tragedy on 3rd December, 1984 was– (a) Methyl Isocyanate (b) SO2 (c) CO (d) NH3 Manure is : (a) an organic substance (b) a chemical substance (c) obtained from decomposition of plants and animals (d) both (a) & (c) O3 in the ozone layer is produced by the action of (a) Blue radiation (b) IR radiation (c) UV radiation (d) Other visible radiations The major constituent of the cell organelle centriole is : (a) Actin (b) Myosin (c) Tubulin (d) Intermediate Filament Both PEP carboxylase and RUBISCO are found in the mesophyll cells of : (a) only C-3 plants (b) only CAM plants (c) only C-4 plants (d) None of the above Which is the age period of origin of prokaryotic microbes : (a) Precambrian (b) Proterozoic (c) Azoic (d) Palaeozoic The rate of transpiration is high when there is : (a) very low light intensity (b) very high humidity (c) very low humidity (d) very high temperature and high humidity Botanical generic name of bladderwort is (a) Drocera (b) Nepenthes (c) Utricularia (d) Dionaea Chiasmata are first seen during : (a) Leptotene (b) Pachytene (c) Zygotene (d) Diplotene Plants that lack perennating buds and are annuals are placed in the life form : (a) Phanerophyte (b) Chamaephyte (c) Cryptophyte (d) Therophyte

Practice Set-9

89.

In an article on ecosystem the following subtitles were used. Which one of them do you think is incorrect? (a) Cycling of minerals (b) Cycling of energy (c) Components of ecosystem (d) Pyramids of ecosystem 90. Which one of the following enzymes is characteristically present in the lysosome : (a) Transferase (b) Ligase (c) Acid hydrolase (d) Protease 91. DNA doubles at : (a) Interphase (b) Prophase (c) Metaphase (d) Telophase 92. Electron flow in cytochrome oxydase in the respiratory chain can be blocked by (a) Retinone (b) Cyanide (c) Amytal (d) Cycloheximide 93. Which of the following is a cytokinin ? (a) 2, 4, 5-trichlorophenoxy acetic acid (b) 3, 6-dichloro 2-methoxy benzoic acid (c) Chlormequat chloride (d) Benzyl amino purine 94. In grana, the reaction ADP + Pi=A T P during day designates : (a) Oxidative phosphorylation (b) Photophosphorylation (c) Substrate level phosphorylation (d) Dephosphorylation 95. The Indian Botanic Garden, Shibpur, Howrah, was founded by : (a) Robert William Kyd (b) Father Santapau (c) Nathaniel Wallich (d) William Roxburgh 96. The aleurone layer in maize grains is especially rich in : (a) Starch (b) Auxins (c) Lipids (d) Proteins 97. Oxidative phosphorylation occurs in : (a) Cytoplasm (b) Ribosomes (c) Mitochondria (d) Plasma membrane 98. Which one of the following elements is a micronutrient for plants ? (a) Magnesium (b) Manganese (c) Potassium (d) Sulphur 99. Cell cycle is governed by : (a) CDK cyclins (b) Glycocalyx (c) CMP (d) None of the above 100. What is the full form of BOD. (a) Biological Ozone Demand (b) Biological Oxygen Demand (c) Biotic Oxygen Demand (d) Biological Oxgen Deficiency

125

YCT

SOLUTION : PRACTICE SET- 9 ANSWER KEY 1. (a) 11. (c) 21. (d) 31. (b) 41. (c) 51. (b) 61. (b) 71. (c) 81. (c) 91. (a)

2. (c) 12. (d) 22. (a) 32. (a) 42. (d) 52. (a) 62. (b) 72. (c) 82. (c) 92. (b)

3. (a) 13. (b) 23. (c) 33. (a) 43. (c) 53. (a) 63. (d) 73. (a) 83. (b) 93. (d)

4. (d) 14. (b) 24. (b) 34. (a) 44. (d) 54. (c) 64. (c) 74. (a) 84. (b) 94. (b)

5. (a) 15. (c) 25. (b) 35. (a) 45. (c) 55. (c) 65. (c) 75. (d) 85. (c) 95. (a)

6. (a) 16. (a) 26. (a) 36. (b) 46. (d) 56. (c) 66. (b) 76. (b) 86. (c) 96. (d)

7. (d) 17. (d) 27. (a) 37. (d) 47. (b) 57. (a) 67. (c) 77. (c) 87. (d) 97. (c)

8. (c) 18. (b) 28. (d) 38. (b) 48. (d) 58. (a) 68. (b) 78. (a) 88. (d) 98. (b)

9. (c) 19. (c) 29. (a) 39. (d) 49. (c) 59. (b) 69. (c) 79. (a) 89. (b) 99. (a)

10. (d) 20. (c) 30. (a) 40. (c) 50. (d) 60. (d) 70. (a) 80. (d) 90. (c) 100. (b)

SOLUTION 7. (d) According to modified Ampere's max well law the variation of electric field with time causes magnetic field. The time varying electric and magnetic fields are dependent on each other. ν 2 = 0 + 2 × 10 × 20 • Electric dipole contains stationary charge hence no magnetic field is produced. ν 2 = 400 ⇒ ν = 20m / s • Diamagnetic material cannot produce their own 2. (c) magnetic field. X–rays do not have an electric and magnetic field. 8. (c) 3. (a) Current I(t) Any component electric circuit is defined as Since there is no time difference between peaks of Rate of flow of a charge Q(t) passing through it. To electric & magnetic oscillation the phase difference study that when charge within an enclosed surface is between electric & magnetic field vector of a lineary charging is Necessary to add Ampere’s law Another polarized electromagnetic wave is zero. current called displacement current 4. (d) dQin dφ Given eqn I = 2 2 cos (ωt + φ) Id = =∈0 e s dt dt I 9. (c) Irms = 0 Wave nature of electron is shown by Electron 2 diffraction. 2 2 10. (d) I0 = 2 2 Irms = = 2A To provide normal vision to the person suffering from 2 myopia, concave lenses of suitable focal length are used 5. (a) which focus the image on the retina by diffracting Magnetic force on a wire of length L and current I in a parallel rays. uniform magnetic field B is given by, F = I.B.L. Sin θ When magnetic field is applied parallel to if then, θ = 0º Hence, F = I.B.L. Sin0 ⇒ F =0 6. (a) Myopia is cured by using concave lens. Given- length l = 40 cm If the reflecting surface of a spherical mirror is raised N = 500 outwards then it is a convex mirror. I = 1 ampere The image of the object in a convex mirror is virtual, Magnetic field inside the solenoid, erect and small because in a convex mirror the mage is N formed before the point of focus. Convex mirrors are B = µ0 I l used as a mirror to increase the field of vision in vehicles, as a reflector in street lamps. −7 500 B = 4π × 10 ⋅ × 1 Tesla Concave mirrors are those spherical mirrors whose 0.4 raised surface r polished and the reflection occurs from B = 0.00157 Tesla the surface buried inwards. The image of a concave −3 B = 1.57 × 10 Tesla mirror object is smaller, inverted and real than that of 1. (a)

The man slides 20 m down under gravity. u = 0, g = 10m/s2 S = 20 m 2 2 ν = u + 2 gs

Practice Set-9

126

YCT

the object. It is used by doctors to examine eyes, nose, teeth, throat etc. 11. (c) If the focal length of two lenses if F1 and F2 and they are located at a distance 'd' from each other, in this case the resultant, 1 1 1 d Focal length = + − F f1 f 2 f1f 2 Power the system ⇒ P = P1 + P2 − d P1P2 12. (d) Magnifying power of a microscope L D × m= fo f e If focal length of objective lens (fo) is increased then magnifying power (m) of a microscope will decrease. Magnifying power of a telescope f m= o fe if focal length of objective lens (fo) is increased then magnifying power (m) of a telescope will also increase. 13. (b)

19. (c) Range of frequency for different waves1. Less than 20 Hz - Infrasonic 2. 20 to 20,000 Hz - audible sound waves. 3. more than 20,000 Hz - ultrasonic.

20. (c) The expression for frequency of a longitudinal wave in

1 E 2ℓ ρ Where ℓ = length of the wire E = elasticity of the wire ρ = density of the material of wire. 21. (d) In a simple harmonic motion the time period remains constant. The kinetic & potential energy depends on the displacement (y) that varies with time. Also the restoring force is directly proportional to the displacement and acts in the direction opposite to that of displacement. 22. (a) X = 5 sin (0.2πt + 0.5π) Comparing the given equation with standard equation X = A sin (ωt + θ) X = 5 sin (0.2πt + 0.5π) ω = 0.2 π Then, for calculating time period 2π 2π T= = = 10s ω 0.2π 23. (c) Bernoulli's theorem, discharge velocity v = 2 gh a wire, f =

1 1  dvirtual = dactual  +  µ µ  1 2  0.25 0.25 dvirtual = + = 0.4 m 1.2 1.3 14. (b) Cutting a hole in the lens will not affect the power. 100 100 Therefore power of the lens. P = = = 4D f 25 15. (c) Ultrasonic is related to sound. Optical communication is Vertical distance before striking the ground related to light. Hence ultrasonics is not associated with H – h optical communication. 1 16. (a) s = ut – gt 2 2 Q In 0.25 sec → One Beat produce 1 2 ∴ In 1 sec _→ 4 Beats will produce Therefore, the number of beats produced in 1 second H – h = 2 gt will be equal to the difference in frequency of the tuning 2(H − h) fork. t= Therefore the no of beats = 4 g 17. (d) x = speed × time 1 2( H − h) According to Rayleigh scattering, IS ∝ 4 x = 2 gh × λ g c Where, λ = wavelength and c = νλ , λ = ⇒ x = 2 h( H − h ) ν 24. (b) ν4 There will be no change in time in taking a clock with IS ∝ 4 Q c = constant spring system to the moon and for pendulum clock when c the value of g on the moon decreases, the value of 4 IS ∝ ν l 18. (b) T = 2π increases due to which the clock becomes The equation of damped mechanical oscillator is g sluggish. Hence clock A working on spring system will d2 x dx m 2 +r + kx = 0 run faster than clock B based on pendulum system. dt dt Practice Set-9 127 YCT

Ethylene glycol HO – CH2 – CH2 – OH has two –OH 25. (b) For the orbital velocity (V0) of the satellite groups the hydrogen bonding is due to these –OH groups more is the number of –OH group more are the mV0 2 GMe m hydrgoen bonds and greater is the viscosity ethanol has = R R2 only one –OH group hence the correct option is glycol. 33. (a) GMe GM e 2 ⇒ V0 = ⇒ V0 = For Ist order reaction R R 0.693 0.693 So, the orbital velocity (linear velocity in circular orbit) K= = = 8.6625×10–3 sec does not depend on the mass of the satellite. That is, t 80 independent of the mass of the satellite. at t = 0, the concentration of reactant = a 26. (a) after t time the concentration of reactant The acceleration due to gravity at the center of the earth 75a is g = 0 =x= = 0.75a = 0.25 a 100 h  because g ' = g  1 −  , [ h = R ] 2.303 a ∴K= log10  R t a − x therefore, weight = mg = 0 27. (a) 2.303 a t= log10 D'Alembert's principle 0.25a 8.6625 ×10−3 For a system of mass of particles, the sum of difference t = 160.04 sec. of the force acting on the system and the time or 34. (a) derivatives of the momenta is zero when projected onto Ostwald's dilution law relates the dissociation constant any virtual displacement. of the weak electrolyte with the degree of dissociation D'Alembert's principle and the concentration of the weak electrolyte. ∑ i ( Fi – mi a i ) δ ri = 0 Where AB ↽ ⇀ A+ + B– i = integral used for the identification of C (1 – α) Cα Cα variable corresponding to the particular particle in the Cα × Cα system. K= (1 − α ) Fi = Total applied force mi = Mass of ith particle cα 2 ai = acceleration of ith particle K = 1−α δri = Virtual of ith displacement 28. (d) where, α = degree of dissociation After going 30 feet, the velocity will be the same. 35. (a) 1 Lewis acids are defined as species which are electron 2 m1v K1 m1 2 1 deficient and can accept pair of electron. 2 = = = = =1:2 therefore, In the above reaction– B(OH)3 is having 6-electron in K2 1 m2 4 2 m2 v 2 its valence shell, hence is electron deficient. It will try 2 29. (a) to accept pair of electron to complete it valence shell. r r r Angular momentum L = r × p where p is linear H 3 BO3 + H 2 O  B(OH) −4 + H + momentum. The angular momentum is also called as Hence B(OH)3 is reacting as a lewis acid. moment of momentum. B(OH)3 + NaOH → NaBO2 + 2H2O 30. (a) Sodium Momentum of P = mP × vP –3 –1 meta burate = 10×10 × 20 = 2 × 10 kg m/s 36. (b) Momentum of Q = mQ × vQ The reaction whose rate depends on the three = 40 × 10–3 × 5 = 2 ×10–1 kg m/s Since momentum of both bullets are same so both will concentration step is called third order of reaction. For a reaction of third order (n = 3) the inverse of the pierce equally. square of the concentration of the reactant that is 1/C2 at 31. (b) Carbonates of alkaline earth metals are insoluble in 't' time graph gives a straight linewater (insoluble) and on heating form metal oxides and carbon decomposed into dioxide Hence barium carbonates (BaCO3) is insoluble in water BaCO3 is not soluble in water. In general alkaline earth metal carbonates are not soluble in water. On the other hand alkali metal Carbonates (such as 37. (d) K2CO3) sulphates (Na2SO4) and chlorides (CaCl2) are The solubility product constant is the equilibrium constant for the dissolution of a solid substance into an soluble in water. aqueous solution. It is denoted by the symbol Ksp. 32. (a) Hydrogen bonding and strong intermolecular interaction The solubility product is a kind of equilibrium constant and its value depends on temperature. increases the viscosity of the liquid. Practice Set-9 128 YCT

PbI 2 ⇌ Pb 2+ + 2I − ⇒ K sp = S × ( 2S) ⇒ K sp = S × 4S2 2

⇒ K sp = 4S3 38. (b) Rusting of iron is a type of corrosion process there are some metals such as iron which are known as air on when left open in the atmosphere, they start corroding by reacting with oxygen and moisture etc. 2+ − Fe → Fe + 2e (Oxidation) −



SO, HI is the strongest acid in aqueous solution. 45. (c) Lines in the spectrum were due to transitions in which an electron moved from a higher energy orbit with a larger radius a lower energy orbit with smaller radius. 46. (d) Hydrometallurgy and pyrometallurgy are used for the preparation of Sodium and Lithium respectively. 47. (b) In the presence of anhydrous aluminum chloride catalyst, The introduction of an alkyl or acyl group into the benzene ring is called Friedel-Crafts reaction, whereas in pyridine one of the carbon of the benzene ring is replaced by a nitrogen atom, which is due to the presence of an electron pair behaves like a nucleus, the benzene ring undergo electrophilic substitution reaction by the Friedel-Crafts reaction. 48. (d) All of them give haloform reaction methyl alcohol does not give haloform reaction ethyl alcohols acetaldehyde acetones methyl ketone and those alcohol containing CH3CH (OH) group give haloform reaction.

2H 2 O + 2e → H 2 + 2OH (Reduction) The combination of ferrous ion (Fe2+) and hydroxide ion (OH-) forms ferrous hydroxide Fe(OH)2 which air oxidizes to ferric hydrated ferric oxide. (Fe2O3.H2O) this is called Rust. 39. (d) Law of multiple proportion -: This law which was first by Dalton in '1804' may be defined as followsWhen two element combine two or more chemical compound, then the masses of one of the element which combine with a fixed mass of the other, bear a simple ratio to one another. Ex-A simple multiple proportion ratio of the combination of carbon and oxygen 1 : 2 (CO, O2) * A simple multiple proportion ratio of in the combination of sulphur and oxygen 2 : 3 (SO2, SO3) * Similarly the combination Sn (tin) and chloride will have a simple multiple proportion. Ratio SnCl2, SnCl4 has a simple multiple ratio of 1 : 2. 49. (c) 40. (c) The radius of an atom = 10-10 m. The reaction which produce heat in gaseous reactions The radius of an atomic nucleus = 10–15 m or 10-13 cm are called exothermic reaction and the reaction which An atom consists of a nucleus in the nectar surrounded by require heat are called endothermic reaction, the electrons revolving around it. temperature and pressure are increased in those reaction The nucleus consists of protons (positively charged) and which are endothermic. Option (c) 2A(g) + B(g)   neutrons (neutrally charged).  2C(g) + D(s), ∆H = +ve the action of increasing the Compared to the size of the atom, the size of the nucleus is extremely small. temperature and pressure will be more helpful. 50. (d) 41. (c) Reaction of phenol with chloroform (CHCl3) in the KI + AgNO3 → AgI ↓ presence of "NaOH", hydroxylbenzaldehyde is When potassium iodide in excess amount reach with generated this reaction is known as Reimer-tieman colloidal particles of AgI acquire negative charge due to reaction here first the strong aqueus solution of NaOH absorption of I– ion on colloidal particles. deprotonates phenol and chloroform carbanion is 42. (d) generated respectively, this chloroform carbanion K3Fe(CN)6 - If the oxidation number of "Fe" is readily undergoes alpha elimination producing dichlro K 3 Fe(CN)6 carbene, then dichlorocarbene attaks sodium phenoxides and an intermediates is formed which on basis (+1) × 3 + x + (−1) × 6 = 0 hydrolysis produces ortho-hydroxy benzalehyde. 3+ x − 6 = 0 x −3 = 0 x=3 Hence the valency of 'Fe' is '3'. 43. (c) Hund's Law of maximum multiplicity electrons in different orbitals of a subshell do not pair up untill each orbital of that subshell is filled with one electron each. So this is the reason that the electronic configuration of 1s2 2s2 2p1x 2p1y 2p1Z is not 1s2 2s2 2p 2x 2p1y 2p0Z because it contradicts Hund's rule. 44. (d) Acidity of hydorgen halide (HX) increase down the group. So order of acidity in aqous solution HF < HCl < HBr < HI. Practice Set-9 129 YCT

51. (b) 58. (a) The decomposition of the silver salt of a carboxylic acid Coagulation power for the precipitation of Fe(OH)3 with Br2 in refluxing CCl4 to from an alkyl or aryl increases by increasing the negative charge because Fe bromide with one carbon less than the original acid is contain positive charge So K3[Fe(CN)6] is maximum called Hunsdiecker reaction. coagulating power due to containing [Fe(CN)6]3– Br2 → R – Br, + CO + AgBr is a free negative charge because Fe has positive charge. RCOO Ag  2 59. (b) CCl4 radical reaction because it is Hunsdiecker reaction. PbSO4  Pb 2+ + SO24− Ksp = 1.3 × 10–8 52. (a) The common ion effect PbSO4 in solution of NaSO4 in Absolute configuration refers to the spatial arrangement 0.2 M. the decrease in solubility. of atoms wihin a chiral molecular entity, will carbon on −8 double or triple bond be chiral centers because they So, PbSO in solubility = K sp = 1.3 × 10 4 can't have bond to four different group of R and E. C 0.2 53. (a) = 6.5 ×10−8 gram/litre. The hydrolysis of cellulose is done by treating with 60. (d) ''conc H2 SO4'' The suitable indicator for the titration of HCl against NH4OH is Methyl orange due to solution is acidic. 61. (b) Smallest size of the cell which can be seen with naked Cellulose glucose hence, when cellulose boiled with dil H2SO4 final eye is 100 micron. The human eye has a convex lens, and the visibility of the eye depends on its resolving product is ''Glucose''. power. Generally, the size range of cells like egg cells, 54. (c) unusually large bacteria, some amoeba and squid nerve 2-chlorobutane cells are of the range of 100 micron and these are H visible to the naked eye. 62. (b) I Cell was first discovered by Robert Hooke in 1665. He CH 3 − CH 2 − C − CH 3 discovered plant cells by viewing the cell walls in its cork tissue under a microscope. He described the cell as I the basic unit of life. He observed that there was a Cl For an optical activity, the compound should have a honeycomb like structure and he named the parts of the chiral carbon. Chiral corbon is a carbon whose all four structure as cells. 63. (d) valencies are satisfied by 4 different group. 2-chlorobutane contains are chiral carbon. So, it is Mitochondrial DNA is double stranded, naked and circular. Mitochondrial DNA is the small circular optically active. chromosome found inside mitochondria. The 55. (c) When bromoethane is reduced by hydrogen at ordinary mitochondria are organelles found in cells that are the sites of energy production. The mitochondria and the room temperature then ethane is produced. mitochondrial DNA, are passed from mother to Zn /C2H5OH offspring. CH 3CH 2 Br + 2 [ H ]  → CH 3 − CH 3 64. (c) Ethyl bromide Ethane The cell progresses through defined phase of the cell 56. (c) cycle during its division. First, the G1 in which some Reaction of alkyl halide reaction with dry ether sodium organelles starts their duplication then comes S phase, metal to form an alkane. Which is called wurtz reaction. where DNA has doubled, then G2 phase and finally the Dry CH 3 Br + 2Na + BrcH 3 ← → CH 3 − CH 3 + 2NaBr M phase. Et 2 O so the correct sequence is "G1, S, G2, M". 57. (a) The presence of electron releasing group like - R, OH, 65. (c) etc. increases the elecron density at O/P position and The sperm cell consists of a head, a midpiece and a tail. thus makes the benzene ring more reactive (at O/P The head contains the nucleus with densely coiled position) toward electrophile on the other hand. electron chromatin fibres, surrounded anteriorly by an acrosome, withdrawing group like- COOH - NO2 etc. If present which contains enzymes used for penetrating the female reduses electron density and thus tthe activity benzene egg. The mid piece has a central filamentous core with nucleus toward electrophile, thus the order of the given many mitochondria spiralled around it, used for ATP production for the journey through the female cervix, compound towards, electrophilic nitration is – uterus and uterine tubes. The tail or flagellum executes the lashing movements that propel the spermatocyte. 66. (b) In a region if population of various species is small, genetic drift will be strongest. Genetic drift is a random fluctuations in the frequencies of alleles from generation to generation due to chance events. It can cause traits to be dominant or disappear from a population. The effects Thus toluene is most reactive towards electrophilic of genetic drift are most pronounced in small populations. nitration. Practice Set-9

130

YCT

67. (c) The theory of origin of life was propounded by Oparin. But experimented by Miller. Miller take CH3, NH3, H2 and water vapour in 2:1:2 ratio in his experiment. 68. (b) Miller synthesized simple amino acids from mixtures of CH4, NH3, H2, (2 : 1 : 2) and water vapour in an experiment. Miller - Urey experiment was carried out in 1952, which showed that complex organic molecules could be synthesized from inorganic precursors. Miller took molecules of methane (CH4), ammonia (NH3), hydrogen (H2) and water (H2O) and put them into a closed system. Then, a continuous electric current was ran through the system, to stimulate lightning storms. At the end of one week it was observed that as much as 10-15% of the carbon was now in the form of organic compounds. 2% of the carbon had formed some of the amino acids, which are used to make proteins. 69. (c) Darwin's finches provide evidence of evolution from geographical distribution. Darwin's finches are a group of about 18 species of passerine birds. They were first collected by Darwin on the Galapagos islands. He observed that the Galapagos finches had a graded series of beak sizes and shapes and predicted that these species were modified from one original mainland species. Darwin called differences among species natural selection, which is caused by the inheritance of traits, competition between indivudals, and the variation of traits. 70. (a) First vertebrates on earth were Ostracoderm. Ostracoderm are the armored jawless fish of the Palaeozoic Era. They are represented by marine and fresh water deposits of the Devonian period. Their fossils are found in the North American and European strata of the Ordovician, Silurian and Devonian periods of the Palaeozoic Era, approximately 400 million years ago. 71. (c) The process of digestion of the raw starch present in the diet is digested first in mouth in presence of enzyme salivary amylase and by pancreatic amylase in small intestine to break down starch, glycogen & dextrins. 72. (c) The krebs cycle was discovered by Kreb's in 1937. This action take place in the Matrix of Mitochondria. In this process a total of 4 molecules of NADH2 are produced from each NADH molecule, two electron' and two hydrogen atom are removed and 3ATP molecule are synthesized in order to reach oxygen. 73. (a) The largest lymphatic vessel of the human body is thoracic duct. The lymph flows away from the tissues to lymph node and eventually to either the right lymphatic duct or the largest lymph vessel in the body (Thoracic duct). 74. (a) Brownish colour of faeces result from the presence of Stercobilin. Biliverdin is present in the bile juice. It is excretory product derived by the disintegration of haemoglobin of the dead RBCs. Stercobilin is a tetrapyrrolic bile pigment and product of heme catabolism. 75. (d) Calcitonin regulate Ca++ level. It is produced by the thyroid gland. It reduces the concentration of blood Practice Set-9

calcium level when it has risen to an above normal level. Calcitonin opposes the effects of parathyroid hormone (PTH) which acts to increase the blood calcium level. Calcitonin lowers blood calcium levels by suppressing osteoclast activity and increasing the amount of calcium excreted in the urine. 76. (b) Largest and permanent ecosystem is marine ecosystem. An ecosystem is a geographic area where plants, animals and other organisms, as well as weather and landscape, work together to form a bubble of life. Ecosystems contain biotic or living parts as well as abiotic factors or non living parts. 77. (c) Carbon monoxide (CO) combines with haemoglobin to form carboxyhaemoglobin. This prevents oxygen binding to haemoglobin, thereby reducing the oxygencarrying capacity of the blood, leading to hypoxia. CO (Carbon monoxide) has higher affinity to haemoglobin as compared to oxygen. 78. (a) First National Park in the world is Yellowstone National Park (USA) was made on 1 March 1872 in USA (America). The park is popular for its wildlife and its many geothermal features. The vast forests and grasslands comprise of unique species of plants. The area of the park is approximately 2,219,791 acres (8983.18 km2). The park consists of lakes, rivers and mountain ranges. 79. (a) Cause of Bhopal tragedy on 3rd December 1984 was methyl isocyanate. Bhopal gas tragedy was a gas leak incident on the night of 2-3 December 1984 at the Union Carbide India Limited Pesticide plant in Bhopal, Madhya Pradesh India. It harms the surrounding climate atmosphere and affects on thousands of animals and plants. 80. (d) Manure is an organic substance or animal dung used for fertilizing land. Garbage in the houses and in the field is constantly coming out in large quantities, animal dung, urine etc, are also obtained in agricultural fields. By scavenging them properly, their nutrients are used to increase production. 81. (c) O3 in the ozone layer is produced by the action of UV radiation. The ozone layer contains high concentration of O3. This layer absorbs harmful UV radiation. On depletion of ozone layer UV radiation (UV B radiation) reaches on earth which causes skin cancer, mutation and cataract etc. 82. (c) The major constituent of the cell organelle centriole is tubulin. Tubulin is a protein that can polymerize into long chains or filaments that form microtubules. Most centrioles are made up of nine circularly arranged triplet microtubules. Centrioles are present in animal cells and the basal region of cilia and flagella in animals and lower plants. 83. (b) PEP (phospho-enol pyruvate) carboxylase and RUBISCO both are found in CAM plants. This metabolism was first of all reported in Bryophyllum, a member of family Crassulaceae and hence is called crassulacean acid metabolism (CAM). In these plants,

131

YCT

there is no kranz anatomy, but there occurs dark acidification i.e. during night malic acid is formed. In these plants, stomata open during night and close during day time. 84. (b) Proterozoic era is called as age of prokaryotic microbes. Beginning and end of the proterozoic were marked by widespread glaciation. About one billion years ago, the dominant organisms of earth were cyanobacteria and protozoans, Hence, it was named proterozoic. During archeozoic era, precambrian era and phanerozoic era, microbes are not dominant. 85. (c) Transpiration is the process of water movement through a plant and its evaporation from aerial parts, such as leave stem and flowers. The rate of transpiration is high where there is very low humidity in atmosphere. 86. (c) The Botanical name of bladderwort is Utricularia. It is an aromatic insectivorus plant. It is a genus of carnivorous plant consisting of approximately 233 species but certain species can be sensitive to harsh, direct light, its come as carnivorous plants. 87. (d) Chiasmata are first seen during diplotene stage of prophase I of meiosis. Chiasmata is the point of contact between homologous chromosomes. In pachytene stage actual crossing over of genetic material occurs. Leptotene is the first stage of meiosis where chromosomes are seen as threads while zygotene is the second stage of meiosis where homologous chromosomes start pairing. 88. (d) Therophytes are annual plants that complete their life cycle and survive in unfavorable conditions in the form of seeds. They are usually found in deserts and other dry areas. 89. (b) In an ecosystem there is transfer of energy and not cycling of energy. Because the flow of energy is unidirectional in an ecosystem. There are loss of energy at each trophic level that's why cycling of energy do not take place in an ecosystem. 90. (c) The Acid hydrolase are hydrolytic enzymes that are active at the acidic pH maintained within the lysosome, for optimal activity they require an acid environment. Lysosomes contain a wide variety of Hydrolytic enzymes (Acid Hydrolases) that break down macromolecules such as nucleic acid using hydrolytic enzymes. 91. (a) Interphase is a period of intense synthesis and growth. During interphase, duplication of the chromosomes occurs. Each chromosome (called the monad) becomes double, and is then called the dyad. 92. (b) Cyanide blocks electron flow in cytochrome oxidase in the respiratory chain. It disrupts respiration by inhibiting the action of cytochrome C-oxidase, an enzyme produced in mitochondria that mediates the last stage in the electron transfer chain in mitochondria.

Practice Set-9

93. (d)

6-Benzylaminopurine is known as cytokinin. Chemical formula – C12 H11 N5 6-Benzylaminopurine is a first generation cytokinin plant growth influencing plant growth and development, setting blossoms and stimulating fruit richness by stimulating cell division. 94. (b) Photophosphorylation is the process in which light energy is converted into chemical energy through the production of ATP. It is the process of formation of ATP from ADP and inorganic phosphate (Pi) utilizing light energy. It occurs in grana of chloroplast during photosynthesis. Electron transport chain is a series of electron carriers over which electrons pass in a downhill journey releasing energy at every step that is used in generating an electrochemical proton gradient which helps in synthesizing ATP. Based on the path of electrons, associated photophosphorylation can be identified as non-cyclic and cyclic phosphorylation. 95. (a) The Indian Botanical Garden, Shibpur, Howrah, was founded by Robert William kyd in 1787. 96. (d) The aleurone leyer is the outermost layer of the endosperm (triploid tissue). The aleurone layer in maize grains is especially rich in proteins. Zeatin protein is found in maize zea mays. 97. (c) Oxidative phosphorylation occurs in mitochondria. Oxidative phosphorylation is the process by which ATP synthesis is coupled to the movement of electrons through the mitochondrial electron transport chain and the associated consumption of oxygen. 98. (b) Essential elements are divided into two parts. The macronutrients are- S, P, H, C, O, N, Ca, Mg and micronutrients are – Ce, B, Fe, Mn, Cu, Ni, Mo. etc. 99. (a) Cell cycle is governed by CDK cyclins. Cyclin dependent kinase (CDKs) are a family of sugar kinases first discovered for their role in regulating cell cycle. They are present in all known eukaryotes. CDKs are small protein. 100. (b) The full form of BOD is Biological Oxygen Demand. It is a measure of the amount of oxygen that is required for the decomposition of biodegradable organic matter in a sample of water by aerobic organic decomposers. The BOD value of the dirty water coming out of the dirty drain of the house is 200-400 ppm oxygen. (for 1L of water). The BOD of clean drinking water should be less than 1 ppm.

132

YCT

PRACTICE SET - 10 1.

2.

3.

4.

5.

6.

7.

8.

9.

10.

A man can throw a ball 100 meters vertically upwards, The greatest horizontal distance, he can throw the ball, is : (a) 100 meters (b) 980 meters (c) 200 meters (d) 1000 meters The wavelength of the microwave of the order of: (a) meter (b) millimeter (c) micrometer (d) angstrom The electromagnetic damping experienced by a metal mass moving in a magnetic field is due to: (a) alternating current (b) eddy current (c) magnetic field (d) alternating potential, produced in the metal mass. The alternating current cannot be used for (a) Heating (b) Lighting (c) Generate mechanical energy (d) Eletroplating The substances which are attracted by the magnetic field are– (a) Diamagnetic (b) Paramagnetic (c) Ferromagnetic (d) Ferrimagnetic Which of the following can produce a magnetic field? (a) Capacitor (b) Electric dipole (c) Diamagnetic substance (d) Time varying electric field A person enters a magnetic field of flux density 1.5 Wb/m2 with a velocity of 2×107 m/s at an angle of 300 with the field. The force on the proton will be : (a) 2.4×10–12 N (b) 2.4×10–5 N –12 (c) 4.2×10 N (d) 1.6×10–19 N The displacement current arises due to (a) flow of electrons only (b) flow of holes only (c) flow of both electrons & hole (d) time varying electric field Scattering cross-section σ(θ) for Coulomb potential using Born approximation is (a) proportional to sin4θ/2. (b) inversely proportional to sin4θ/2 (c) proportional to sin2θ/2 (d) inversely proportional sin2θ/2 The phenomenon of diffraction was discovered by: (a) Fraunhofer (b) Fresnel (c) Huygens (d) Grimaldi

Practice Set-10

11.

To minimize spherical aberration, two lenses of focal lengths f1 and f2 are placed at a distance equal to (a) f1 -f 2 (b) f1 +f 2

f1 -f 2 f +f (d) 1 2 2 2 A convex lens of power + 6D is placed in contact with a concave lens of power - 4 D. What is the nature and focal length of the combination(a) Concave, 25 cm (b) Convex, 50 cm (c) Concave, 20 cm (d) Convex, 100 cm A luminous object is placed at a distance D from a screen. A converging lens, placed exactly at the mid point between the lens and screen, produces a real image on the screen. Focal length of the lens will be : (a) D (b) D/2 (c) D/4 (d) D/8 4 If the refractive index of water is and that of 3 5 glass is , then critical angle of light entering 3 from glass into water will be : (a) sin −1 4 / 5 (b) sin −1 5 / 4 (c) sin −1 1/ 2 (d) sin −1 2 Which one of the following is not correctly matched ? (a) When the optical Principal points system is situated coincide with the in the same medium nodal points (b) The planes passing Nodal planes through the nodal point and perpendicular to the axis (c) The planes passing Focal planes through the principal foci and perpendicular to the axis (d) The number of Four cardinal points of an optical system One of the causes of the breadth of the spectral lines is : (a) Doppler's effect (b) Dispersion (c) Reflection (d) Refraction A source of unknown frequency produces 8 beats with a source of 250 Hz and 12 beats with a source of 270 Hz. The frequency of the unknown source is (c)

12.

13.

14.

15.

16.

17.

133

YCT

18.

19.

20.

21.

22.

23.

24.

25.

26.

27.

(a) 258 Hz (b) 242 Hz (c) 262 Hz (d) 282 Hz In a mass spring system oscillating with angular frequency ω, the kinetic and potential energies of the system vary harmonically with frequency : (b) 2ω (a) ω (c) ω/2 (d) zero Wavelength of heat radiation is : (a) 1Å – 104 m (b) less than 4000 Å (c) 4000 Å – 7800 Å (d) more than 7800 Å Ultrasonic waves are generated through (a) Photo–electric effect (b) Piezo–electric effect (c) Faraday effect (d) Comptom effect Two rectangular SHMs are represented by x=asin (2 ωt + φ) and y=b sin ωt. The resultant of superposition of two represents (a) a parabola (b) the figure of 8 (c) an ellipse (d) a straight line Vibrations of the diaphragm of a microphone are (a) free oscillation (b) Damped oscillations (c) resonant oscillations (d) forced oscillations The apparent weight of a body in a lift is more when the lift is : (a) going up with a constant speed (b) being accelerated downward (c) going down with a constant speed (d) being accelerated upward If the earth stops rotating, the value of 'g' at the equator will? (a) Increase (b) Remain same (c) Decrease (d) None of the above The distance of earth from the sun = 1.495 × 1011 m revolution time = 365.4 days. If the revolution time for some other planet is 730.8 days, its distance from the sun will be (in meter): (a) 41/3 × 1.495 × 1011 (b) 21/3 × 1.495 × 1011

28.

(a)

πηr 4 l

(b)

2πηr 4 l

πηr 4 πηr 4 (d) 2l 3l Numerical values of the momentum and kinetic energy of a particle are equal. Velocity of the particle is : (a) 1 ms–1 (b) 2 ms–1 (c) 4 ms–1 (d) Momentum cannot be equal to the kinetic energy An object with a constant speed: (a) is always accelerated (b) is not accelerated (c) might be accelerated (d) also has a constant velocity Frenkel defects of a crystal arise when (a) Some of the lattice points in a position unoccupied (b) An ion occupies an interstitial position between lattice points (c) Ions occupy its own appropriate site in lattice (d) All of the above Which one of the following statements is wrong? (a) Ostwald "dilution law" is based on the Arrhenius theory of "electrolytic dissociation" (b) The ionization may take place even in the absence of water. (c) The degree of dissociation obtained from colligative, properties and conductance measurements for a bi-bivalent electrolytes are nearly the same. (d) The heat of neutralization of all strong acids and strong basis it 57.3 kJ. The false statements among the following is (a) Hardy-Schulze rule is related to coagulation. (b) Gold number indicates protective power of a lyophilic colloid. (c) Colloidal solution of a liquid in liquid is called gel. (d) Tyndal-effect is a characteristic of colloid. When a substance is dissolved in a solvent the vapour pressure of the solvent is decreased. This results in (a) An increase in the b.p. of the solution (b) A decrease in the b.p. of the solvent (c) The solution having a higher osmotic pressure than the solvent (d) The solution having a lower osmotic pressure than the solvent (c)

29.

30.

31.

32.

33.

(c) 8 × 1.495 × 1011 (d) 2 × 1.495 × 1011 The weakest force of interaction is : (a) electrostatic (b) gravitational 34. (c) nuclear (d) electromagnetic A collision between two particles is perfectly elastic, when their : (a) Linear momentum is not conserved. (b) Kinetic energy is not conserved (c) Linear momentum and kinetic energy are fully conserved (d) Linear momentum is conserved but kinetic energy is not conserved

Practice Set-10

Twisting couple per unit angular twist is : where symbols have their usual meanings-

134

YCT

35.

36.

37.

38.

39. 40.

41.

42.

43.

44.

According to which law CO and CO2 are formed(a) Law of constant proportion (b) Gay-Lussac's law (c) Law of multiple proportion (d) Law of reciprocal proportion Amongest the following, strongest acid is : (a) CH3COOH (b) ClCH2COOH (c) CH3CH2COOH (d) Cl3CCOOH Which of the following statements does not form part of Bohr's model of the hydrogen atom? (a) Energy of the electrons in the orbit is quantized (b) The electron in the orbit nearest the nucleus have the lowest energy (c) Electrons revolve in different orbits around the nucleus (d) The position and velocity of the electrons in the orbit cannot be determined simultaneously If the value of the equilibrium constant (Kc) for dissociation of PCl5 at 3000C is 0.0625 mole–1, then value of equilibrium constant Kp is: (a) 0.0625 (b) 6.25 (c) 62.5 (d) 2.94 Lepidolite is an ore of : (a) Ti (b) V Which one of the following produces chemical valcano on heating? (a) K2CrO4 (b) ZnCr2O7 (c) (NH4)2Cr2O7 (d) K2Cr2O7 Ultramicroscope has been devised on the principle of: (a) Brownian movement (b) Elecrokinetic potential (c) Electrodialysis (d) Tyndal effect (Scattering of light) How many grams of calcium oxalate will dissolve in one litre of sturated solution? The solubility product of calcium oxalate is 2.5 × 10-7 : (a) 0.0032 g/L (b) 0.0128 g/L (c) 0.0005 g/L (d) 0.064 g/L In Which of the following compounds, the oxidation state of oxygen is +2 ? (a) HClO (b) OF2 (c) Cl2O7 (d) H2S2O8 The percentages of copper and oxygen in samples of CuO obtained by different methods were found to be the same. This illustrates the law of : (a) Constant proportion (b) Conservation of mass (c) Multiple proportion (d) Reciprocal proportion

Practice Set-10

45.

46.

47.

48.

49.

50.

51.

52.

53.

54.

55.

135

The half-life period of a reaction is 80 minutes at a particular concentration when intial concentration of the reactant is reduced to half, the half-life period is also reduced to half. The order of the reaction would be: (a) First order reaction (b) Second order reaction (c) Zero order reaction (d) Third order reaction Which of the following has the highest boiling point? (a) CH4 (b) CH3OH (c) CH3Cl (d) CH3Br Electronic configuration of H- is : (a) 1s0 (b) 1s1 2 (c) 1s (d) 1s1 2s1 pE-pH diagram is : (a) Pourbaix diagram (b) ε 0 − ph diagram (c) Important region diagram (d) All of the above Number of (OH–) in 1 ml solution of pH = 13 is (a) 1×10-13 (b) 6.00×107 13 (c) 6.00×10 (d) 6.02×1019 An electron in an atom can be identified by(a) It's spin (b) Charge (c) The path in which it is spinning (d) Four quantum number Reaction of benzoic acid with diazomethane gives (a) Phenyl acetic acid (b) P-methyl benzoic acid (c) Methyl benzoate (d) Phenyl diazoketone When Naphthalene is oxidised with Na 2 Cr2 O7 / H 2SO 4 we get ? (a) Phthalic acid (b) Benzoic acid (c) Tetralene acid (d) Phenyl acetic acid The starting compound in case of Backmann Rearrangement is an: (a) Amide (b) Acid (c) Indole (d) Oxime Reimer-Tiemann reaction of phenol with chloroform and alkali is an example of: (a) nucleophilic substitution (b) addition reaction (c) elimination reaction (d) electrophilic substitution Which one of the following is used to change nitrobenzene into N-Phenyl hydroxylamine? (a) Zn/NH4C1 (b) Sn/HC1 (c) Zn/NaOH (d) Sn/KOH

YCT

56.

57.

58.

59.

60.

61.

62.

63.

64.

65.

66.

Silver salt method is used for the determination is molecular weights of the organic compound (a) Acids (b) Bases (c) Hydrocarbons (d) Sulphonic acid The reaction of sulphur with Grignard's reagent followed by acidification leads to the formation of : (a) thioether (b) sulphoxide (c) mercaptan (d) sulphonic acid Which of the following is an example for addition reaction? (a) Friedel Crafts reaction (b) Aldol reaction (c) Bromination of phenol (d) Nitration of benzene Which of the following liberates methane gas on treatment with water? (a) Silicon carbide (b) Calcium (c) Aluminium carbide (d) Iron carbide In the three heterocyclic compounds, for example, furan, pyrrole and thiophene, the order of aromaticity is : (a) Thiophene > Pyrrole > Furan (b) Pyrrole > thiophene > furan (c) Pyrrole > furan > thiophene (d) Furan > pyrrole > thiophene The most abundant molecule of a cells is : (a) DNA (b) RNA (c) Protein (d) Water The 'Unit-membrane' concept for the structure of the plasma membrane was developed in which of the following proposed models: (a) Fluid mosaic model of 1972 (b) Robertson model of 1959 (c) Garter and Grendel model of 1926 (d) Danielli and Davson model of 1926 The organelles in which aerobic respiration occurs in a cell are : (a) chloroplasts (b) ribosomes (c) dictyosomes (d) mitochondria Which of the following does not happen in the Prophase-1 of meiosis? (a) Segregation (b) Chromosome condensation (c) Pairing of homologous (d) Chiasmata formation In humans, during cell cycle, G1 phase last for about : (a) 12 hours (b) 8 hours (c) 10 hours (d) 6 hours P and Q are gene frequencies of dominant and recessive alleles respectively. Which expression is correct for Hardy-Weinberg law? (a) P2 + Q2 = 1 (b) P + Q = 1 (c) P – Q = 1 (d) P2 – Q2 = 1

Practice Set-10

67.

68.

69.

70.

71.

72.

73.

74.

75.

76.

77.

78.

136

Which one of the following is not a vestigial organ in man ? (a) Vermiform appendix (b) Plica semilunaris (c) Ear muscle (d) Epiglottis Foot in horse is : (a) Mesaxonic foot alone (b) Paraxonic foot alone (c) Both (a) and (b) (d) None of the above In his theory of evolution by Natural Selection Darwin had not taken into account the phenomenon of : (a) adaptations (b) over-production of members of a species (c) variations (d) mutations Cro-magnon was : (a) frugivore (b) carnivore (c) herbivore (d) omnivore Animals usually store energy supplying nutrients in the form of : (a) carbohydrates and lipids (b) protein (c) nucleic acid (d) vitamins The rate of oxygen consumption of an animal is equal to x at 00C. If Q10 is equal to 2, at which of the following temperatures the rate will be 8x? (a) 50C (b) 100C 0 (c) 30 C (d) 350C Most of the CO2 in blood is carried : (a) within erythrocytes (b) dissolved in plasma (c) as bicarbonate acid (d) in combination with plasma proteins About 80% of the glomerular filtrate is reabsorbed in the : (a) Proximal convoluted tubules (b) Distal convoluted tubules (c) Descending limb of loop of Henle (d) Ascending limb of loop of Henle Which of the following is C19 steroid hormone? (a) Progesterone (b) Estrogen (c) Cortisol (d) Testosterone Eutrophication refers to : (a) High production in an aquatic ecosystem (b) Low production in a terrestrial ecosystem (c) Stable production in a terrestrial ecosystem (d) Low production in an aquatic ecosystem The Taj Mahal is threatened by pollutant: (a) Chlorine (b) Sulphur dioxide (c) Hydrogen (d) Oxygen In a national park, protection is provided to : (a) Flora only (b) Fauna only (c) Both flora and fauna (d) Complete ecosystem YCT

79.

80.

81.

82.

83.

84.

85.

86.

87.

88.

89.

90.

Which is not a carcinogen ? (a) X-ray (b) HNO2 (d) Starch (c) Virus In submerged hydrophytes, which of the following is present ? (a) well developed sclerenchyma (b) well developed vascular tissue (c) Air spaces (d) cuticle Tropical dry deciduous forests can be found in: (a) Andaman Islands (b) Kerala (c) Eastern Himalayas (d) Madhya Pradesh Cell organelle containing hydrolytic enzymes is: (a) Lysosome (b) Microsome (c) Ribosome (d) Mesosome How many ATP are gained by complete oxidation of one molecule of glucose in green plants ? (a) 36 (b) 30 (c) 38 (d) 26 Nitrogen fixation is inhibited by : (a) Phosphate (b) Iron (c) Acetate (d) Ammonia The enzyme Nitrogenase contains a protein and the metals : (a) Fe and Co (b) Fe and Mo (c) Fe and S (d) Fe and Mg Which of the following has the largest flower : (a) Nymphaea (b) Helianthus (c) Rosa (d) Rafflesia The division in cleavage is (a) Meiosis (b) Mitosis without growth phase (c) Mitosis with growth phase (d) Amitosis A nutrient element essential for the formation of microtubules of the mitotic spindle apparatus during cell division is : (a) P (b) S (c) Ca (d) Zn Three important events in plants lead to speciation. What is the correct order of the events: (a) Mutation, Selection, Isolation (b) Isolation, Selection, Mutation (c) Selection, Isolation, Mutation (d) Isolation, Mutation, Selection The membrane enclosing cell vacuoles is known: (a) Tonoplast (b) Ectoplast (c) Endoplast (d) Plasma membrane

Practice Set-10

91.

Relationship in an ecosystem can be depicted through(a) Pyramid of energy (b) Pyramid of biomass (c) Pyramid of number (d) all of the above 92. Which of the following electron carrier in photosynthesis is mobile and carries electron from cyt b6 to PS-I through thylakoid lumen? (a) Cytochrome C (b) Ferredoxin (c) Plastocyanin (d) Plastoquinone 93. Azolla has become important in agriculture because it is being used as a : (a) biopesticide (b) herbicide (c) biofertilizer (d) insecticide 94. Cyclic photophosphorylation associated with : (a) Photosystem I (b) Quantasome (c) Pigment system II (d) Photolysis 95. During stomatal opening and closure, the CO2 sensor is located in : (a) Guard cells (b) Subsidiary cells (c) Mesophyll cells (d) Other epidermal cells 96. Red Data Bookes are compiled by: (a) WWF (b) IUCN (c) BNGS (d) WPSI 97. For isolation of protoplasts from leaf mesophyll cells, the enzyme mixture used containing mannitol is : (a) Cellulase + Hemicellulase (b) Hemicellulase + Macerozyme (c) Hemicellulase + Pectinase (d) Cellulase + Macerozyme 98. Desmosomes are concerned with : (a) Cytolysis (b) Cell division (c) Cell adherence (d) Cell excretion 99. Pyrimidine dimers in DNA are formed when the cells are exposed to: (a) Ultraviolet light (b) Cosmic rays (c) X-rays (d) Temperature 100. Which of the plant species represent the submerged stage of hydrosere? (a) Eichhornia and Trapa (b) Juncus and Corex (c) Typha and Rumex (d) Utricularia and Elodea

137

YCT

SOLUTION : PRACTICE SET- 10 ANSWER KEY 1. (c) 11. (a) 21. (c) 31. (b) 41. (d) 51. (c) 61. (d) 71. (a) 81. (d) 91. (d)

2. (b) 12. (b) 22. (d) 32. (b) 42. (c) 52. (a) 62. (b) 72. (c) 82. (a) 92. (c)

3. (b) 13. (c) 23. (d) 33. (c) 43. (b) 53. (d) 63. (d) 73. (c) 83. (a) 93. (c)

4. (d) 14. (a) 24. (a) 34. (a) 44. (a) 54. (d) 64. (a) 74. (a) 84. (d) 94. (a)

5. (c) 15. (d) 25. (b) 35. (c) 45. (b) 55. (a) 65. (c) 75. (d) 85. (b) 95. (a)

6. (d) 16. (a) 26. (b) 36. (d) 46. (b) 56. (a) 66. (b) 76. (a) 86. (d) 96. (b)

7. (a) 17. (a) 27. (c) 37. (d) 47. (c) 57. (c) 67. (d) 77. (b) 87. (b) 97. (d)

8. (d) 18. (b) 28. (c) 38. (d) 48. (a) 58. (b) 68. (a) 78. (c) 88. (c) 98. (c)

9. (b) 19. (d) 29. (b) 39. (c) 49. (d) 59. (c) 69. (d) 79. (d) 89. (a) 99. (a)

10. (d) 20. (b) 30. (c) 40. (c) 50. (d) 60. (a) 70. (b) 80. (c) 90. (a) 100. (d)

SOLUTION 1. (c) 7. (a) 2 Maximum range Given B = 1.5web / m u 2 2 gh v = 2 × 107 m / s and θ = 300 Rmax = = = 2h = 2 × 100 m , (u = 2gh ) g g Force on proton F = qv Bsin θ = 200m = 1.6 ×10−19 × 2 ×107 ×1.5 × sin 300 2. (b) 1.6 × 2 ×1.5 Microwaves are electromagnetic waves whose F= ×10−12 wavelength is of the order of millimeters. 2 3. (b) = 1.6 × 1.5 × 10−12 N = 2.4 × 10−12 N The electromagnetic damping experienced by a metal 8. (d) mass moving in a magnetic field is due to eddy current. James Maxwell in 1861 conceived the idea of 4. (d) displacement current. It produces current due to the In the case of alternating current (50 Hz), the direction change in electric field with time. It is similar to of current keeps changing every 1/100th of a second. conduction current but is not real as it does not describe Due to the changing direction of the current, the charges flowing through a region. metallic ions in the electrolyte keeps shunting between It is given by :the electrodes that keeps changing polarity. ∂D ID = J D ⋅ S = S ⋅ Hence AC current con not be used in electroplating. ∂t 5. (c) Where, S = area of capacitor Ferromagnetic materials are those which get strongly JD = displacement current density magnetized when kept in an external magnetic field. D = εE These material retain their magnetism even in absence of magnetic field. This property of ferromagnetic 9. (b) material is known as exhibition of spontaneous dσ  zz 'e 2    1 ≈   magnetization at the atomic level.   4 dη  16πε0Ε   sin ( θ / 2 )  • Paramagnetic materials are those which are weakly Scattering cross section (σ(θ)) for coulomb potential attracted to magnet. using born approximation is inversely proportional to • Diamagnetic materials are those which when kept in sin4θ/2. a magnetic field repels the magnetic field lines. 10. (d) • Anti ferromagnetic material are those which when The phenomenon of diffraction was first discovered by kept in magnetic field are weekly magnetised in the Francesco Maria Grimaldi in 1660, which was direction of applied magnetic field. established by Fraunhofer and Fresnel. 6. (d) 11. (a) Electric field varying with time can produce a magnetic Spherical Aberration (SA) is a type of departure of a field normal ray from normal path in case of lenses with since according to Faraday's electromagnetic law spherical surface. This aberration can be minimized by using a combination of convex/concave/ Plano convex −∆φ N∆φ e= ⇒− or Plano concave lens etc. For this the two lenses must ∆t ∆t Practice Set-10 138 YCT

be kept at a distance equal to difference in their focal length i.e.

The frequency of the unknown source will be 258 Hz because the frequency is common for both source. 18. (b) d = f1 − f 2 1 12. (b) Potential energy U = mω2 a 2 (1 − cos 2ωt ) 4 Power of the combination of lens, 1 P = P1 + P2 Kinetic energy K = mω2 a 2 (1 + cos 2ωt ) 4 P = +6 D + (−4 D ) = +2 D Therefore the kinetic and potential energies of the 100 Therefore the focal distance f = = +50cm system vary harmonically with frequency 2 ω. 2 19. (d) (convex lens) The thermal radiation is electromagnetic radiation emitted 13. (c) from all matter that is at a non zero temperature.The Let focal length of converging lens is f. wavelength range of thermal radiation is from 7700 Å to 4 Then according to question×106 Å. 1 1 1 1 20. (b)  1  = − = −  Ultrasonic waves are generated through Piezo–electric f v u D / 2  −D / 2  effect. The Piezo-electric effect is an electrochemical 1 2 2 4 property of certain material like quartz, where an = + = f D D D mechanical stress generate when they are placed under D electric current and vice versa. Piezo-electric material f= generates ultrasonic waves under pulsed current. 4 21. (c) 14. (a) It is a Lissajous figure with unequal frequencies. Critical angle If in the given question SHM eqn are represented by ∴ a ng 1 only sin and only cosine then the resultant of =n= a superposition of two represents ellipse. sinθ c nw 22. (d) 1 5/3 5 The vibration of body which take place under the = = sinθ c 4 / 3 4 influence of an external periodic force acting on it are called force vibration, similarly when body oscillates by 4 ⇒ θc = sin−1 being influence by an external period force is called 5 force oscillation, Ex-The vibration produced in the 15. (d) diaphragm of microphone sound box with frequency The cardinal point consist the three pairs of points corresponding to the speech of the speaker are the force located on the optical axis of a rotationally symmetric, oscillations focal, optical system. 23. (d) These are the focal points, the principal points and the In case of lift accelerading upwards nodal points. R − mg = ma then, R = m ( g + a ) • Nodal points are points arrange in way where a ray aimed at one of them will be refracted by the lens it Therefore, the apparent weight (R) of an object in a lift will be greater than its actual weight (Mg). appears to have come from the other. • Principal points are the points where the principal 24. (a) planes cross the optical axis. g' = g − R e ω 2 cos2θ • So a thin lens has 6 cardinal points whereas thick lens If the earth stops rotating, then ω = 0 that is, the value has two cardinal points. of g ′ will increase to g instead of decreasing at the 16. (a) Doppler's effect is the cause of the breadth of the equator. spectral lines. It is also known as Doppler broadening. It 25. (b) 3 occurs due to the thermal motion of emitting atoms or T1  r1  ions. =  T2  r2  17. (a) frequency = n ± beat 3 1/ 3 1.495 × 1011  365.4   r1  1 In the first case the unknown source produces 8 beats = ⇒       = r2 with a source of 250 Hz then the potential frequency  730.8   r2  2 = 250 ± 8 r2 = 21/ 3 1.495 × 1011 m = 242 or 258 In second case unknown source produce 12 beats with 26. (b) The weakest force of interaction is gravitational force a source of 270 then potential frequency = 270 ± 12 while the strongest force of interaction is nuclear force. = 282 Hz or 258 Hz

(

Practice Set-10

139

)

YCT

27. (c) A collision between two particles is perfectly elastic, when their linear momentum and kinetic energy are fully conserved. 28. (c) twisting a couple per unit twist of a cylinders is also called the torsional rigidity of the modulus. T=Cθ T=

πηr 4 θ 2l

C=

πηr 4 2l

29. (b) momentum · mv

1 2 mv 2 givenmomentum · kinetic energy ·

1 m v2 2 v = 2m / sec

mv =

kinetic energy

35. (c) This law which was first studied by Dalton in ''1804'' may be defined as follow ''When two elements combine to form two or more chemical compound, then the masses of one of the element which combine with a fixed mass of the other bear a simple ratio to one another. Ex – The masses of oxygen which combine with a fixed mass of carbon (12 parts) in carbon monoxide (CO) and carbon dioxide (CO2) are 16 and 32 respectively these masses of oxygen bear a simple ratio of 16 : 32 or 1 : 2 to each other. So one of the simple multiple proportions. 36. (d) The greater the number of electron - withdrawing substituents, the greater the dispersal of the negative charge and hence stronger will be the acid Hence, the order of acidity and -I effect is CCl3CCOOH > ClCH 2COOH > CH3COOH > C 2 H5COOH Carboxylic acids are weak acids because they only partially ionise in solution. Their solution do not contain many hydrogen ions than a strong acid solution at the same concentration. Acetic acid is ten times weaker than formic acid confirming the electron - donating character of an alkyl group relative to hydrogen. 37. (d) The position and velocity of the electron in the orbital are not the same to known, this is Heisenberg's uncertainty principle. According to this principle- "Of any moving tiny particle. It is impossible to determine both position and velocity simultaneously. 38. (d) K p = K c (RT) ∆n

30. (c) When a body move with constant speed in circular path with uniform speed has acceleration because direction of body changes every time so there is change in velocity at every point on circular path. 31. (b) Frenkel defect– This defect arises, when a cation leaving the lattice positionand assuming an Interstitial space. Takes the space between the interstitial lattice points is called frenkel defect. In this defect the crysta T = 300 + 273 = 573 remains electrically neutral. Because the number of All reaction cations in the crystal is equal to the number of negative PC l5 → PCl3 + Cl2 ions. ∆n = 2 − 1 = 1 32. (b) K p = (0.0625) × (0.0821× 573)1 In the auto ionization of water, a proton is transferred from one water molecule to another to produce a K p = 2.94 hydronium ion (H3O+) and a hydroxide ion (OH–). The equilibrium expression for this reaction is 39. (c) + – Lepidolite, also called lithia mica, the most common K w =  H 3 O   OH  lithium mineral, basic pottassium and lithium Where, Kw is the auto ionization constant for water. aluminosilicate, a member of the common mica group. It 33. (c) is economically important as a major source of lithium. The since colloidal solution of liquid in liquid is called emulsion not gel-Gels are colloidal solution of a liquid 40. (c) On heating of (NH4)2Cr2O7 produces green coloured dispersed in solid is called gel. powder of Cr2O3 which is blown in the air by the large 34. (a) volume of N2 and water vapour produced and settles like When a substance (volatile) is dissolved in a solvent when dissolved, the vapour pressure of the solvent dust from a valcano. decreases. As a result the boling point increase, vapour (NH 4 ) 2 Cr2 O 7 → Cr2 O3 + N 2 + 4H 2 O pressure of a liquid depends on its temperature, by increases the vapour pressure of a liquid it the 41. (d) atmospheric pressure is equal to (1 atm) then the liquid Tyndall effect has been used zsigmody for making an will boil looks like this means that there is a difference ultramicrocope. ultramicroscope is a microscope the between vapour pressure and atmospheric pressure arranged to that light illuminates the object from the side insted of from below. hence its boling points increase. Practice Set-10

140

YCT

42. (c) Ca C2 O4

Ca ++ + C2 O−− 4

∴ Ksp = [Ca 2 + ] [C2 O42 − ] Let us assume solubility of = S ∴ [Ca+2] = 1 × S [C2 O 24 − ] = 1 × S Given the Ksp = 2.5 × 10–7 so putting these values in the above relation, 2.5 × 10–7 = S × S = S2 ∴

S = 25 × 10−8 S = 5 × 10–4 g/L.

1 = –log [OH] or [OH] = 10–1 moles/litre Hence, 1000 ml contain = 10–1 mole So, 1 ml will contains = 10–4 moles The number of particles in 1 mole = 6.02 × 1023 The number of particles in 10–4 moles will be 6.02 × 1023 × 10–4 = 6.02 × 1019 Hence, the number of [OH–] in 1 ml solution of pH = 13 is "6.02 × 1019" 50. (d) An electron in an atom can be identified by four quantum numbers. An atom consists of many orbitals which are distinguished form each other based on their shape, size and orientation is space. Thus, Quantum numbers are those numbers that designate and distinguish various atomic orbitals and electrons present in an atom. There are four types of quantum number(i) Principal quantum number (n) (ii), Azimuthal quantum number (l) (iii) Magnetic quantum number (m) (iv). Electron spin quantum number (s) 51. (c) Reaction of benzoic acid with diazo methane give, methyl benzoate. C H COOCH 3+ N 2 C6 H 5 COOH + CH 2 N 2 → 6 5 Methyl Benzoate Benzoic acid diazomethane 52. (a) Naphthalene is oxidised with Na2Cr2O7/H2SO4 or with V2O5 gives thalic acid.

43. (b) OF2 Let the oxidation number of 'O' is 'x' OF2 x + (−1) × 2 = 0 x−2= 0 x = +2 Hence the oxidation number of 'O' in OF2 is '+2'. 44. (a) Law of constant proportion -: This law was discovered by a French chemist J.L proust and deals with the composition of element present in given compound. A chemical compound is always found to be made up of the same element combined together in the same fixed proportion by mass. Ex -: Pure water obtained from whatever source (well river lake or sea) or any country (India, Russia America) etc. will always be made up of only hydrogen NaCr2O7 / H 2SO4 and oxygen element combined together in the same → fixed ratio of 1 : 8 by mass. 45. (b) The half -life period of a reaction is 80 minutes at a Naphthalene Phthalic acid particular concentration when initial concentration of 53. (d) the reactant is reduced to half the life period is also reduced to half the order of the reaction would be The starting compound in case of Backmann rearrangement is an oxime. The acid catalysed second order of reaction. conversion of ketooximes to amide is known as the 46. (b) mann rearrangement. The Beckmann CH3OH has hydrogen bonds due to the very 'Beck rearrangement, named after the German Chemist electronegative O atom bonded to the H-atom. Beckmann. This rearrangement is occurs in both cyclic 47. (c) 1 and acyclic compounds. The electronic configuration of H atom is 1s O N – OH NH Atomic number of H is 1 NH2OH Acid – 2 So, electronic configuration of H = 1s O amid Oxime 48. (a) In electrochemistry and more generally in solution 54. (d) chemistry of a Pourbaix diagram also known as poential Reimer-Tiemann reaction pH diagram Pe – pH diagram is a plot of possible thermodynamacally stable phases - (i.e. chemical equilibrium) of an aqueous electrochemical system. 49. (d) pH of the solution = 13 Hence, pOH = 14 – 13 = 1 We know pOH = –log [OH] where [OH] = conc of OH– ions in moles/litre or molarity It's an example of electrophilic substitution reaction. thus, Practice Set-10

141

YCT

55. (a) 61. (d) The reduction of nitrobenzene in the presence of • Water is the most abundant molecule in cell, near Zn/Ammonium chloride gives N-phenyl hydroxy about 70% or more of total cell mass. lamine zinc dust and ammonium chloride convert • Cells are composed of water, inorganic ions, and nitrobenzene to the corresponding hydroxylamine. carbon-containing (organic) molecules. • Most of the organic molecules belong to carbohydrates, lipids, proteins, or nucleic acid. 62. (b) Robertson proposed unit membrane concept for biological membranes in 1959. According to this Nitrobenzene N-Phenyl hydroxylamine concept, the biological membrane is a lipid bi-layer Zn / NH 4Cl 2[H] surrounded on either side by proteins with a difference C6H5 NO2 + 2[H]  → C H NO  → C H NHOH 6 5 6 5 − H 2O Nitrisobenzene Phenyl hydroxylAmine in their type for the outer and inner side. The external 56. (a) layer is a hydrophilic layer made of protein molecules The silver salt method is a chemical method of of diameter 20Å-25Å. The middle layer is a light detection of organic acids. The principle of this method hydrophobic layer made of phospholipids of diameter based on the fact that most carboxylic acids from water- 25Å-35Å. insoluble silver salt when treated with ammonical silver 63. (d) nitrate. On heating, these salts decompose to leave a In the process of cellular respiration mitochondria residue of metallic silver. function as the main organelle. Mitochondria are known as the power house of the cell. They are rod – shaped RCOOH + NH 4 OH → RCOONH 4 + H 2 O organelles which convert nutrients into ATP. ATP is the RCOONH 4 + AgNO3 → RCOOAg + NH 4 NO3 energy currency of the cell which is produced by 2RCOOAg → 2Ag + 2CO 2 + R – R mitochondria. Aerobic respiration takes place in the powerhouse of the 57. (c) Mercaptan is made when sulphur reacts with Grignard's cell, i.e. mitochondria. Ribosomes are involved in protein synthesis. Chloroplast are concerned with the reagent followed by acidification. + photosynthesis. Dictyosomes perform the packaging and H 2O / H CH 3MgI + S → CH 3 − S − MgI  → CH 3SH + Mg(OH)I modification of the biomolecules. ( Methyl mercaptan ) 64. (a) 58. (b) In Prophase - 1 of meiosis segregation does not happen Aldol reaction is an example of an addition reaction and homologous pairs of chromosomes align at the carbonyl compound that have an H-atom on their α - metaphase plate does not occur during prophase I of carbon exhibit aldol condensation an this aldehyde or meiosis. ketone is obtained by product. In meiosis there are usually two steps, meiosis I and II. In Meiosis I, homologous chromosomes segregate, while in meiosis II sister chromatids segregate. 65. (c) G1 is typically the longest phase of the cell cycle. This can be explained by the fact the G1 follows cell division in mitosis, G1 represents the first chance for new cells 59. (c) Aluminium carbide when treated with water gives have to grow. Cells usually remain in G1 for about 10 hours of the 24 total hours of the cell cycle. methane gas. 66. (b) the reaction is given as follows. P and Q are gene frequencies of dominant and recessive Al C + 12H O  → 4Al(OH) + 3CH alleles respectively. P + Q = 1 is correct for Hardy 4 3 2 3 4 Weinberg law. This law states that the genetic variation Aluminium Aluminium Methane in a population will remain constant from one generation carbide Hydroxide to the next in the absence of disturbing factors. 60. (a) 67. (d) Thiophene > Pyrrole > Furan Thiophene has more resonance structure. Higher than Epiglottis is not a vestigial organ in man while all that of pyrrole and furan. It has more resonance energy vermiform appendix, plica semilunaris, ear muscle are than pyrrole and furan. Thus the aromatic property of vestigial organs. thiophene is high. However, the aromatic properties of Vestigial organs is a parts of human, plant and animal bodies pyrrole furan are similar. Nitrogen has more that do not have any clear function and are considered to be electronegativity than oxygen compared to nitrogen. residual parts from their respective ancestors. The negative pair of electrons is more easily represented 68. (a) on the five-membered ring in the furan than on the • Foot in horse is mesaxonic foot. Horses and their relatives run on one toe. Because this is the middle oxygen atom. Practice Set-10

142

YCT

toe of the five toes, it is referred to as a mesaxonic foot. The axis of the foot is formed by the middle digit. • When the axis of the foot is between the third and fourth digits, it is referred to as paraxonic foot. ex.– cattle, deer, camel, etc. [Note- In the book, the answer of the given question is (c)] 69. (d) Darwin Natural selection include following phenomenon; overproduction, struggle for existence, variation and heredity, survival of the fittest or natural selection (Adaptation) and origin of species. Mutation had not taken into account in theory of evolution by natural selection of Darwin. 70. (b) Cro Magnon were carnivorous and not herbivorous, frugivorous and sanguivorous. Like Neanderthals, the Cro-Magnons were primarily big-game hunters, killing mammoth, cave bears, horses and reindeer. They hunted with spears and with javelins and atlatl. Bow and arrow had not yet been invented. 71. (a) Animal often store and supply energy in the form of carbohydrates and fats. * Carbohydrates are a type of macronutrient found in certain food and drink. Sugar, starch and fiber are carbohydrates. * Body use fat as a fuel source and fat is the major storage form of energy in the body. 72. (c) The rate of oxygen consumption of an animal is equal to X at 00C. If Q10 is equal to 2, the rate will be 8x at 300C. The Q10 defines the temperature coefficient, that explains the factor by which the rate of a reaction (R) increases for every 10-degree rise in temperature. Q10 = 2 means the rate of the reaction doubles for each 100C rise in temperature. 73. (c) Transport of CO2 is maximum through bicarbonate. Nearly 20-25% carried out through RBCs, about 7% CO2 carried out in dissolved state through plasma. 74. (a) About 80% of the glomerular filtrate is reabsorbed in the Proximal convoluted tubules. Proximal convoluted tubule helps in the reabsorption of most electrolytes and water from the glomerular filtrate, independent of hormones. They are reabsorbed mostly by Na+K+ transport and water is reabsorbed due to the Presence of many aquaporins. The loop of Henle is responsible for reabsorption of Na+ and Cl– only 15% water is reabsorbed from the loop of Henle. Distal convoluted tubule regulates water and electrolyte absorption only in presence of the hormones, Aldosterone and Vasopressin. So, only fewer amounts of electrolyte and water are reabsorbed. 75. (d) Testosterone is a C19 steroid hormone. ⇒ A steroid hormone is a steroid that acts as a hormone. They are secreted by the adrenal cortex,

Practice Set-10

testes, and ovaries. C18 steroids consist of estrogen derivatives. C19 steroids consist of androgens derivatives, and C21 steroids consist of progesterone derivatives. ⇒ Androgens are a group of sex hormones, and testosterone is the most common androgen. Testosterone is the primary sex hormone in males and plays a key role in the development of secondary sexual characters. 76. (a) Eutrophication refers to high production in an aquatic ecosystem. It is the gradual increase in the concentration of phosphorus, nitrogen and other plant nutrients in an aging aquatic ecosystem such as a lake. 77. (b) The Taj Mahal is threatened by pollutant sulphur dioxide (SO2). Threat to Taj Mahal from Mathura refinery is due to pollutant gases like SO2, H2S and NO2. These gases convert calcium carbonate of white marble into calcium sulphate and calcium nitrate. Increased sulphur dioxide and nitrogen oxides in atmosphere are source of acid rain. Damage to Taj Mahal white marble is caused by acid rain. 78. (c) In a national park protection is provided to both flora and fauna. A park in use for conservation purposes is known as a national park. It is basically a reserve of natural, semi-natural or developed land that a sovereign state declares or owns. Flora and Fauna are provided immense protection in such areas. Currently in India, we have 106 national parks. 79. (d) X-Ray, HNO2, Viruses are a carcinogens. Carcinogen is an agent which has the capacity to cause cancer in humans. Starch is not a carcinogen but it is a Polysaccharide. 80. (c) In submerged hydrophytes, air spaces are found. Aerenchyma are the spaces or air channels in the leaves, stems and roots of Hydrophytes. It is an adaptation of hydrophytes, these air spaces provides buoyancy to the plants. • Cuticle : It is a thick layer present on the cells of Xerophytes to prevent the loss of water. 81. (d) Dry, tropical forests are found in Madhya Pradesh. 82. (a) A Lysosome is a membrane bound organelle found in nearly all animal cell. They are spherical vesicles that contain 30-35 hydrolytic enzymes that break down many kinds of Biomolecules. 83. (a) 36 ATP are gained by complete oxidation of one molecule of glucose in green plants. C6H12 O6 + 6O2 + 36 Pi2– + 36 ADP3– + 36 H+ → 6CO2 + 36 ATP 4– + 42H2O 84. (d) Nitrogen fixation is inhibited by Ammonia. With the higher amount of oxygen, the nitrogenase activity of the cells in decreased and strongly inhibited by ammonia.

143

YCT

85. (b) Nitrogenase is a complex enzyme, with the most common form containing two proteins termed the molybdenum iron (Mo Fe) protein (dinitrogenase) and the Fe protein (dinitrogenase reductase). These enzyme are produced by cyanobacteria (blue-green algae). These enzyme are responsible for the reduction of nitrogen (N2) to ammonia (NH3). 86. (d) Rafflesia is genus of parasitic flowering plants. It contains approximately 28 species all found in south east Asia, mainly Indonesia. Rafflesia is a species of flowering plant in parasitic genus. Rafflesia has the largest flower. 87. (b) When there is continuous mitosis in the zygote, it is called cleavage. ⇒ Cell size does not increase in cleavage because growth does not take place in the interphase stage, the growth phase is absent. Hence, there is no change in the size and shape of the embryo. 88. (c) A nutrient element essential for the formation of microtubules of the mitotic spindle apparatus during cell division is Ca. Calcium is required for normal process of cell division and also required for spindle formation. It is important for movement of Chromosomes during mitosis. • Calcium is present in cell wall as calcium pectate and forms middle lamella. 89. (a) The three important event in plants lead to speciation. So the correct order of the events are mutation, selection, isolation. 90. (a) The membrane enclosing cell vacuoles is known as Tonoplast. The tonoplast is the cytoplasmic membrane surrounding a vacuole, separating the vacuolar contents from the cells cytoplasm. It is also known as vacuolar membrane. 91. (d) A diagram showing the relationship between different trophic level in an ecosystem is called a pyramid and these relation to the pyramid of energy, the pyramid of biomass is represented by a pyramid of numbers. Pyramid of energy is always straight. Charles Elton gave the concept of pyramid. 92. (c) During electron transport e- from cyt b6 is carried to PSI by Plastocyanin. 93. (c) Azolla is a floating pteridophyte which contains as endosymbiont the nitrogen-fixing cyano-bacterium Anabaena azollae. Azolla has become important in agriculture because it is being used as a Biofertilizer. It has been used to control mosquito larva in rice fields. Practice Set-10

94. (a) Cyclic photophosphorylation lation is associated with photosystem-I (PS-I). It is the process in which organisms (like prokaryotes), just accomplish the converison of ADP to ATP for immediate energy for the cells. This type of photophosphorylation usually occurs in the thylakoid membrane. The electron begins in a pigment complex called photosystem-I in cyclic electron flow.

95. (a) During stomatal opening and closure, the CO2 sensor is located in the guard cells. Guard cells are bean shaped structure which surrounds the stomata. Guard cells form epidermal stomatal gas exchange valves in plants and regulate the aperture of stomatal pores in response to changes in the CO2 concentration in leaves. If CO2 concentration inside the leaf start to fall, the plant will open its stomata so that more CO2 can enter. 96. (b) The red data Books is compiled by IUCN (International Union for Conservation of Nature). The Red Data Book contains a list of organisms that are on the verge of extinction. These organisms are shown on Pink pages and as soon as an organism has increased in sufficient number it is transferred to the green pages. 97. (d) For isolation of protoplasts from leaf mesophyll cells, the enzymes mixture used containing mannitol is cellulase and macerozyme. Mannitol is a type of sugar alcohol which is also used as a medication. 98. (c) A desmosome are concerned with cell adherence or macula adherens. Desmosome is a cell structure specialized for cell to cell adhesion. It is a type of anchoring junction in animal tissue that connect adjacent cell. 99. (a) Pyrimidine dimers in DNA are formed when the cells are exposed to ultraviolet light. 100. (d) There are three types of hydrophytic plants. (I) Submerged plants–Hydrilla, Potamogeton, Vallisneria, Ceratophyllum, elodea, Utricularia (II) Free swimmers – Wolffia, lemna, Eichhornia. (III) Water terrestrial plants – Sagittaria, Ranunculus sceleratus, cyperus, typha etc.

144

YCT